Anda di halaman 1dari 101
No ‘This material is copyrighted. All rights reserved. Edward Goljan, M.D. 2002 HIGH YIELD NOTES PATHOLOGY & = —_USMLE pictures: sec handout describing high yield pictures in Robbin's Sixth edition = USMLE hematology pictures not in Robbins: * supravital stain showing reticul filaments, © coarse basophilic stippling in Pb poisoning, » x-ray with density in ict RNA poisoning, « ringed sideroblast, * acute progranulocytic leukemia lots of Auer rede aoa tig’, DIC, « gross of non-Hodgkin's lymphoma encircling trachea and arch vessels wenn et them, + micro of B cell follicular lymphorna, » multiple myeloma marrow with plaeme ale urine, + Reed-Sternberg cell, * apple green birefringence of amyloid, » back of hand of elderly patient with senile purpura, + arms of elderly patient with multiple bruises~ probably abuse midbrain with yellow discoloration of kemicterus, + fetus with hydrops fetal i 2 — USMLE cardiovascular pictures not in Robbins: » xanthelasma, » Achilles tendon xan pathognomonic of familial hypercholesterolemia, » turbid supranate (chylomicrons) and inffanate (VLDL), « stasis dermatitis of lower leg~ sign of decp venous thrombosis, « Osler-Weber-Read discase— telangiectasia on fingers and mucous membranes, + spider telangiectasia, « lymphedema post mastectomy, * Kaposi's sarcoma, « micro of small vessel vasculitis with fibrinoid necrosis, « thromboangiitis obliterans, * patient with Henoch-Schénlein purpura, * gross of renovascular hypertension due to atherosclerosis, * capillary hemangioma on the face of a newbom, « Sturge Weber~ port wine nevus in trigeminal nerve distribution, « gross and micro of pulmonary edema, « coronary artery+ * identify platelet thrombus, + fibrofatty plaque, + dystrophic caleification, © atherosclerosis (empty slits), + section through ascending aorta~ what part dilated with postductal coarctation? proximal aorta, « rupture of posteromedial papillary muscle in an AMI, * rupture of anterior wall of LV in an AML, + ruptured interventricular septum in patient with an acute myocardial infarction rupture occurs in site of coagulation necrosis and leads to right heart failure, © mural thrombus in an AMI, « fibrinous pericarditis, « old AMI with scar tissue on anterior wall- do not confuse with a pale infarct, « valve vegetations- * MV vegetations in rheumatic fever: along the lines of closure of the valve, * infective endocarditis: see destruction of valve, + marantic vegetations on MV: * similar in appearance to rheumatic fever, + Hx of colon cancer or ulcerative colitis, + Libman Sacks endocarditis: occur on MV in patients with SLE, + gross of mitral valve « prolapse, * micro of myocardium with lymphocytic infiltrate~ coxsackie myocarditis © — USMLE respiratory pictures not in Robbins: « x-ray with spontaneous pneumothorax (tracheal shift to side of the pneumothorax) or tension pneumothorax (tracheal shift to contralateral side), « gram stain of Streptococcus pneumoniae, * x-ray or gross of lobar pneumonia, * gram stain of filamentous gram positive bacteria— Actinomyces or Nocardia (partially acid fast), « micro of macrophage with Histoplasma yeasts, « gross of cavitary apical lesion of TB~ possible acid-fast stain of organisms, + micro of a macrophage with acid-fast bacteria— TB, * micro of broad based bud of Blastomyces, « micro of wide-angled non-septate hyphae of Mucor, « gross of lung abscess, + micro of non-caseating granuloma in sarcoidosis, « gross of Hamman-Rich honeycomb lung, * x- tay of classic COPD, + gross of main-stem primary lung cancer 7 2 USMLE gastrointestinal pictures not in Robbins: + patient with Herpes simplex, + exudative tonsillitis, hairy leukoplakia, « thrush with or without gram stain of pseudohyphae and yeasts, « gross of two tongues with leukoplakia— one tongue is squamous dysplasia and the other is invasive cancer (look for asymmetry of tongue), « lower lip with squamous cancer, + Peutz-Jeghers lips, « patient with mixed tumor of parotid, « gross or barium study of Zenker's diverticulum, * gross of esophageal cancer~ think squamous if mid-esophagus or adenocarcinoma if distal esophagus, * X° ray of classic small bowel obstruction with air-fluid levels and step-ladder effect, + gross of intussusception in a child, * micro of absent ganglion cells in rectal biopsy in Hirschsprung’s, * Note: This material is copyrighted. All rights reserved. Edward Goljan, M.D. 2002 i o jodysplasia in cecum, * gross of carcinoid tumor (yellow c. ! sen pte ce a ott cern tobiliary/pamcreas pictures not in Robbins: * eyes with jaundice. « ee tal cee tan eee ey eet eae | phagocytosis,» gross of liver echinococcosis, « gross or micro of obstructive jaundice Hees green, look for stone in common bile duct, » Kayser-Fleische ring in Wilson's disease ven’ | total copper is low, ceruloplasmin is low, and free copper is high, » patient with Dupmreen contracture, + xray showing calcifications in the pancreas in chronic pancreatitis, © meee pancreatic cancer of head of pancreas USMLE genitourinary pictures not in Robbins: + casts in urine~ hyaline (fuzzy and round borders) vs waxy (sharp borders), RBC cast, renal tubular cast, fatty cast with Maltese rose cclls in urine~ WBCs, RBCs, + crystals in urine uric acid, calcium oxalate, cystine crystals, « horseshoe Kidney, * gross or x-ray of duplicated ureter, + gross of bilateral polyeysicladney discasc in a child, * gross of adult polycystic disease with or without brain with thes © subarachnoid bleed (ruptured berry aneurysm) or intracerebral bleed (hypertensive bleed), « linear immunofluorescence- think Goodpasture's syndrome, * lumpy bumpy IF— think ype I immunocomplex glomenalonephritis, « EM pictures fusion of podocytes in lipoid nephrosis, subepithelial deposits (post-streptococcal, membranous), subendothelial (type IV. SLE, membranoproliferative), * micro of nodular glomerulosclerosis in DM look for hyaline ancriolosclerosis of afferenvefferent and pink balls in mesangium, "Christmas ball” disease, micro of ischemic ATN- look for coagulation necrosis of tubular cells, « picture of kidney and show where Hg or Pb damages— proximal tubule, « gross of staghom calculus, « gross or micro or renal infarction— look for pale, sunken areas on the cortex and coagulation necrosis, & gross of benign nephrosclerosis— granular appearing cortical surface, * gross of malignant hyperteasion— look for petechial lesions on surface of the kidney, * micro of foreign body giant cell reaction in tubules BJ protein reaction in multiple myeloma, « gross of prostate hyperplasia— bulging nodular surface with occlusion of the urethra, * patient with Klinefelter’s— look for signs of feminization, « patient with testicular feminization looks like a woman but they will give history of blind vaginal pouch, * gross of renal adenocarcinoma~ yellow mass with hemorthage, yellow mass extending up the inferior vena cava USMLE gynecology pictures not in Robbins; * Herpes genitalis with or without a Tzanck prep showing multinucleated squamous cells with intranuclear inclusions, « condyloma acuminata— looks like a fern, HPV 6/11, ¢ Chlamydia- cell with vacuole in the cytoplasm containing an elementary body, # cervix or penis with thick exudate of GC with or without gram stain showing gram negative diplococei in polys, « clue cell of Gardnerella vaginalis— squamous cell covered by bacteria, « primary syphilitic chancre on penis or labia~ + painless, + if painful look for Hemophilus ducreyi, « picture of lymphogranuloma venercum— C. trachomatis subtype, papules with draining sinus from nodes, lymphedema, * Candida— cottage cheese discharge with or without gram stain of Pscudohyphae and yeasts, * squamous cell carcinoma of vulva— large ulcer on labia, * gross of cervical cancer, « micro of squamous dysplasia, « woman with hirsutism- * increased testosterone indicates ovarian origin, + increased DHEA-sulfate indicates adrenal cortex origin, * ovary with blood in cyst— endometriosis, * gross of ruptured tube (small embryo) from ectopic pregnancy, © ‘goss of follicular cyst, * gross or micro of serous cystadenocarcinoma of ovary- look for Psammoma bodies, * micro of signet ring cells with mucin in ovaries~ Krukenberg tumor from Stomach metastasis, © gross of abruptio placenta, * micro of choriocarcinoma- look for multinucleated cells representing syncytiotrophoblast and clear cells representing cytotrophoblast, « ‘&0ss of fibrocystic change, * patient with Paget's disease of nipple, « patient with inflammatory carcinoma of the breast with peau du orange appearance Note: This material is copyrighted. All rights reserved. Edward Goljan, M.D. 2002 USMLE endocrine pictures not in Robbins: » patient with cicst visual field defect, which is bitemporal hemianopsia, = inte ears cyst ae age thyroglssal duct eysh «think branchial clet cyst if iis in the anterolateral nee eR * ticro of papillary thyroid eancer~ look for psamsmoma bodies, « patent sith Gre oe ise act cxophhalmos,» patient with Hashimoto's thyoidits~ lock fr penrbital por me thumb adducted into palm~ tetany, * micro of adrenal cortex/medulle ee, synthesizes mineralocorticoids, * ona fasciculata synthesizes, corti ‘synthesizes sex hormones, * adrenal medulla synthesizes catecholamines, * patient with Cushin; syndrome look for purple stra, « brown colored tumor in adrenal medulla pres id Patient with buccal mucosa hyperpigmentation— Addison's with increase in ACTH, © child with arebiguous genitalia~ 21 or 11 hytroxylase deficiency, foot with dry gangrene in'¢ mance coagulation necrosis, » retina with microaneurysms in a diabetic duc to somone damage of Pericytes, « well circumscribed tumor in adrenal cortex- could be Cushing's, Cons syndneen, oe non-functioning adenoma depending on the question = * USMLE musculoskeletal pictures not in Robbins: + synovial fuid erystls~ + yellow and parallel to slow ray is MSU in gout, * blue and parallel is calcium pyrophosphate in pseudogout, « chunky crystal in calcium pyrophosphate, « patient with osteoarthritis of hands Heberden's nodes 'n DIP joint « patient with zheumatotd arthritis in the hands, « patient with inflamed big toe_ acwee Sout, ¢ patient with tophus with or without microscopic showing multinucleated giant celle and Polarizable material, + patient with ankylosing spondylitis, « patient with Reiter's syndrome who has conjunctivitis with or without history of Achille’ tendon periosttis, « gross of osteomyelitis with or without picture of sickle cells + Salmonella osteomyelitis, * if no HX of sickle cell, Staphylococcus aureus is the answer, # young patient with inflamed knee with or without pustules on wrists or feet~ disseminated gonococcemia, » patient with Lyme's disease and rash of erythema chronicum migrans, » patient with blue sclera of osteogenesis imperfecta « defect in synthesis of ‘ype I collagen, * blue color due to loss of collagen in sclera and visualization of underlying choroidal veins, + gross of bone with osteoporosis, « MRI of the knee- know how to identify anterior and posterior cruciate ligaments, © gross of osteochondroma cartilage overlying bone Protuberance, * micro of muscle atrophy, * patient with Duchenne's muscular dystrophy, « patient, with myotonic dystrophy— * look for drooping mouth and frontal baldness, * trinucleotide repeat disorder, * patient with myasthenia gravis— look for drooping eyelids, « gross of lipoma, LE cell- neutrophil with phagocytosed red staining altered DNA, * seram ANA with rim pattem— SLE Patient with anti140 mg/dL to >126 me/dL) ‘on sensitivity, specificity, PV", and PV: ‘Note: This material is copyrighted. AIl rights reserved, Edward Goljan, M.D. 2002 1, increases sensitivity and negative predictive value (PV) — « dro the reference interval to a lower vale means that more people wine vee Upper limit of likely tobe TNs (not have DM) rather than FNS, » sensitivity and PY ene fet Ul ae the upper limit of a test reference interval is lowered YS increase when 2. decreases specificity and positive predictive value (PV) « fewer pe DM, a test result 7126 mg/AL is more likely to be a FP than Te as schematic It, © summary Normal _Disease 0 4 0 Interval O- 4 Sensitivity 100% (no FNs) PV 100% Specificity decreases PV" decreases —_—_——_> Interval 0-10 Specificity 100% (no FPs) PV" 100% Sensitivity decreases PV decreases = Prevalence: 1. Prevalence (number of people with disease in the population studied) = Incidence (number of new cases over a period of time) x Duration of the disease- P =I xD, «as duration (D) decreases, prevalence (P) decreases, « as D increases, P increases, » incidence (1) is. 2 constant in this relationship 2. prevalence calculation TP + FN (all people with disease)/ TP + FN + TN + FP (all people with and without disease} 3. example— if treatment for leukemia lengthens the survival period but does not Iead to its ‘cure, prevalence (P) of leukemia increases owing to the increase in duration (D)- no effect on incidence (aumber of new cases of leukemia) @ Example of a calculation for Sensitivity, specificity, PV’, PV, prevalence: Disease No Disease Rositive test (TP) 60 (FP) 40 ‘Negative test (EN) 20 (TN) 80 ‘Sensitivity of the test: TP / TP + EN = 60/80 = 75% Specificity of the test: TN / TN + FP = 80 / 120 = 66% PV": TN/ TN + EN = 80/ 100 = 80% (80% chance it is a TN and a 20% chance it is FN) PV: TP/TP + FP =60/ 100 = 60% (60% chance itis a TP and 40% chance it is a FP) Prevalence: TP + FN/TP + FN + EN + EP = 80 /200= 40% & Normal changes in pregnancy: |. greater increase in plasma volume than RBC mass + decreases hemoglobin (Hb) and hematocrit (Het) dilutional effect, « increases glomerular filtration rate (GFR) and creatinine clearance (CCr)- due to increased plasma volume, « decreases serum BUN/creatinine/uric acid- dilutional effect + increased renal clearance | Note: This material is copyrighted. All rights reserved. Edward Goljan, M.D. 2002 2. increased alkaline phosphatase~ placental origin respiratory alkalosis~ estrogen/progesterone increased clearance of CO; per breath 4. increased T, and cortisol- globulin, transcortin, respectively), free hormone. leven pid hyperthyroidisny’ hypercortisolism~ e.g., normal serum ‘* Main laboratory difference in adult male and female oy €.8., serum iron and ferritin, * lower Hgb concentration iy = Children: 1. imereased serum alkaline phosphatase (ALP) « 3-: release enzyme when stimulated by vitamin D 3.__ slight decrease in hemoglobin concentration whex % Newborn: high hemoglobin (Hgb) due to increase in Hgbr-- Left shifts oxygen dissociation curve (ODC)~ causes tissue hypoxia 2. stimulus for erythropoietin (EPO) release» i pprnsresses RBC production wich subsequent increase in Hgb concentration = HgbF: eat shifts oxygen dissociation curve (ODC) 2 Protects newborns with sickle cell disease « most of the RBCs at birth contain HOF inhibits sickling, « less HgbS- + concentration not high enough for sickling, + HgbS must be om in RBC for spontaneous sickling, « dactylitis (bone infarctions Of digits) begins in 6-9 ni 3 aly eabF synthesis is increased with hydroxyurea used to reduce sickle cell crises * Analytes increased with hetnolyzed blood sample secondary to veuipunctare. TDE« isoenzyme fraction is primarily increased and is greater than LDH, isoenzyme fraction (LDHy/LDH, flip), « false positive diagnosis of acute myocardial infarc” LDH, isoenzyme is also in cardiac muscle 2. potassium pseudohypcrkalemia~ false increase, « K*is the major intracellular cation, « ECG will not show a peaked T wave © Lipids and fasting: 1. triglyceride (1G) component coming from chylomicrons falsely increases serum TG levels chylomicrons contain diet-derived TG, hence the patient must be fasting wo eliminate this variable . 2% fasting or lack of fasting does not affect cholesterol (CH) and high-density lipoprotein CADE) concentration— » normally, CH is <3% of the chylomicron fraction, « fasting is unnecessary for an accurate CH or HDL 3. fasting is necessary for an accurate calculated low-density lipoprotein (LDL)- + LDL = CH - HDL - TG/5, if TG is falsely increased by chylomicrons from the dict, it will falsely lower the calculated LDL , . ‘Drugs eahancing the cytochrome system im the liver smooth endoplasmic reticulum (SER): 1. drugs alcohol, barbiturates, rifampin, phenytoin - 2 eftecton SER- + SER bye agi, reseed synthesis of y-glutamyltransferase (GGT)- enzyme is normally located in SER, * decreases drug levels owing to increased metabolism of the drug . . 3. elinieal scenario- woman on phenytoin who is also taking birth contro! pills gets pregnant: Phenytoin revved up the SER and increased metabolism of birth coatrl pill ‘© Drugs inhibiting cytochrome system in the liver: 1. drugs— Hy blockers, proton blockers Note: This 2 = Signi process material ls copyrighted. All rights reserved. Edward Goljan, MD. 2002 danger of drug toxicity ificance of erythrocyte sedimentation rate (ESR) in old 38°: probably indi , ICAteS & disease & Laboratory test alterations in alcoholics: 1 2 bor q Bee PS be hie fa enhancement of the liver cytochrome P-450 system. « ; — gintamylranferass (GGT) excellent enzyme marker fr rcaga liver disexee “S Of 1+ increased production of NADH in its metabolic breakdows causes biochem involving NADH to move in ies direction resulting in toe following ee hiram pyro ate © fasng eat oruvate is unavailable fr gloconespe so iy lyeeridemia~ 1,3 bisphosphoglycerate-> dih . bypereabie "aroxyacctone phosphate-> slyeert a, increase in ketoacid syuthesis— acety] CoA, the end luct of in the following reactions: acetyl CoA + acetyl Goa acne ne a ae acetoacetic acid increase in NADH converts it into 8 hydroxybutyric acid (BOHR) ae Acid syuthesis— due to the increase in acetyl-CoA, the initial substrate for hyperuricemia lactic acid/kctoacids compete with uric aid for ‘xeretion in the kidneys increased anion gap metabolic acidosis— lactate + B-OHB ratory test alterations in smokers: prsPiratory acldosis~ air gets in but cannot get out, so CO, is retained hypoxemia (low P20,) Increased carbon monoxide (CO) levels—-CO is present in cigarete smoke secondary polycythemia~ low PaO, stimulates eryehtopoietin wlan apsoine neutrophilic leukocytosis metabolites in smoke mebilize the neutrophil yee natng poo! in the cteulation by decreasing leukocyte adhesion fo eae vat cells Plasma/serum turbidity: due to an increase in triglyceride (TG)- turbidity does not occur with an increase in cholesterol (CH) in plasma 2 TG is carried by lipoproteins~ + chylomicrons 85%, « very low-density lipoprotein (VLDL)- 55% 3. TGis falsely increased after eating— due to diet-derived chylomicrons 4. chylomicrons form a supranate in plasma— contain very little protein: less dense than VLDL 5. VLDL forms an infranate (no supranate)- contains more protein than chylomicrons and does not float on the surface of plasma 6. _ imcreased turbidity interferes with measurement of enzymes and serum Na*~ falsely low enzyme values and sodium (pseudohyponatremia) Supranate Infranate (VLDL) (chylomicrons) 2. as ‘& Relation of serunt albumin concentration with Serum calcium concentration: 1. albumin binds 40% of total calcium in blood. « 13% of calcium is bound to other Substrates, « 47% calcium is free, ionized calcium metabolically active calcium i ‘ow serum albumin decreases calcium bound to albumin « hypocatcemia, « noteuny is Present, since the ionized levels are normal, « PTH is normal o o 94 owessan, MLD, 2002 Lyon's hypothesis: 1. one of the 2 X chromosomes in a female is chromosomes are matemal, « ~50% X chromosomes omes a Barr body- projection from the : eae from the buccal mucosa nucleus counted in s males have J Barr body and norm: Klinefelter syndrome (KXY. Toor Ene ae Barr bodies . female with Turner's syndrome (XO) has no Barr bodies : lolecular genetics: stic fibrosis— » deletion of 3 nucleotides coding for transmerabrane reguletor is defective and ie degaced bee ae aed CF after it leaves the Golgi appararus ing the cell membrane Tay Sachs— = 4 base insertion produces a frameshift mutation hexosaminidase 7 3. sickle cell disease/trait— point mutation involving thymidine . valine instead of glutamic acid in 6th postion of Pglobin chaig Pea? SERINE codes for 4. Betbalassemia major point mutation produces a stop codon leading to termination of DNA transcription of B-globin chain 5. trinucleotide repeat disorders— + progressively worse disease occurs in future generations (phenomenon is called anticipation), « constant repetition of 3 nucleotide bases (gs CAG, CAG, CAG etc.) « more trinucleotide repeats are added each generation leading to progressively worse disease and earlier manifestation of the disease, + examples Huntington's disease (AD), fragile X syndrome (SXR, female carriers with timeleoude repeats eventually become symptomatic), Friedreich's ataxia (AD), myotonic dystrophy (AD) Nondisjunction: + cause of the majority of chromosome number disorders (¢.g., tisomy 21, Tumer’s XO), * due to unequal separation of chromosomes in first phase of meiosis Mosaicism: # nondisjunction of chromosomes in mitotic division in the early embryonic period, « two chromosomally different cell lines are derived from a single fertilized egg, « most cases involve sex chromosomes—e.g., gonadal dysgenesis with XO/KX, XO/XY Translocation: * one part of 2 chromosome is transferred to a non-homologous or homologous chromosome, * called a balanced translocation if the translocated fragment is functional, Robertsonian translocation in Down syndrome: + type of balanced translocation with a reciprocal translocation between 2 acrocentric chromosomes (Ome), » usually chromosomes 21 and 14, * produces 1 long chromosome (14=21): extremely small translocated chromosome usually lost, # mother has the translocation: 45 chromosomes with 1 long #14+21, 1 normal #14, and 1 normal #21, + mother is normal, since both translocated fragments are functional, « Down child receives 1 normal #21 from uninvolved parent (father) and | 14*21 and 1 normal #21 from affected mother for a total of 46 chromosomes but all 3 #21s are functioning Ben “z = * codes for defective 21 2 4 14 2 2 “| TT It Parent: 45 chromosonies Down syndrome: 46 een em cat Microdeleton see a eras pom elcomosome can oly ieniied wh i . tion from 1 josome fue Microdeletion syndro: loss of a small portion 1 Oe ee high resolution techniques, + microdeletion on chromosot 5 Lsesaoien, syndrome~ * chromosome 15 deletion is of patemal_origin, obesity, ST ace retardation, + microdeletion on chromosome 15 may result in Angelman syndrome * < Note: This material Is copyrighted. AU rights reserved, Edward Goljan, M.D, 2002, 15 deletion is of matemalorigin. * “happy puppy” syndrome (chi cannot talk), « term applied to these syndromes in (child always happyfaughing but © Risk of recurrence of genctle disorders: Mendelian disorders have the recurrence: AR diseases have # 25% recurrence rate Sreatent risk of * Genetics disorders in African-Amerteans: « sickle cell rnitidixease— cell gene, © avp-thalassemia, * glucose G-phosphate dehydrogena hereditary persistence of UgbF? @ — Genetle disorders in Ashkenazt Jews: « factor XI deficiency, 10% prevalence of sickle se (GOPD) deficiency, « * Gaucher's disease, « Tay-Sachs discase © Genetic disorder ta Northern Europeans: cystic fibrosis~ MC genetic disease in patient's ability to. ronraduce owing to early death or problems with feriig netme ah the @ Genetic disorders in Mediterrancan peoples: + Ge ” Genetic aisorde peoples: * GORD deficiency,» sickle cell taivdisease,« pe Genetic disorder im Southeast Asians: o-thalasscmia MC genetic syndrome associated with advanced maternal age: trisomy 21 Down syndrome pathogenesis: trisomy 21- * 95% of all cases, origin for extra chromosome, Robertsonian translocation— + 4%, mother with 45 chromosomes- usually mother, 25-50 in 80% ‘* Down's clinteal findings: « epicanthal folds with upward slanting, simian paimar crease, « poor reflexes/nypotonicity, + cardiovascular * endocardial cushion defects (combined ASD and VSD), ing factor for survival in carly infancy and childhood, * GI- + duodenal atresia (polyhydramnios, vomits bile at birth, double bubble sign on x-ray), * Hirschsprung's disease , « hematologic~ increased incidence of leukemia, « CNS- * Alzheimer's disease, * chromosome 21 codes for B-amyloid proteins, which are converted into amyloid and are toxic to neurons, « universal by age 35 (any patient with Alzheimer’s discase under 40 is a patient with Down syndrome), reproductive + all males are sterile, + females have 50% chance of having a child ith Down’ © — Risk for future children with Down's: * 1-2% overall risk for trisomy 21, ¢ matemal age- women >35 yrs of age, + 5-15% risk for parent with a balanced translocation, © karyotype of affected child should always be determined to evaluate risk for siblings to have affected children * Trisomy 18 (Edward's syndrome): + scvere mental retardation, © clenched hands_with overlapping 2nd and Sth fingers, + rocker bottom feet © Trisomy 13 (Patau’s syndrome): * cleft lip/palate, + severe mental retardation, » polydactyly, * cystic kidneys = Mendelian disorders in descending order of frequency: * autosomal dominant (AD), * autosomal recessive (AR), * sex-linked recessive (SXR), * sex-linked dominant (SXD) = AD disorders: + only 1 abnormal allele is necessary to express the disease- + "dominant gene”, * €¢.g., aa (disease, usually lethal) or Aa (disease), © only on¢ parent has to have the gene to pass \ton to their children, * one affected parent + normal parent 50% normal children, 50% affected children * AR disorders: ¢ both abnormal alleles must be present (homozygous) to express the disease * 3% (disease), + Aa (heterozygote asymptomatic carrier), + both parents must have the abnormal allele= + 2 asymptomatic carriers (children- 25% normal AA, 50% Aa, 25% aa), * patient with the disease has children with an asymptomatic carrier (children 50% Aa, 50% aa) . 7 — SXR disorders: « males with abnormal allele express discase~ males are "homozygous", since he only have 1 X chromosome, © affected males transmit the disease to both daughters but none a their sons daughters are usually asymptomatic carriers, » female carrier transmits the disease 944 * 47 chromosomes, * maternal * 46 chromosomes in child, « + MC genetic cause of mental retardation 1Q Note: This material is copyrighted. All rights reserved. Edward Goljam, M.D. 2002 50% of the boys and 25% of daughters are asymptomatic 3 3 50% of males are affected, 25% of females are asymptomatic oe symptomatic- maternal and paternal X chromosome are abnormal s SXD disorders: males and heterozygous fernales both express the disease AD disorders: « associated with structural defects in proteins and receptors~ * enzyme deficiencies are uncommon, * AD disorders that are enzyme deficiencies acute interminent porphyna (deficiency of uroporphyrinogen synthase), * hereditary angioedema (C1 est taba deficiency), « late manifestations of disease~ c.g., Huntington's disease with chorea/dementia later in life, ¢ exhibit penewance- * person has the abnormal gene but never expresses the disease. however, the gene is transmitted to their children, * some AD disorders have 100% penetrance (€.g,, familial potyposis, adult polycystic kidney disease), while others have incomplete penetrance (cg., Marfan's), © exhibit variable expressivity affected people have different levels of severity of the disease, » mechanisms of AD disease without a family Hx: * MCC is incomplete penetrance, « new mutation, » examples of AD diseases (in order of decreasing frequency): + von Willebrand's disease + familial hypercholesterolemia, adult polycystic kidney disease, hypertrophic cardiomyopathy, + Huntington's disease, + neurofibromatosis, * congenital spherocytosis, * familial polyposis, * acute intermittent porphyria, + osteogenesis imperfecta, * Marfan syndrome = — Example of a pedigree with complete penetrance: 94 © — Example of a pedigree with incomplete penetrance: @ AR diseases: ¢ no evidence of penetrance, variable expressivity, late manifestations, « most (not all) AR discases are enzyme deficiencies~ inbom errors of metabolism- * acute intermiment porphyria and Cl esterase inhibitor deficiency are both autosomal dominant, * G6PD and Lesch- Nyhan are both SXR, « examples of those that are not enzyme deficiencies~ * cystic fibrosis, * sickle cell waiv/disease, * hemochromatosis, * Wilson's discase : Not o te: This material is copyrighted. Alll rights reserved. Edward Goljan, M.D, 2002 Example of an AR pedigree: [_} SXR disorders: examples (in order of decreasing frequency): © fragile X syndrome, « deficiency, * Duchenne’s muscular dystrophy, * hemophilia A/B, » sso cabin immunodeficiency, + Wiskott-Aldrich syndrome, * testicular feminization, » color bie s chronic granulomatous disease of childhood, «Bruton’s agammaglobulinemis ° Fragile X syndrome: » some geneticists say itis a sex-linked dominant disease, owing to the fact that female carriers may express the disease in future generations (concept of anticipation), = second MC genetic cause of mental retardation, « MC Mendelian disorder associated with mental retardation, * clinical- * mental retardation, * macroorchidism— at puberty, ¢ ~30% of female camiers are mentally retarded or have impaired leaming- due to anticipation and addition of Sinucleotide repeats with future generations, * abnormal fragile X chromosome- DNA analysis for carrier identification (identify CGG trinucleotide repeat is the best test to confirm) Lesch Nyhan syndrome: * SXR disease with a deficiency of HGPRT- no inhibition of PRPP in purine metabolism, « clinical- * hyperuricemia, * mental retardation, * self mutilation SXD disorders: « percentages of children with the abnormal allele are the same as those in SXR disorders: + dominant abnormal allele causes disease in both males and females, « affected woman transmits symptomatic disease to 50% of her daughters and 50% of her sons, + affected males ‘transmit symptomatic disease to all of their daughters and none of their sons, « examples: + familial hypophosphatemia (defect in the proximal reabsorption of phosphate and conversion of 25(OH)D, to 1,25 (OH)rD;), * Alport_syndrome (hereditary glomerulonephritis associated with nerve deafness) Multifactorial (polygenic) inberitance: © multiple small mutations plus the effect of environment- * should be suspected when there is an increased prevalence of disease among relatives of affected individuals, + parents and offspring have 50% of their genes in common, + enzymes involved in ndria— Calculation of the prevalence of 2 genetic disease given the carrier rate: , i (CF), with a carrier rate of CF is 1/25— * number of couples at risk is equal - Gene males x the carrier rate in females, or 1/25 x 1/25 = /625 couples are at risk, * risk of having a child with CF (AR disease) is 1/4, hence 1/625 x 1/4 = prevalence of ~1/2500 Simple way of calculating the carrier rate of a disease given the Prevalence of a genetic disease (Hardy Weinberg equation): + reflects the distribution of a mutant gene in the population, example using prevalence of CF of 1/2500-« number of couples at risk = 1/2500 + U4 = 625, + carrier rate of CF = 1/V625 = 1/25 ? Deformations: * anatomical defects resulting from mechanical factors (extrinsic forces) that usually occur in the last two trimesters after organs have developed, + e.g., oligohydramnios Producing facial and limb abnormalities (called Potter's facies) Malformation: * disturbance (c.g, drugs, infection) that occurs in the morphogenesis of an organ(s), + hypospadias faulty closure of urethral folds, + epispadias due to a defect in genital tubercle, « club foot, © ventricular septal defect Mechanismis of teratogens: « teratogens are most detrimental during the embryonic period~ first 9 weeks of life (4th-Sth week most sensitive for teratogens), * ¢.g., open neural defects occur when tube normally closes between the 23rd~28th day, * specific effects of some teratogens: * some interfere with formation of mitotic spindle, + interfere with production of ATP, some interfere with gene production (¢.g., isotretinoin effect on Hox/hedgehog genes, which are important in embryonic patterning) Teratogen cocaine: * maternal effects: + hypertension, + abruptio placenta, + newborn effects: * hyperactivity, * microcephaly (MC effect), * interruption of blood flow leading to infarction (CNS, bowel, missing digits) Maternal diabetes mellitas aud the teratogenic effects in newborns: © increased birthweight (macrosomia, large for gestational age): * hyperinsulinism in the ferus from poor maternal glycemic control increases muscle mass (insulin increases amino acid uptake in muscle), + hyperinsulinism increases fat deposition (insulin increases deposition of TG in adipose), * open neural tube defects, © cleft lip/palate, * respiratory distress syndrome: fetal hyperinsulinism in response to maternal hyperglycemia inhibits fetal surfactant production by type II pneumocytes . Teratogen diethylstilbestrol (DES): * mothers received DES to prevent threatened abortions, * DES interferes with the development of miillerian structures in female fetus causing abnormalities in the tubes, uterus, cervix, upper one-third of vagina, « female siblings: + vaginal adenosis is the MC abnormality and is the precursor of clear cell adenocarcinoma of the vagina/cervix, ¢ cervical 13 Note: This material is copyrighted. All rights reserved. Edward Goljan, M.D, 2002 incompetence~ increased incidence of spontaneous abortions, « ; with implantation, « fallopian tube abnormalities fertility problema nT Alities— problems = Fetal alcohol syndrome and teratogenic effects in newborns: « States: 2:1000 live births, + clinical + mental retardation (MC Sena a in United recardation, + maxillary hypoplasia, microcephaly, atrial septal defeets (Ieast o ine growth + hypoglycemia at birth -ommen finding), % Teratogen heroin: » srall for gestational age, » irritabilityyperactivty, excessive hunger, salivation, sweating, tremors, fist sucking, «temperature instability, « se 7 Teratogen isotretinoin: * used in treating cystic acne~ * must order a pregnancy corto Placing a woman on the drug, * patient must be on birth control pills while taking the aaice newbom effects (3 Cs): * craniofacial abnormalities (small ears), micrognathia, cleR paler,» cardiac defects, CNS malformations (hydrocephalus, microcephaly) palate), ‘7 Teratogen phenytoin: « nail hypoplasia, « CNS abnormalities, « cleft lip/palate, « ae ip/palate, * congenital heart 7 Maternal smoking: * vasoconstrictive effects of nicotine produce placental i = damage increases the risk for thrombosis in placental vessels low birth pues ee Mothers with SLE: SLE patients who have anti-Ro - i i have newboms with complete heart block {en SS-4) 1eG ansbodies in heir serum may * Teratogen thalidomide: + previously used in the United States to control nausea associated with pregnancy— currently used in Rx of leprosy, © limb abnormalities~ * amelia (absent limbs), + phocomelia (seai-like limbs) = — Teratogen valproate: open neural tube defects © Teratogen warfarin: « contraindicated in pregnancy: should use heparin, * CNS defects, « nasal hypoplasia Congenital infections: « TORCH syndrome: + Toxoplasmosis, + Other (HBV, AIDS, parvovinus, syphilis, etc.), + Rubella, + CMV (cytomegalovirus), + Herpes, « increase in IgM in cord blood, + vertical transmission (mother to fetus)~ * transplacental MC type, * blood contamination during delivery(c.g., HBV), * cervical infection (HSV 2, Chlamydia), + breast feeding (HIV, HBV, CMV) Congenital CMV: + MC_in-utero viral _infection: majority are asymptomatic, * primarily transplacental, « clinical~ + bilateral sensorineural hearing loss MC complication, + periventricular calcification, + hepatosplenomegaly, chorioretinitis (may lead to blindness), « urine culture is gold standard- urine cytology reveals large, basophilic intranuclear inclusions (“owl eyes”) in renal tubular cells, * Rx- * ganciclovir (begin with this), + foscamet (if the former is not working) ‘Congenital rubella: « primarily transplacental— highest incidence of congenital anomalies in first 8 weeks- virus interferes with protein synthesis and produces a vasculitis, » clinical- * sensorineural deafness MC complication, * cataracts, * patent ductus arteriosus Congenital toxoplasmosis: « primarily transplacental, + contracted by women after exposure to cat litter * pregnant women should avoid cleaning cat liter during pregnancy, * contracted also by handling or eating undercooked meat products, * greater risk of fetal infection later in pregnancy than earlier in pregnancy, © clinical- * chorioretinitis MC late complication (often leads to blindness), « calcifications in basal ganglia, « Sabin Feldman dye test (uses live organisms)}- "gold standard" test but is rarely performed Congenital Herpes type 2: + primarily contracted by passing through the birth canal in women actively shedding the virus- * women actively shedding the HSV-2 virus are delivered by C- section, * greater chance of fetal infection with primary rather than recurrent Herpes, « clinical~ local or systemic disease (encephalitis, skin infections), « Rx: acyclovir * high pitched cry with Congenital syphilis: * primarily transplacental— fe Brcenaiey~ anatomical borers prevent access tothe Teal caine ee SOM of (first 2 yrs): * hepatomesaly MC sign, * osteochondritis (inflammation often, ie lesions, + pneumonia alba (lobar pneumonia), persistent rhinitis (muffle), lane Se (2 yrs: + frontal bossine_is MC sign, + saber shins, + rhagadet (cenieal nen ils Murhinsons id eth (necked upper ental incisor called Hutchinson's tech nd nels molars called mulberry molars), + interstitial keratitis (blindness), * sence Rx: penicillin > ™ 1 loss, « Congenital Varicella-zoster virus: + chorioretinitis— cortical atrophy in the brain, « vesicular skin lesions Age dependent disorders (Inevitable with age): + increased body fat: decreases insulin receptors leading to glucose intolerance, * res} howe ore jiratory— * ob: ‘ pulmonary function tests (so-called “senile apliiand jgheneers alti Cee expiration), * decreased forced expiratory volume in 1 second (FEVise), * decreased forced vital capacity (FVC), * increased total lung capacity (TLC), + increased residual volume VY), * low normal PaO;, « cardiovascular * loss of elasticity in aorta, + decreased cardiae output ard heart ate in response to sttess, * at rest, the cardiac output is unchanged, * musculoskeletal Osteoarthritis in weight bearing joints, *CNS— + cercbral atrophy with mild forgetfulness, = impaired sleep pattems (insomnia, early wakening), Parkinsonian-like gait, « sensory changes- * cataracts, * arcus senilis (ring of cholesterol around the comea), + presbyopia (inability to focus on near objects), * presbycusis (sensorineural hearing loss particularly at high fiequency), « otosclerosis (fusicz. of the ear ossicles producing conductive hearing loss), + immune system « decreased T cell function (often anergic to common skin antigens, positive PPD non-reactive or absent), + loss of isohemagglutinins to blood antigens (may not develop a hemolytic transfusion reaction if blood is mismatched), « skin changes + loss of skin elasticity (increased cross-bridging of collagen), + senile purpura over the dorsum of the hands (common USMLE picture) and lower legs where bumping into objects is most likely to occur, + decreased sweating (eccrine glands fibrosed, danger of heat stroke), # GE tract- + decreased gastric acidity, + decreased colonic motility (constipation which predisposes to diverticulosis), + decreased activity of the hepatic cytochrome P450. system(danger of drug toxicities), + male reproductive + prostate hyperplasia (increased urine residual volume with subsequent increase in urinary tract infections), * prostate cancer (only cancer that is age dependent), * decreased testosterone,+ female reproductive— » breast and vulvar atrophy due to decreased estrogen, * increased gonadotropins, « endocrine system— increased glucose intolerance (due to increase in body fat and subsequent reduction in insulin receptor synthesis), © renal changes~ decreased GER with reduction in the creatinine clearance (risk of drug toxicity due to slow clearance of drugs), Age-related changes (increased incidence in elderly but not inevitable): + cardiovascular- atherosclerosis with increased incidence of coronary artery disease, peripheral vascular disease, aad strokes, * temporal arteritis, * aortic stenosis (MC valvular abnormality in the elderly), * systolic hypertension due to loss of aortic elasticity, « musculoskeletal system— « osteoporosis (particularly the vertebral column in females and femoral head in males), * Paget’s disease of bone, * respiratory system- pneumonia usually Streptococcus pneumoniae (underscores importance of Paeumovax vaccination in elderly), « CNS— * Alzheimer’s disease, * Parkinson's disease, + strokes (atherosclerotic type is MC type of stroke), * subdural hematomas (falls cause tearing of bridging veins in subdural space leading to a venous clot), + sensory changes- * macular degencration (MCC of blindness in elderly), « skin disorders * UVB light-induced cancers, * actinic (solar) keratosis (common USMLE picture, precursor of squamous cancer), © endocrine system- type Il diabetes mellitus Potential for blindness, » limb hypoplasia, 15 : This material is copyrighted. All rights reserved. Edward Goljan, M.D. 2002 Top 5 causes of death in males and females regardless of age and sex i denendlag ‘order: * heart disease, « cancer, © stroke, » Sate shame we neuet Shienin (Corb), « motor vehicle accidents~ MC COD in males and females between 1-39 yes or ge Top 3 causes of death in children aged 1-14 yrs in descending order: * accid see MVAs, « falls 25-30%, » bums 10-15%), « cancer, « congenital anomalies Se Top 3 risk factors leading to increased morbidity/mortality in the United States in desce order: « cigarette smoking, * dietary factors and activity patterns: + ding diet, + lack of exercise, « aleohol abuse 'y partcms: « high sanurated fat, low fiber Gunshot wounds: « contact wounds~ contain soot and gunpowder in the wound (called fouling), intermediate wounds powder tattooing (stippling of skin around the entrance site), « distant wounds no powder tattooing, * exit wounds are larger and more irregular than entrance wounds Motor vehicle accidents (MVAs): * MCC accidental death between 1-39 yrs of age: commonly alcohol-related, particularly in teenagers, « seat belts and air bags have reduced morbidity/mortality Drowning: « common COD in children from I-14 yrs of age, + near drowning: survival following asphyxia secondary to submersion, © wet drowning: * 90% of cases, * initial laryngospasm on contact with water> rclaxation/aspiration of water, * fresh/salt water drowning: whether fresh or salt water drowning, surfactant is destroyed in lungs-» atelectasis with intrapulmonary shunting» diffuse alveolar damage and initiates spasm in the bronchioles, immediate COD in drowning is cardiac archythmia Burns: « first degree bums: * painful partial thickness bums (¢.g., sunburn), * produce ceil necrosis limited to the epidermis, + heal without scar, + second degree bums: + painful partial thickness bums, * iavolve entire epidermis,* form blisters within epidermis, heal without scarring unless they are deep, © third degree bums: painless full thickness bums, + extensive necrosis of epidermis/adnexa, * extensive scarring complicated by keloid formation (propensity for squamous cell carcinoma), * healing comes from residual epithelium at the margins of the bum and from adnexal structures, « COD: infection due to most commonly to Pseudomonas aeruginosa followed by Staphylococcus aureus, * house fires: * smoke inhalation MC COD, + carbon monoxide (CO) and cyanide poisoning commonly occur (source of cyanide is polyurethane in upholstery) Heat injuries: + hyperthermia: core body temperature >37.2°C, heat cramps: * afebrile, * common in untrained athletes/laborers who become volume depleted, * lose excess amounts of saltwater, « heat exhaustion * mild elevation in core body temperature (539°C), * common in athletes training in ho/humid environment, * develop severe volume depletion, * exertional type of heat stroke: # core body temperatures 2 41°C, + people working or running on het day, * skin hovdry, * profound lactic acidosis, + rhabdomyolysis common, + non-exerti stoke- * elderly/chronically ill patients, * skin hot/dry without sweating (hypohidrosis), * lactic acidosis/rhabdomyolysis uncommon, « malignant hyperthermia~* AD disease, * defect in calcium release channels in the muscle sarcoplasmic reticulum, + massive muscle contractions with extremely high temperatures after induction of anesthesia by halothane and succinylcholine (muscle relaxant), * Bx with dantolene, * screen family members with muscle biopsy and caffeine/halothane contraction test on muscle, * body temperature and redness of skin for patient walking briskly on a hot day- + no increase in rectal temperature, + vasodilatation of vessels in skin producing redness, * choice D in schematic, marathon runner on a hot day~ * increase in rectal temperature (probably heat exhaustion), * vasodilatation of vessels in skin producing redness, * choice Cin schematic AB c Rectal T A D Vasodilatation 16 9 44 q q 4q Note: This material is copyrighted. All rights reserved. Edward Goljan, M.D. 2002 Cold injuries: « frostbite: + localized tissue injury secondary to direct (ice crystallization in eels) cells) and indirect damage (vasodilatation/thrombosis), + tissue eS B hypothermia: * core body temperature <35°C, « tt exposed te teecag nig prolonged period of time, * uncoupling of oxidative phosphorylation, + venous pooliny wae Progress into circulatory failure and death ig that may Electrical injury: + Ohm's Law: ¢ current (1, amps) = voltage (E)/resi: 6 EE (HULGNR (olin, Semceat emmase knpeRiant seaerin eee temo direct current (DC) risk for electrocution, «resistance: * dry sin has the highest tissue resistance to current, particularly the hands and feet, * wet skin lowers the resistance to current, since voltage ig a constant, lowering resistance increases current, « ing from the left i is most dangerous- * involves the heart-> ventricular fibrillation, « blood is an excellent conductor of current, « COD: cardiorespiratory arrest with ventricular fibrillation and respiratory paralysis Jouizing radiation: * cxamples- y-rays, x-rays, particulate radiation emitted by radioactive substances (@/B particles), shorter the wavelength the greater penetration- + low penetration include /3-particles, * high penetration include y-rays, « radiation injury: direct or indirect injury to DNA, * indirect type produces hydroxyl free radicals from hydrolysis of water in the tissue, = DNA is most susceptible protein, » tissue susceptibility to radiation + directly related to degree of mitotic activity with the greatest effect on actively mitosing cells, + indirectly related to degree of tissue specialization (see below), * cell cycle— * peak sensitivity is in G, (synthesis of tubulin for mitotic spindle) and M phase (assembly and disassembly of mitotic spindle), * § phase Icast sensitive, + G, phase intermediate sensitivity, © tissues with a high radiosensitivity— * hematopoietic cells are most affected (e.g., lymphocytes > granulocytes > platelets > mature RBCs), * germinal cells in the testes, ovaries, « tissues with low radiosensitivity « brain is most resistant to radiation, * bone, * mature cartilage, * muscle, « total body irradiation: + hematopoietic system first system affected (greatest overall effect» lymphopenia (first change)» thrombocytopenia» bone marrow hypoplasia), « vessel changes thrombosis (early), fibrosis (late, ischemia), * skin changes including erythema cdema- blistering chronic radiodermatitis> potential for squamous cell carcinoma, * GI tract with diarrhea Ultraviolet light (non-ionizing radiation): « UVA- Wood's lamp (black light), ¢ UVB- + sunbum, * corneal bums from skiing, * mutagenic effect on skin (thymidine dimers), « UVC- germicidal, * UVB-related cancers- * basal cell carcinoma is MC skin cancer, * squamous carcinoma (precursor is actinic/solar keratosis), * malignant melanoma Laser radiation: intense localized heat that is equivalent to a third degree bum Microwave radiation: « produces a skin bum, * adversely affect pacemaker devices, * inconclusive complications— * cataracts, * cancer, + sterility Infrared radiation: * bums, + cataracts Bligh altitude it general- * O, concentration is 21% (normal), * decreased barometric pressure (e.g. only 200 mm Hig on top of Mt. Everest), + lation is essential at_high altitude since it lowers alveolar CO (respiratory alkalosis) > automatically increases alveolar PAOp-> increases arterial PO2, © acute mountain sickness: * occurs within the first 24-36 hrs of an ascent above 8000-10,000 feet, + headache, lethargy, insomnia, dyspnea, + Rx with descent, increase fluid intake (increased insensible water loss from mucous membrane evaporation), and ‘oxygen, * prevention by acclimatizing before ascending and using acetazolamide, which is 2 carbonic anhydrase inhibitor that produces metabolic acidosis compensation for the expected respiratory alkalosis, » high altitude pulmonary edema: non-cardiogenic Changes after death: rigor mortis is due to decrease in ATP in muscle Sites of alcohol (ethyl alcohol, ethanol) reabsorption: ¢ small intestine- 75%, » stomach + 25%, « partially metabolized by alcohol dehydrogenase 17 Note: This material is copyrighted. All rights reserved. Edward Goljan, M.D. 2002 = Liver metabolism of alcohol: alcohol dehydrogenase aldehyde deh ‘ Sige at relia Wate” R me eee aN) Pharmacologic action of alcohol: + CNS depressant in descending order> oe seenylcon limbic system-» cerebellum-> lower brain stem, * potentiates inhibitory neurowansmittere He aminobenzoic acid, * aldehyde dehydrogenase deficiency: * affects ~40% of Asians, = aie ays acetaldehyde and GI upset (similar to action of disulfiram), * alcohol toxicinws sy ne euphoria, gregarious, + 100 mg/dL, legally crunk in most states (slurred ee mat speech, uncoordinated), + eens mg/dL. stupor or coma, * >500 mg/dL. death, + delirium temens- « following occur 3-5 ys aftr complete witraval-» wemblousnes, disorientation, visual hallucinstions, agaton, + Diseases where alcobol is the leading cause: © thiamine deficiency: o Korsakoff’s psychosis, congestive cardiomyopathy, « macrocytic eee ieee acquired sideroblastic anemia— microcytic anemia with ringed sideroblasts, « Mallory Weiss syndrome: tear of the distal esophagus/proximal stomach from retching, + Boethaave's syndrome: rupture of the distal esophagus/proximal stomach from retching, © cimthosis, « esophageal varices— effect of portal vein hypertension due to alcoholic cirrhosis, « fatty change in the liver, » hemosiderosis— alcohol increases the reabsorption of iron, * acute and chronic pancreatitis, © type IV hyperlipidemia alcohol increases synthesis of VLDL, + Klebsiella pneumoniae pneumonia ‘Alcohol as a cancer risk: squamous carcinoma (synergistic with smoking): oropharynx, mid- esophagus, larynx, » adenocarcinoma: pancreas, liver Alcohol effects on CNS/PNS: + Wernicke’s syndrome/Korsakoff's psychosis, « cerebellar degeneration: Hu and Yo antibodies noted in spinal fluid, « dementia, « DTs, » distal peripheral neuropathy, © central pontine myelinolysis: demyelination syndrome due to rapid IV Rx of hyponatremia Smoking epidemiology: * MCC of premature death in the United States, * MC single preventable cause of cancer, * incidence of smoking is increasing in women and decreasing in men, nicotine intake can be monitored by measuring plasma or urine level of cotinine~ cotinine is only derived from the metabolism of nicotine, « MOA of nicotine: + absorbed rapidly into the pulmonary circulation, * moves into the brain where it attaches to nicotinic cholinergic receptors to produce its gratifying cffects/complication of smoking, * highly addictive agent, + inactivation of the p-53 suppressor gene by a point mutation on chromosome 17 is the MC genetic defect in smoking induced cancer Cancers where smoking is the leading cause: ¢ hmg cancer: squamous, small cell, and adenocarcinoma to a lesser extent, * MCC of death due to cancer in both men and wornen, * oral pharyngeal and laryngeal and mid-esophageal squamous cancer, * pancreatic adenocarcinoma, © transitional cell carcinoma of bladder, + renal adenocarcinoma, Cancers where smoking has been implicated but is not the MC risk factor: » cervical cancer~* squamous cancer, * carcinogens found in cervical secretions, * stomach adenocarcinoma, « breast adenocarcinoma primarily in women who are slow acetylators of N-acetyltransferase 2 enzymes, * prostate adenocarcinoma, * colon adenocarcinoma, * leukemia~ increased risk of both lymphoid and myeloid leukemias, » alcohol is a cocarcinogen with smoking that further enhances the risk of oropharyngeal, esophageal, laryngeal cancers, ¢ smoking + asbestos exposure markedly enhances the incidence of primary lung cancer no association of smoking with mesothelioma, whether the patient is a smoker or not, lung cancer is the most common cancer associated with asbestos exposure 18 Note: Ths materia s copyrighted, All igs reserved. Btward Goljau, M.D. 2002 7 Smoking effects om the cardiovascular/CNS recurrent AMI as well, increases risk for sudden cardiae death, vascular disease, + increases risk for strokes, « contributing factors, to chemicals in smoke and nicotine effect on blood pressure and fy Profile, + tssus hypoxia secondary to excess carbon monoxide (CO), = Smoking effects on the respiratory system: « COPD. 80% ey all i e 4 of all cases, chronie bronchi Emphysems, » recurrent infections--Pocamonia, URIs, «exacerbates bronetny abe cee 7 Smoking effects om the Gl system: GERD, «delays the rate of uses heaigg Fisk for oral, upper and lower Gil cancer, « USMLE scenario advice to piven Rismryot ant, ulcer disease: stop smoking = sevoker th Hits 7 Effects af smokeless tobacco (enuff, chewing tobacco): * nicotine addion leukoplakia/cancer— inside the lip, under the tongue or cheek. « vernicous sausmaus canes cea cancer snuff users, « aggravation of cardiovascular disease~ nicotine oats Smoking effects on bone and menopause: » increases the risk for ssteoporosis in men and women, # biochemical reaction in women— * estradiol (most metabolized in the liver into estrone, which is metabolized into AaMIO) oF estriol (strong estrogen actvity)-> smokers have greater conversion of ouene nee an, inactive metabolite leading to tow estriol levels-> low levels of estrogen incieace the nue we osteoporosis and premature menopause ‘* Passive smoking effeet on children: « pathogenesis of passive smoke effects~ * ~ 75% of tot combustion product in a cigarente is exhaled, » risk of passive smoke extends to children as well adults, « increases the incidence of SIDS, » increases risk for lung canccr- 1-2 times inmeacea ases risk for otitis media, * increases risk for recurrent upper and lower respiratory infections Miscellaneous smoking effects: + increases risk for developing proteinuria in diabetes mellinis, directly responsible for ~25% of residential fires, ¢ vitamin C deficiency 2 Beneficial effects smoking cessation: « longevity— smokers who quit before 50 yrs of age have half the risk of dying over the next 15 ‘yrs than a smoker has, * lung cancer- + in 10 ‘yrs, there is a 50% reduction in lung cancer when compared to a smoker, after 15 yrs, there is only a 16% risk for Jung cancer when compared to a smoker, « AMI- AMI risk approaches that of a nonsmoker after | y# of abstinence, * pregnancy— pregnant women who stop smoking in the first trimester reduce the tisk of a low birthweight baby to that of a nonsmoker, « forced expiratory volume in 1 second (FEV) * it is not improved by cessation of smoking, * rate of decline is similar to that of a non- smoker Drugs and interstitial pulmonary fibrosis: * amiodarone, « bleomycin, © busulfan, « cyclophosphamide, » nitrofurantoin, « nitrosourea, * methysergide— also retroperitoneal fibrosis and Raynaud phenomenon, « methotrexate, + procarbazine Occupation exposure relationships: + automobile mechanic carbon monoxide, * pesticide industry- * organophosphates, * arsenic, ¢ meat packing— polyvinvl chloride with risk of hepatic angiosarcoma, + insulation/demolition/roofing material- + asbestos: lung cancer, mesothelioma, fibrous pleural plaques (MC overall complication of asbestos), + formaldehyde, dry cleaning carbon teirachloride with liver necrosis due to free radicals, « rubber/chemical industry * benzene: aplastic anemia, leukemia, + aniline dyes: bladder cancer, « battery, smelter, plumber/foundry— lead poisoning, painter- + methylene chloride: converted into carbon monoxide, * solvents, * lead, © petroleum- * benzene, + polycyclic hydrocarbons: lung cancer, « sewer worker~ hydrogen sulfide gas: sulthemoglobinemia Isopropyl alcohol (rubbing alcohol) poisoning: ¢ metabolism * metabolic end-product in the liver_is acetone: no metabolic acidosis unlike other alechols, + increases serum osmolal gap: 19 VETHe 4 G44 q q differmnee between calculated and measured serum osmolality >10, « etinial- Methyl alcohol (Wood's alcohol): © increased anion i formic acid), © optic neuritis and potential for blinnicans Toke v Sins Converted into competitive antagonist with methyl alcohol for alcohol dehydrogenase ANCE: ethancl is a Ethylene glycol (antifreeze): « increased anion gap metabolic acidosis (con * renal failure from calcium oxalate crystals obstructing the lumens, « Rec is a competitive antagonist with methyl alcohol for alcohol dehydrogenase Mercury poisoning: * MOA- * toxic in inorganic (elemental) fori: dental amalgan used in hat making industry (‘mad hatter disease"), + toxic in organic fom oo contaminated fish, + clinical~ + diarrhea, * visible on x-rays, + nephrotoxic ATN mene broximal tubules, + cerebral/cerebellar neuron loss, * constricted visual fields, = Rec deseo Arsenic poisoning: * sources + pesticides, + animal dips, « Fowler's solution, » Ra af sey 1930's, « clinical * garlic odor to breath, » severe diarthea: "rice water" stools similas ie ele seni melanosis: wry in with dark macules, « Hl carcinoma o} = nails have transverse (Mees nails): concentrates in keratin/hait/nails, © j ; MC COD, * nephrotoxic acute tubular necrosis involving proximal alae lieve ar Rx- dimercaprol NSE Saeiosarcoma, Mushroom poisoning (Amanita): « MOA— toxin inhibits RNA polymerase, © : pain/vomiting, « bloody diarthea, « jaundice: extensive fatty change re eee Petroleum product (gasoline, Kerosene) disorders: » euphoria (drunk acting) when inhaled (or ingested), » addicting, « toxic doses— + convulsions, + tinnitus, « non-cardiogenie pulmonary edema Strychnine poisoning: « MOA- CNS stimulant that blocks postsynaptic inhibition, clinical (Similar to tetanus)- + tetanic convulsions, + opisthotonus, + risus sardonicus, + death Poisonous snake envenomations: = types~ + pit vipers: rattlesnakes (MC bite), water moccasins, copperheads, * true cobras: coral snake (neurotoxin that blocks acetylcholine release-> paralysis and death, similar to botulinum poison), coral snake has following color banding “red and yellow Kill a fellow", harmless scarlet king make~ "ted and black friend of jack", » pit viper cavenomations~ + local swelling/necrosis, + hematologic problems: DIC, + antivenin is available: danger of serum sickness CDyo: common acute lymphoblastic leukemia antigen (CALLA) Kid staring into space that has broken bones: absence seizures Testicles in hernia sac: * testicles are not translucent, * translucency in the scrotal sac implies a hydrocele Dock worker for 20 yrs with lung mass: primary jung cancer secondary to asbestos exposure from asbestos insulation around pipes in ships Belly full of scars: acute intermittent porphyria due to deficiency of uroporphyrinogen synthase Flame hemorrhages ia the retina: sign of hypertensive retinopathy Visual field defect with craniopharyngioma: bitemporal hemianopsia (impinges on optic chiasm) Stacks of hemoglobin when it the blood is deoxygenated: HgbS Patient with normal PT, PTT, bleeds after surgery even after receiving fresh frozen plasma: probable platelet problem~ ¢.g., thrombocytopenia, patient on NSAIDs Cavitary lesions in upper lobe: « MCC is reactivation TB (not primary), + histoplasmosis, * Klebsiella pneumoniae, squamous carcinoma . Well-differentiated squamous cell carcinoma of lung or any other tissue: * keratin is red, ¢ look for squamous pearls Patient with Marfan's syndrome has diastolic murmur: aortic regurgitation due to stretching of aortic valve ring by the dissection deep coma with verted into oxalic acid), with TV ethanol: ethanol 20 Note: w8 oy ¥¢ 994Gq 4 q a4 ‘This material is copyrighted. All rights reserved, Edward Goljan, M.D. 2002 Xray Gindings in osteogenic sarcoma: mass in met i % muscle, * upper tibia or lower femur, + x-ray~ « Codman'e eee invasion imo surrounding made by osteoid, « Rb suppressor gene relationship + * sunburst appearance of bone X-ray fladings ia Ewing's sarcoma: onion skinning around bone shaft Young woman with episodic hematuria after upper res absent uroporphyrinogen synthase, « belly full of scars Mone ‘with maculopapular rash during pregnancy and neonate with saber shins congenital Baavte infarct of brain in patient with atria! fibrillation: embolic infarct "atient with twitching of face with tapping of facial nerve: « tetany calcium threshold potential is (GE ca Blears are partially sea eee ‘gn, * Trousseau’s sign is when thumb adévcts into palm when taking blood preseare joe ares not develop ‘ = transfusion reaction when given wrong ABO roup: elderly patients normal} isohemas attack BOantger ineckootml ieslutnins with age, so none may be presentto Screen for autoimmune disease in relatives: screen for HLA antigens unique to the autoimmume disease or serum ANA depending on the way the question is worded Source of Legicrelia infections: + air conditioning cooling tawersicondensers, * showers, « vegetable misters in grocery stores, + produces interstitial nephritis with type TV renal mibulr acidosis: desuruction of JG apparatus with hyporeninemia and kypoaldosteronicen Patient taking penicillin for streptococcal infection develops hemolytic anemia: due to sutoimanune hemolytic anemia (type TI) against BPO attached to RBC membrane Epidural hematoma: » fracture of temporoparictal bone and rupure of middle meningeal arery Malabsorption with blunt celiac disease, « order anti-gliadin or endomysial antibodies EK disease from brain instrument treated with formaldehyde: prions are the infective agent “Bronze diabetes”: hemochromatosis 40 yr. old man with dementia and senile plaques in brain: Down syndrome patient with Alzheimer's Easy bruising in hospitalized patient on antibiotics: vitamin K deficiency from destruction of colon bacteria by the antibiotic Pancreatic tumor associated with peptic ulcers: ZE syndrome with secretion of gastrin Multigucleated giant cell associated with viral infection: measles~ Warthin-Finkeldcy giant cell Mechanism of cerebral edema in Pb poisoning: « increase in 8-aminolevulinic acid in brain is toxic and produces demyelination and increased vessel permeability, can be prevented by taking heme, which inhibits ALA synthase Alcoholic with bad breath and cavitary lung lesion: + lung abscess from aspiration of ‘oropharyngeal material, * mixed aerobes and anaerobes Main site for acetaminophen toxicity in liver: « around terminal hepatic venule (centrilobular) least amount of oxygen in this site (zone IM), « also the site for fatty change in alcoholic or shock Biock left renal vein: © produces a left-sided varicocele, + left spermatic vein normally empties into the left renal vein mechanism for varicocele, « the right spermatic vein ermpties into TVC Injury to head with polyuria: central diabetes insipidus from transection of pituitary stalk Supine and where does foreign body go: superior segment of RLL 21 Note: This material is copyrighted. All rights reserved, Edward Goljan, M.D. 2007 va 994 a4 a4 In a patient who is lying ou the right side, a foreign bod: commonly localize to these primary sites in the nag SHER a penterioe segment middle lobe, » right upper lobe [a a patient who is lying on their back, aspirated a ‘ segment of the lung: right lower lobe superior segment VOUE Most likely localize to this Lymph node: « B cells in follicles, « T cells in parafoliicul . Patient with an increase in amylase and lipase: « pation mase ReneS Siuses Pancreatitis, « cannot be mumps because of lipase Mely an alcoholic with acue Staghorn calculus: « magnesium ammonium hate, = alkaline urine pH Phosphate, © urease uropathogen like Pro‘eus, « Overdose of barbiturates: acute respiratory acidosis Bye effects in Cushings syndrame: cataracts from increased glucocorticoids Flank mass in a child: © Wilms tumor, « associated i 4 ia ine ssociated with aniridia and. hemihyperrophy in the AD What makes a laboratory test more specific?: raise the upper I orm i specificity (less FPs) and positive predictive value per limit of normal, which increases Findings at autopsy of RDS: * hyaline membranes with massi ys eaBloctas eats massive intrapulmonary shunting ive atelectasis, » atelectasis leads to Baby bleeds after circumcision on 10th day in a mother who i Baby Deeds atts: dr is breast feeding her baby: no Abetalipoproteinemia: + absence of apo B leads to low CH levels, « malabsorption, « all lipid fractions, + blindness, « hemolytic anemie, «treated with vitamin PN EEO Sudden onset left flank pain, hypotensive, pulsatile mass: i Saddea sunita sib puls Tuprured abdominal aortic ancurysm palpitations particularly when anxious: + mitral valve prolapse, + myxonatous degeneration due to an increase in dermatan sulfate Click murmur relations in mitral valve prolapse: « dectease preload causes click and murmur to come closer to SI- + anxiety, * standing up, * Valsalva, « increase preload causes click and murmur to come closer 10 $2- + lying down, + clenching fist (increases systemic vascular resistance and decreases cardiac output), squatting (increases systemic vascular resistance) Aortic stenosis: MC valvular lesion associated with~ + hemolytic anemia with schistocytes, syncope with exercise, * angina with exercise 28-year-old patient has a family history of sudden cardiac death at a young age: hypertrophic cardiomyopathy Chambers or vessels with the highest SaO, in Tetralogy of Fallot: « pulmonary vein, « left atrium Chambers or vessels has the lowest SaO, in a newborn with a machinery murmur (PDA): « right atrium, « right ventricle Shuats or groups of shunts cardioprotective in Tetralogy of Fallot: « patent ductus~ unoxygenated blood goes to PA for oxygenation in lungs, « atrial septal defect— step up of oxygen in the right heart Viral myocarditis, develops hypotension, ueck vein distention, a drop in blood pressare 03 inspiration, and muffled heart sounds: pericardial effusion from coxsackie myocarditis~ Bx of heart would show a lymphocytic infiltrate with destruction of muscle First step in management of pericardial effusion: echocardiogram Newborn baby girl, swelling of dorsum of bands/feet (lymphedema), cystic mass in neck (dilated lymphatics that stretch skin and produce webbed neck): + lymphedema in child with Tumer's syndrome, + XO, « defects in lymphatics 22 yr old, ADDS, raised, red lesions on hard palate: + Kaposi's sarcoma, « most common site for KS in GI tact, due to fellatio, « Rx with intralesional o-interferon ertal would most - umiterial Is copyrighted. All rights rererved. Edwarg Goljan, M.D. 2002 ‘$8 yr old smoker, painless jaundice with clay colored stool! in z urobilin in stool), painful varicosities in ‘ight upper Shoe aren sat ee fem i 2 few weeks ago: » migratory thrombophlebitis ine patient with carcinoma of the head of ete et with obstruction of bile flow, called Trousseau's sign ofpancreas Patient with diarrhea and episodic flushi, "16 yr old died suddenty playing basketbat, i hypertrophy of the interventricular seprum with conduction defects, (worse) with factors decreasing preload (standing, Valsalva, venodilator), « murmur intensity Gecreases (better) with factors increasing preload (Sustained hand a blockes) SAP, squatting, lying down, B. ‘& Pulmonary capillary wedge pressure readings (normal < 12 mm Hg) expect bypovol shock (volume depletion) versus an acute myocardial infarction ino ee eae (increased hydrostatic py is neuroblastoma, © an APUD 2ye old girl with necrotic, bloody, grapetike mass reveals malignant cells with cross-striations; tumor si NTL related antigen, and carcinoembryonic antigen, and positive for desmin; cancer is most likely derived from: « muscle embryonal shabdomyosarcoma, * MC sarcoma in children * 6S-yr old woman with abdominal distention, Primary cancer most likely responsible for induration in pouch of Douglas on rectal exzmuinatica, overian cancer induration due to seeding by the cancer 7S-yr old man with point tenderness in the lower vertebral column, elevated serum alkaline Phosphatase (osteoblastic metastasis), tests ar procedinc would be your first step in the cvaluation of this patient: digital rectal exam think cheap, + bone tenderness implies stage IV disease and DRE should be positive Metastatic adenocarcinoma in left supractavicular node (called Virchow's node) would most Ukely have its primary origin in: stomach adenocarcinoma ‘* 1S-yr old girl, sudden onset of right lower abdominal Pain, pregnancy test negative, cystic paass with bone and calcifications in the right ovary; cystic teutone * Pediatric cancers into the order of decreasing frequency: acute lymphoblastic leukemia—> primary CNS tumors—> Burkit’s lymphoma» neuroblastoma = Most comunon ionizing radiation-indaced cancer: acute leukemia An ulcerated lesion that develops in a keloid Secondary to third degree burn or an ulcer tains negative for cytokeratin, factor ———--~ located..at the orifice of » chronically raining sinus that does not respond to medical Hanaeement is most likely due to: development of a squamous cell carcinorms ‘* Hematuria in a S8-yr old smoker would mast likely be associated with which of the following TrouPs of cancers: renal adcnocarcinoma/transitional cell carcinoma of the blades * Most responsible for the increased incidence of basal cell carcinoma ta the United States: sun exposure beginning at an early age ; % — Exudative lung reaction associated with exposure to moldy hay: Farmer's lung~ thermophi actinomycetes 23 ee ee ee ee * Young man presents with hemoptysis and then develops acute Goodpasture's syndrome, + anti-basement membrane antibodies, « type I ie . © — Hypersensitivity pneumonitis that is primarily seen in textile workers: byssinosi 7 Bypersensitivity pneumonitis that commonly occurs in farmers who enter a a it fermenting corn: silo filler's disease— nitrogen dioxide room with F _ Pneumoconiosis whose most common lesion is a benign pleural plaque: asbestosis A pregnant woman in her 32nd week of gestation has following drugs should she be given to protect her aby from develogiey aie eee ot te distress syndrome: glucocorticoids increase surfactant eynthesis in the baby ey as a yt old woman with gheenls Lesiichns develops episodic asthmatic attacks. The most cause is: ingestion of non-steroic angi . iat i-inflammatory agents with release of leukotrienes_ © Farmer and his wife are brought to the ER by their son because they are too weak to walk or drive and their vision is blurry and exam reveals ptosis, facial weakness, nonreactive dilated Pupils, dry mucous membranes, and normal DTRs—? diagnosis: « C. bonulinum food poisoning, Rx with botulism antitoxin, danger of respiratory paralysis toxin blocks the release of acetylcholine * Young woman with intermittent houts of diarrhea and constipation associated with cramping right and left lower quadrant pain: + irritable bowel syndrome— intrinsic motility defect in bowel, « flexible sigmoidoscopy is negative Intravenous drug abuser in prison—?type of hepatitis: HBV with outbreak of hepatitis: HAV most likely Hepatitis associated with urticaria, fever, arthralgias, and the nephrotic syndrome: « HBV, « serum sickness type of disease~ type IN immunocomplex mechanism, + vasculitis associated with polyarteritis nodosa = Immunizations given at birth to a baby whose mother is positive for HBsAg: * Hep B vaccine- active immunization, + HBIG- passive immunization = — Ina patient with By: deficiency who is being treated with pharmacologic doses of folate, which of the following will be corrected; * neurologic deficits remain, hence the importance of making the correct diagnosis, ¢ megaloblastic anemia is corrected © Im an alcoholic with a macrocytic anemia and hypersegmented neutrophils and a normal neurologic exam, which of the following tests is most indicated: RBC folate is more sensitive than serum folate * A 4 yr old child has eaten rat poison and is hemorrhaging: « rat poison is warfarin, which blocks all the vitamin K-dependent factors, « both PT end PTT are prolonged, © Rx with IM vitamin K * — Child living with elderly grandparents develops a GI bleed: child ate grandparents warfarin = A child has eaten raw hamburgers and now has a hemolytic anemia and renal failure: « HUS due to 0157: H7 serotype of E. coli, + low platelet count and prolonged bleeding time A woman has menorrhagia, easy bruising, and epistaxis: « VWD, « prolonged bleeding time and prolonged PTT = A man bas a family history of a bleeding disorder which began with bis mother's father: + hemophilia A, + matemal father transmits the disease to all his daughter's (SXR trait) who are asymptomatic carriers, « the daughters transmit the gene to 50% of their sons, prolonged PTT © A449 yr old woman with fibromyalgia is taking NSAIDS for pain: qualitative platelet defect, « prolonged bleeding time Major crossmatch: patient scrum reacted against donor RBCs, * does not guarantee survival of the infused donor RBCs, * does not prevent patient antibodies developing against donor RBC antigens, « detects the presence of patient antibodies against donor RBC antigens Note: This material is copyrighted. Allright reserved. Edward Goljan, M.D. 2002 A phlebotomist inadvertently sticks himself wit with AIDS. Which of the following infections is ¢) 1:300 chance of becoming HIV positive A phiebotomist inadvertently sticks himself with a needle after drawing b Which of the following infections is the phlebotomist at most risk Sireteag ee the greatest ial load in blood ofall vines, «you cannot get syphilis fom tance bee Y Te Most common antibody encountered in clinical practice: it alse impossible to find bi that is negative for antibodies against CMV ee ‘Hemorrhagic skin necrosis associated with warfarin therapy: BY: heterozygote C become homozygote when given warfarin in ~6-8 his when the enn ccarboxylated protein C disappears now the patient is hypercoagulable A patint who bas chronic bepatids and has been transfused ia the past most Likely has antibodies directed against: HCV is the MCC of transfusion henesit 28 yr old woman bas an anterior mediastinal mass and non tender lymmphad. Fight supraclavicular node: nodular sclerosing Hodgkin's disease, ™PNAG@ROPatby In the An afebrile 72 year old man with won-tender, generalized lymphadenopathy hepatosplenomegaly, normocytic anemia, thrombocytopenis and 1iypticieglhelioene! most likely has: « CLI is. B cell malignancy, « neoplastic B cells cannot transform into plasma cells Rationale for beginning heparin and warfarin at the same time: © the shortest are factor VIO and protein C and the langest is prothrombin, = previously activated vitamin K-dependent factors have long half-lives A.7S year old woman in a nursing home has nou-palpable ecchymoses limited to the back of her hands. A CBC is unremarkable, The patient most likely bas: senile purpura Most common type of hereditary thrambosis disorder: factor V Leiden cannot be degraded by protein C and S Most common infection transmitted by a blood transfusion is due to: « CMV is Present in donor lymphocytes, + radiation of the blood kills the lymphocytes and prevents infection in the recipient « bone marrow transplant patents are particularly prone to CMV infections in the lings Which of the following vitamin toxicities predisposes a patient who is taking warfarin to bleed and have an INR outside the normal range: vitamin E toxicity decreases the synthesis of vitemin K dependent factors in the liver A weightlifter develops a sudden onset of 2bdominal pain along with bypovolemic shock. At surgery, his abdominal cavity is filled up with blood. The cause of the intraabdominal bleed is most likely associated with: anabolic steroids cause liver cell adenomas, which have 2 tendency to bleed A 30 year old man with acquired immunodeficiency syndrome has pitting edema, hypertension, proteinuria > 3.5 g/24 brs, and fatty casts in the urine. A renal biopsy exhibits glomerular disease: focal segmental glomerulosclerosis A 29-year-old woman presents with generalized pitting edema and hypertension. A renal biopsy reveals diffuse glomerular disease with increased thickness of basement membranes and bypercellularity. Silver stains reveal "tram track" splitting of the basement membranes. An electron micrograph exhibits "dense deposits” in the glomerular basement membranes. The patieat has a persistently low serum C3: * type II MPGN or “dense deposit disease”, Patients have C3 nephritic factor- autoantibody against C3 convertase in alternative system that causes it to continually activate C3 causing very low levels A 25-year-old man, who initially presented to the hospital with hemoptysis, has progressed into renal failure. A renal biopsy shows linear immunofluorescence.: Goodpasture's syndrome patient has rapidly progressive crescentic glomerulonephritis, 2 needle after drawing blood from a pati ¢ phlebotomist at most risk for contracting: 25 A 74-year-old man with colon cancer develops generalized an: . 24 by aed Cutty enets fn hiscorine. A renal biopsy shows dite gloat > 3.5 stains demonstrate epimembranous spikes and the presence of subepithelial wena electron microscopy: diffuse membranous GN leposits. on A 10 yr old boy living in Salt Lake City, Utah bas bilateral sensori 4 ir ity ig Su Lae CD, Ua bs ha sence ot visceral epithelial cells: Alport's syndrome sex-linked dominant disease notedin A65 yr old man with chronic HCV hepatitis has a history of cyanosis of hi suid cars durtug cold weather: These findings subside when tae comes betoora ee palpable purpura in cold-exposed areas: cryoglobulinemia s A 12 year old bas a history of cellulitis ~2 weeks ago. He now pr: Vl pectoris byperdcadiem, and sueley colored uring, Urinslpsi shows Ge casts Chl ce (15 g/24 h). "Renal biopsy reveals a diffuse increase in cellularity along with a neutrophilic inflerate, IF shows granatar deposits Subepithelial deposits are noted on EM: acute pot streptococcal GN- immunologic reaction secondary to either ee or skin infection may 3 group A strep Pharyngitis A 62-year-old woman with a long-history of severe rheumatoid arthritis and restrietive cardiomyopathy now presents with pitting edema and hypercholesterolemia. Urinalysis shows oval fat bodies and fatty casts. Protein dipstick is 2+, sulfosalicylic acid 2+. Renal biopsy shows hyalinization of the glomerular mesangium. A special stain is order: amyloidosis ‘An 89-year-old woman with a history of chronic HCV hepatitis from a blood transfasion now presents with generalized puffiness and mild hypertension. Urinalysis shows oval fat bodies, fatty casts with Maltese crosses, and 4+ protein (> 3.5 g protein/24 hr), A renal biopsy reveals hypercellular glomeruli with an increase in basement membrane thickness and tram-track splitting of the basement membranes. The IE is granular and EM reveals subendothelial deposits, The patient has depressed levels of C3: type I MPGN with HCV association A 24 year old man with a prior history of an upper respiratory infection is noted to have microscopic hematuria and mild proteinuria (1.5 9/24 brs) during a routine physical exam. Be is not hypertensive. A renal biopsy shows a granular IF with predominantly IgA deposition in the mesangium and electron dense deposits in the same area: ig glomerulonephritis : A 48-year-old worvan has proteinuria (> 3.5 gm/24 hrs), bypertension, and fatty casts in the urine, A renal biopsy reveals eosinophilic nodular masses in the mesangium of the glomeruli, hyaline arteriolosclerosis of the afferent and efferent arterioles, and increased thickness of the basement membranes of the tubules. IF is negative. EM reveals fasion of the podocytes and increased collagen deposition in the mesangium and basement membranes: nodular glomerulosclerosis in diabetes mellitus A 28 yr old man, with a history of renal disease in his maternal grandfather, has end-stage renal disease requiring dialysis. Physical exam reveals papular red lesions on his skin, peripheral neuropathy, and corneal disease. Previous renal biopsies exhibited vacuolated visceral epithelial cells which correspond with lamellar bodies on EM: Fabry's discase~ SXR disease with deficiency of a-galactocerebrosidase A and accumulation of ceramide trihexoside A normotensive 10 yr old boy, with a previous history of an upper respiratory infection, presents with generalized pitting edema. He has had problems with allergies since early childhood. Urinalysis reveals proteinuria (> 3.5 9/24 hrs), fatty casts, and oval fat bodies. His clinical findings improve rapidly with high dose corticosteroids: lipoid nephrosis A 25 yr old woman presents with hypertension. A urinalysis reveals mild proteinuria, hematuria, and RBC casts. A serum ANA exhibits a rim pattern. An anti-dsDNA titer is extremely high: type IV SLE glomerulonephritis |: systemic 26 Note: a4 te ‘Ths tera is copyrighted. All ights reserved, Edward Gaia, Mp. 2002 A child with Tarner's syndrome is noted to have an abo horseshoe kidney,» lower pole fusion,» behind the inferior meneie ene Patient with Wegener's granclomatosis develops hematuria seg + wansitional cell carcinoma, * patient is on eyclophomphaennns cystitis and TCC. " Ax Feyptian man bas microscopic hematuria and an abnormal atine cytology: squamous c arcnoma: patient has Schitzosoma hematobium involving the bladder viene 7 “ An uncircumcised male bas an utcerative lesion on the undersurface of the zlans peais and palpable inguinal adenopathy: « squamous cell earcinoma (penis) = hak of ‘ireunocision is greatest risk factor note the bilaterality A Pacene mith acute myelogenous leukemia developed acute renal failure after being, aggressively treated with multiple chemotherapy agents; « urie acid crystals « example of tumor lysis syndrome Crystal associated with ethylene glycol poisoning: calcium oxalate inborn error of metabolism associated wich renal stones and hexagonal crystals inthe urine: cystine eystinuria Gephlication associated with sickle cell trait, acate pyelonephritis, analgesic abuse, and diabetes mellitus: renal papillary necrosis Depression related to = vitamin deficiency: niacin deficiency comclation with decreased tryptophan conversion into serotonin Vitamin difference between ovo Inctovegetarian and 2 pure vegan: pure vegan lacks Bra, while ovolacto does not Farmer rubs lesion off from the back of his neck but it grew back: actinic (solar) keratosis Taborn error of metabolism associated with pellagra: Harmup's discasc~ « loss of neu amino” acids in the Gl and GU tact, * loss of ryptophan leads to pellagra due to decrease in synthesis of niacin MCC of increased plasma homocysteine in United States: folate deficiency Patient taking dapsone develops cyanosis resistant toy therapy: patient has methcmoglobinemia due to oxidation of iron to ferric state Difference of B° and B° thalassemia: f° synthesizes some B chains, while ° does not synthesize any B chains due to a stop codon Sequence in hypoxic cell injury: + hypoxia+ 4 oxidative phosphorylation in mitochondria leading toa 4 ATP—+ J ATP leads to: « T anaerobic glycolysis (1 intracellular pH from lactic acid, ¥ etycogen), * dysfunction of Na‘/K” ATPase pump (reversible cellular swelling), « ribosomes seach fiom RER (+ protein synthesis, fatty change) ~» 1, jmeversible cell membrane injury: « lnuacellular release of lysosomal enzymes damages membrane, « endogenous activation of Phospholipases (7 influx of Ca” into cytosol) with release of toxic lipid products, « cytoskeletal alterations (activation of proteases by Ca”), 2. imeversible nuclear changes: « activation of nuclear enzymes by Ca", = nuclear pyknosis and lysis) —> 3, reversible mitochondrial dysfunction: « entry of Ca™ into mitochondria with activation of phospholipases causing destruction of inner and ‘outer membrane, + Ca" produces large densities SLE glomerulonephritis: antibody excess immune deposits combined with IgG a Note: This material is copyrighted. All rights reserved. Edward Goljan, M.D, 2002 a4 a4 99 F949 G4 GE a qa aq a4 Patient with SLE with Raynaud's phenomenon: more common with PSS and CREST African American (ook primaquine and then developed an ae Gna x ped an anemia with bite cells: G6PD Patient with intense occipital headache and blood in CS) patient with a ruptured congenital berry aneurysm Septic arthritis in young adult: Neisseria gonorrhoeae Chitd with disease characterized by neutrophils that cannot ki ia: sganulomatous disease of childhood or myeloperoxidase defcieney nari Cou be chronic Thrombotic thrombocytic purpura: * platelet thrombi (not DIC), * CNS hemorhage, « hemolytic anemia with schistocytes, «renal failure, «thrombocytopenia (consumed in roving CSF analysis in AIDS patient with meningitis showed an encapsulated organism: crypioceccus Calculate anion gap: + AG ~ scrum Na* - (serum CI’ + serum HCO,), « increased AG (add atid) duc to lactate, salicylate, B-OHB, AcAc, oxalate (ethylene glycol), formate (methyl alcohol) Phosphate/sulfate (renal failure), ¢ normal AG (lose bicarbonate) due to~ diarrhea, renal tubula: acidosis (proximal and distal) Organism causing pyelonephritis: + E. coli, « ascending infection due to vesicoureteral reflux Febrile woman with non radiating flank paiu on right side. What would be present in her urine?: WBC casts (pyelonephritis) Woman with painless nodule lateral to cricoid cartilage: probable papillary cancer of thyroid MEN I: « pinuitary tumor, » parathyroid adenoma, + pancreatic tumor (usually ZE), + peptic ulcer (due to ZE and hypercalcemia from hyperparathyroidism) MEN Dla: « medullary carcinoma thyroid, * hyperparathyroidism, « pheochromocytoma MEN Ib: medullary carcinoma, « pheochromocytoma, * mucosal neuromas Calcitonin is marker for medullary carcinoma of thyroid: calcitonin lowers calcium by inhibiting osteoclasts ‘Bypercalcemia in a patient with MEN syndrome: hyperparathyroidism with increased PTH Conn’s (primary aldosteronism) syndrome: = key findings~ « hypertension, « no pitting edema, ‘* hypernatremia (mild or upper limit normal), * hypokalemia, + metabolic alkalosis Hypoaldosteronism effects (spironolactone, Addison's, destruction of JG apparatus): + hypovolemia from salt loss, + hyperkalemia, + normal AG metabolic acidosis Mutation producing neurofibromatosis: inactivation of NF suppressor gene Hepatic encephalopathy with flapping tremors: relates to false neurotransmitters and increase in ammonia from bacterial degradation of urea in the intestine Sexually active man with sterile pyoria and no organisms seen on gram stain: probably Chlamydia trachomatis non-specific urethritis Man with pain in posterior aspect of left testicle: « epididymitis, + depending on age of the patient organism could be Neisseria/ Chlamydia if < 35 or E. coli/Pseudomonas aeruginosa if > 35 Young woman with 2 stroke and irregular left ventricle filling: embolization from atrial myxoma Lady that just returned from Asia has severe dyspnea pulmonary embolus from sitting too long Cystic hygroma in the neck area in a fetus: + Tumers syndrome, « lymphatic defects are conimon— cystic hygroma becomes webbed neck, « lymphedema of dorsum of hands and feet Tumor of 4* ventricte: * ependymoma- neoplastic ependymal cells, « usually in children Ascites in a patient with 2 normal liver biopsy: portal vein thrombosis leading to portal hypertension Elderly woman with 2 days of eye pain and dilated vessels on white of the eye that don’t blanch with vasoconstricting drugs: probable glaucoma Atrophy of tail of caudate nucleus: Huntington's disease Vitamin D excess: « hypercalcemia, « urinary stones * probable subarachnoid bleed in a in a day of arrival: probable 28 ae qq 9494 9999gad 4 ‘Vitamin C excess: calcium oxalate stones Paget's disease of bone in an elderly woman: « early phasc of lysis (resembles multiple myel except it does not have clear margins) and then bone deposit seo: fractures), mereased seram alkaline phosphatase Gepesition with soft mosaic bone (pathologic Man with severe hyponatremia and high urine osmolality: » i small cell cancer in the lung, * high urine osmolality is ian eed patie Pe Prrarareand Joes of sodium in the urine as well as constant concentration of the urine from excess Factitious insulin injection: « hypoglycemia, « high serum insulin, ¢ low C-peptide by hypoglycemia), « if C-peptide is increased, patient has ‘nsanoma’ (orppressea Type 1 vs type II: « type J has insulitis due to autoimmune destruction, type I has amyloid in islets, « type Thas HLA relationship, type I has family history, « type I has antibodies against islets and insulin, type 0 has no antibodies, * type I is complete insulin deficiency, type HI is relative baal deficiency with decreased insulin receptors (increased plasma insulin) and postreceptor Post-transfusion hepatitis: HCV Pulmonary fibrosis with pulmonary artery hypertension: » could be duc to drugs (amiodarone, bleomycin, busulfan, methotrexate), « Hamman-Rich hung from alveolitis syndromes AIDS patient with purple lesions on skin: Kaposi's sarcoma due to Herpesvirus 8 Case study of child with medulloblastoma: malignant cerebellar tumor Alzheimer's disease: © remember Down syndrome relationship and chromosome 21: 40 yr old with AD is an adult with Down's syndrome, « senile plaques, * loss of higher intellectual function Man with extrapyramidal signs: Parkinson's disease due to loss of dopamine Association of polyarteritis nodosa with HBsAg. Case study of 2 woman with gonorrhea: probable PID shortly after menses Anorexic patient has more risk for osteoporosis due ta ioss of estrogen Left sided heart murmurs and abnormal heart sounds increase in expiration Right sided heart murmurs and abnormal heart sounds increase in ixspiration ‘Aortic stenosis: « systolic ejection murmur with radiation into carotids (S, and increases on expiration), + diminished pulse, + MCC is congenital bicuspid aortic valve, © MCC of microangiopathic hemolytic anemia with schistocytes, « MC valvular lesion associated with syncope and angina Description of an x-ray of emphysema: vertically oriented heart Tension pneumothorax, spontaneous pneumothorax, and atelectasis: + spontaneous~ subpleural blebs in tall, slender males is a common cause of spontaneous pneumothorax, pleural pressure same as atmospheric pressure, lung collapses, diaphragm rises, trachea shifts to side of collapse, decreased breath sounds, tympany to percussion, » tension pneumothorax tear in pleura acts like check valve, pleural pressure higher than atmospheric, lung compressed and not collapsed, mediastinal structures shift to opposite side, diaphragms low, decreased breath sounds, tympany to percussion, « atelectasis (collapse of alveoli)- decreased percussion, high diaphragm, increased tactile fremitus, decreased breath sounds, inspiratory lag, trachea shift to side of atelectasis Chronic alcohol abuse leads to congestive cardiomyopathy and cardiac failure: could be duc to thiamine deficiency or direct toxic effect on the heart ‘Type Hl pneumocytes are the repair cells of the lung in pneumonia and ARDS: lamellar bodies (surfactant) are cytoplasmic markers of the cell Drug addict with fever and pansystolic murmur increasing on inspiration: wicuspid regurgitation due to S. aureus infective endocarditis ‘Type 2 diabetic with no diet control: Heb Alc is high increased A-P diameter, » depressed diaphragms, * 2 Note: This material is copyrighted. All rights reserved. Edward Goljan, M.D, 2002 4 § PHHTE q q e444 8a qaqa ae”d q 4 gad Polycystic ovarian syndrome: + |7-ketostervids ‘ cevere are elevated, © LH.> FSH (DHEA and androstenedione), testosterone, Benzene: causes aplastic anemia and acure leukemia ‘Testicular torsion: testis raised on affected side and loss of cremasteric reflex ‘cal case of ulcerative colitis in young man; bloody diarrhea ical case of an elderly man with prostate cancer: © remember osteoblastic metas tower lumbar vertebra, « if the history is only one. : in at night, itis prostate hyperplasia tory is only one of winery retention and problems with geting up 21-Hydroxylase deficiency: * salt loser, » increased 17-KS and decreased 17-1 increased ACTH, « female pseudohermaphroditism a Ey Aenea LI- Hydroxytase deficiency: + hypertension— increased deoxycorticosterone, + increased 17-KS increased 17-hydrexyeorticaids (1 -deoxycortisol), « increased ACTH, » deoreased serum cortical, * female pseudohermaphroditism — 17- Hydroxylase deficiency: * hypertension increased mineralocorticoids, « decreased 17-KS, « decreased 17-hydroxycorticoids, * increased ACTH, © decreased serum cortisol, + Teale hypogonadism, « male pseudohermaphrodite 3 Cardiac tamponade occurs 3-7 days after an AMI Pregnant patient accidentally swallowed I will probably devel Duchenne's muscular dystrophy: SXR diciarwii sone at erect pestgrian PGE, synthesized in the placenta keeps the ductus arteriosus open during pregnancy In 2 patient with acute nephritis, what lab test should be ordered: ASO titers to R/O post- streptococcal glomerulonephrits Renal casts and their disorders: « RBC casts~ nephritic syndrome, « WBC casts cute pyelonephritis and drug induced tubulointerstitia! nephritis, renal tubular casts— acute tubular necrosis, # fatty casts~ nephrotic syndrome, * waxy casts— chronic renal failure Teratomas: « germ cell origin, « located in midline testes/ovaries, anterior mediastinum, pineal Glandular metaplasia of distal esopbagus: Barrett's esophagus in GERD Kwashiorkor: « has decreased oncoric pressure due to decreased protein intake but adequate number of calorics, + defects in cellular immunity, « anemia, « ascites, « fatty liver from decreased apoproteins Marasmus: « total calorie deprivation, + broomstick extremities . Mycoplasma pneumoniae is the MC organism for atypical pneumonia in adults ‘Neutrophils phagocytose monosodium urate crystals in gout Pancreas biopsy in cystic fibrosis patient: atrophy is the growth alteration Epidural hematoma: ruptured middle meningeal artery Rapidly progressive dementia with neurofibrillary tangles: Creutzfeldt Jacob Defective synthesis of type I collagen in Eblers-Danlos syndrome ‘Auer rods: « only seen in acute myelogenous leukemia (includes progranulocytic), + not seen in any chronic leukemia (includes CML) or monocytic leakemie Case of extravascular hemolytic anemia. Hyperseusitivity is equivalent to: EHA is a type Tl reaction, 50 look for another type Il reaction-¢.g., Goodpasture's, Graves, myasthenia Primary hyperparathyroidism electrolytes: « hypercalcemia, « hypophosphatemia, « normal AG metabolic acidosis Case of hemolytic uremic syndrome: + schistocytes, « D-dimers (fibrin strands with inks benveea them), « thrombocytopenia platelets consumed, « increased bleeding time, * normal PT and PTT not DIC Common complication of severe hemophilia A: hemarthroses Kiinefelter syndrome: « 47 XXY, + low testosterone, « high FSH and LH, « high estrogen Macrocytic anemia with neurologic symptoms: cobalamin deficiency (Bi) and 30 Note: This material is copyrighted, All rights reserved. Edward Goljan, M.D. 2002 4H 449 F499 Gag 99°99 9944 | q "4 Miroteloms: «assis expomme(hip-yd, roofer >20 yr) «nosing eaonhip Temporal arteritis: + granulomatous giant ce{l oun oveeent tema = Ssiris, * mereased sed rate, « jaw pain when Proximal dissecting aortic aneurysm: « aortic regurgitation murmur, « wi ineeaguha, openaetst Ineiengee ete ation murmur, « widening of sori knob, « Hypophysectomy: zona fasciculata undergoes atophy due to loss of ACTH Case of emphysema and pathology: « destruction of elastic pat < cenuilobular involves respiratory bronchioles, « panscinar involves we imine Meee es duct, and lveoti, + paraseptal involves the alveolar duct and tiveoli~ commen ca of preumothorax Down syndrome with vomiting and maternal polyhydramnios: duodenal atresia Down syndrome with constipation at birth: Hirschaprung’s denace ‘High risk of colon cancer: multiple polyps Elevated alpha fetoprotein in pregnant woman: do an amniocentesis to rule ‘out open neural tube fect Decreased alpha fetoprotein in preguant woman: Down syndrome Complications of prostate hyperplasia: « infection, » urinary retention, « it does not piers. cancer High serum TSH: primary (not secondary) hypothyroidism Thyromegaly, normal TSH and T,, increased antimicrosomal and thyroglobulin antibodies: early Hashimoto's thyroiditis Osteogenesis imperfecta: « deficiency of type I collagen, « blue sclera~ due to visualization of choroidal veins MCC of a fatty liver and cirrhosis: alcchol abuse ‘Case of mass in fallopian tube ina Patient with an increase in B-RCG: ectopic Bacterial meningitis: « increased opening pressure, + high protein, * low glucose, « increased neutrophils Brain tamor in the third ventricle: could be colloid cyst or choroid ptoxus papilloma Patient deterioration after CNS trauma; hemiation Kartagener's syndrome: absent dynein arm in cilia Rinne and Weber test are like if you have cerumen in your ear: « Rinne lateralizes to affected car and Webers has bone conduction > air conduction, « signs of conductive hearing loss Injection of I liter of saline, start seeing diuresis: due to an increase in auial natriuretic factor After head trauma, the arine esmolality is 700 mOsmukg: « inappropriate ADH syndrome, «not diabetes insipidus (low urine Osm) Primary hypoparathyroidism: primary due to previous surgery or autoimnume destruction 1,25 dihydroxycholecalciferol: « active form of vitamin D, « requires PTH for its synthesis 1 a- hydroxylase Graves disease: + type II hypersensitivity: IgG antibody against TSH receptor, « unique to Graves: exophthalmos, pretibial myxedema, * TSH low and T, high, « Rx with B-blockers to inhibit adrenergic effects and propylthiouraci to inhibit enzyme synthesis Differences between primary/secondary hypothyroidism in relation to T, and TSH: » both have low T, © primary has high TSH, « secondary has low TSH Woman who was vomiting: « hypokalemia, metabotic allalosis normal AG metabolic acidosis, « normal serum sodium (isotonic loss), « hypokalemia A patient in the hospital accidentally gets infused with saline contaminated with £. coli and develops shock, 2 days later there is bleeding from all puncture sites, what will be the findings: « DIC with decreased fibrinogen and platelets, « increased D-dimers 31 Note: This material is copyrighted. All rights reserved. Edward Goljan, M.D. 2002 @ a9 #€4 sang a4 a4 A.16 year old football player suddenly dies during a Ae ee pearl Ties ienmateaar apeaaeper gee eee ‘A person develops congestive cardiac failure and decreased GFR. The renal cobale cals hydrople change. What is the cause of the bydropic change?: damage to the Na-K om Graph showing collagen formation after an AMI: graph that stars and increases afer 19-14 da An AlDs patieat develops diarrhea with an acid-fast organism. What is the most likely ‘cause?: Cryptosporidium © A patient with liver cirrhosis and ascites develops 5 scent 5 thie saopt Ukaly orjaaisn?: Bvcoll Sapiens Peritenitis, vehot is A patient is being treated for leukemia with anti-cancer drugs. hiring hoes RSE id of Hiney: sine A person has a crescende-decrescendo murmur heard that radiates to the carotids, what is the defect”: aortic Sg ene econ tae Which type of hepatitis is not affected by alcohol?: HAV 46 XX female is born with male genitalia and a vagina that ends in a blind pouch, bas hypertension and hypernatremia?: « adrenal hyperplasia, « 11 hydroxylase deficiency Baby is bora with paffy face, sluggish and diminished deep tendon reflexes?: cretin Ghypothyroidism) : A systolic murmur is heard io the mitral area. It increases in intensity by 1 grade after a pause. What murniur is it?: mitral regurgitation Girl comes in with corneal rings, hepatic and brain changes. What mineral defect?: copper in Wilson's disease Patient comes in with leukemia. Over the past year she has developed a decreased hematocrit with microcytic bypachromle anemia: anemia of chronic disease Pregnant woman has polyhydramnios. What does the fetus have?: anencephaly ‘Tumor with loss of pS3 suppressor gene. Loss of regulation at what point?: Gl to S phase Boy with pica for paint lias anemia with basophilic stippling. What will be the finding in this patient?: increased fice erythrocyte protoporphyrin in Pb poisoning 90 yr old lady suddenly dies. On antopsy the cerebral veatricles are enlarged. What is the most likely cause?: cerebral atrophy : Previously healthy 12 yr old girl develops aplastic anemia. Canse?: parvovirus Different diagrams of wound healing and must identify which one is correct. Pick the one with the basal layer of the epidersois joining with the one on the other side. Patient with small-medium vessel disease is HBsAg positive. Diagnosis?: polyarteritis nodosa Graph with increased basophils, neutrophils and monocytes. What is the cause?: GM-CSF Patient bas a stab wound to chest and there is lung collapse. What is the pressure inside the plearal cavity?: same as atmospheric pressure Scenario of 2 man who bas had a stroke and died. Autopsy shows brain with hemorrhage. What is the eause?: stroke involving lenticulostriate branches of the MCA due to hypertension Inheritance patterns for genetic disease: see High Yield pathology Man working with rocks develops pulmonary symptoms: silicosis Know Rh and ABO hemolytic disease of newborn: sce blood bank notes Budd-Chiari syndrome (hepatic vein thrombosis): see hepatobiliary notes Physical diaguosis of lang conditions: + tactile fremitus- increased in consolidations and decreased in pneumothorax, + egophony- ¢ sounds like a, sign of consolidation, + percussion decreased in consolidation, pleural effusion, tympanitic in pneumothorax ‘Scar tissue in brain; called gliosis and due to astrocyte proliferation ; rae most affected by decreased oxygen: renal medulla (particularly staight portion of proximal tubule) followed by neurons in the brain Hypertensive brain injury: intracerebral hemorthage Patient with acute pyelonephritis: WBC casts 32 Note: a4 a4 4 a4 VETTVadedg a4 a “Tuls material is copyrighted. All rights reserved. Edwara Goljen, M.D, 2002 Complications of anorexia nervosa (marathon runner): « decreased GnRH, * osteoporosis 3}: * secondary amenorthea due to Patient with bilateral hydronephrosis: urethral obstruction murmem Increnied 26 ether; Alfosterone due wy deceit caidian uput and = decreased metabolism by liver Organ secondarily affected in cirrhosis of liver: sp ; Pancreatitis: pain referred tothe back, since itis reuoperaregt ou © POTal hypertension Amniotic fluid embolism: causes DIC Serum haptoglobin: decreased in intravascular hemolytic anemia Robe ef anemia in chronic lymphocytic lenkema:aoimmune hemolytic anemia Neonatal respiratory distress syndrome: no surfactan, Findings 30 min after acute myocardial Infarction: no changes Phosphorus increased in chronic renal failure MC TS site én reactivation: + upper lobe of lung, «high O, tension Pancoast tumor causes Horner syndrome: destucton of superior cervical ganglion in posterior Gp 120 attachment of HIV virus to CD, molecule Pedigrees: all types + mitochondrial DNA Causes of children getting a disease when parents don't have any evidence of it: incomplete penetrance im an autosomal dominant disease Patent mith a history of & bone marrow transplant and who is om Cyclosporin, has neutropenia and devclaps esophagitis: « picure of inwanucleasincusion= ifa single meleror Howl ov, pick CMY, ifmulninucleated cell with inranuclear inclusions pick Hones Enzymes decreased in lead poisoning: ferrochelatase and ALA dehyarece pauent witha bistory of bypertension develops an intracranial hemorrhage. What part ofthe Drain is affected?: basal ganglia (putamen isthe most coramon area) Huntington's disease: « caudate atephy, « dementia, « odd movements, © autosomal dominane, « trinucleotide repeat disorder Firefighter with a history of dyspnea 6 hours following a fire. Eistologic section of alveoli with hyaline membranes: ARDS Patient with septic arthritis. What is most likely organism”: Neisseria Alcoholic with a histary of depression and hypoglycemic episodes. Low blood glucose and no: ‘C-peptide: patient has been taking insulin Cause of acute renal failure ia 2 patient in a nursing home who hasn't eaten or drank anything for a while. Patient is dehydrated, has a history of a stroke 6 months ago, and has increased BUN and creatinine from previous values. He has pyuria but no hematuria, He has not urinated in 12 hours. After catheterization, 100m. of very dark yellow urine is removed, ‘What is it?: probably prostatic hyperplasia causing urinary retention and infection Cause of pneumonia in cystic fibrosis: Pseudomonas aeruginosa An alcoholic has 2 Klebsiella infection involving the upper lobe of the right lung, what kind of necrosis is it: liquefactive necrosis Granular IgG and C; deposits are found in the glomerular basement membrane: type memibranoprolifcrative glomerulonephvitis (dense deposit GN) What part of the kidney is most involved in diabetes: glomerulus 7 Tall, skinny 17 year old basketball player with long arms and legs and hyperflexible joints, what is the boy likely to develop: Marfan's with danger of dissecting aortic aneurysm A 6 year old boy ingests rat poisoning, along with supportive therapy, what should be given: IM vitamin K injection ; Prenatal vitamins: «iron: women only have 400 mg of iron stores and lose 500 mg in pregnancy, * folate: only 3—4 mth supply in liver, « By, is not necessary unless a woman is a pure vegan 33 ee ee | 4 V994ad 4q 994 a4 Most cost effective way of preventing hepatic coma in cirrhotics: « decreases ammonia load to liver where urea cycle is dysfunctional Vitamin deficiency associated with depression; «nisin deficiency due to typtcph : fophan synthesizes serotonin, which is decreased in depression deficiency, Vitamin deficiency associated with a tea and toast diet in elde Vie rly patieat with bleeding gums: Patient being treated for TB develops » microcytic anemia: sideroblastic deficiency related to INH Rx — blastic anemia from B6 Vitamin D deficiency in 2 patient on phenytoin: increased metabolism by the cytochrome sysiem Pare vegan is breast feeding baby. Baby develops anemia: By, dsfiiency Chile with milk allergies develops macrocyic anemia: faking gous mule which is deficient in ic acid After am earthquake in southern California many develop pulmonary symptoms most likely casual organism: Coccidioides oe eas reduce protein intake, « ‘Spherule with endospores! Cause of anemia in a patient with nodular sclerosing type of lymphoma?: anemia of chronic disease Folic acid must be taken before conception to prevent anencephaly in subsequent pregnancy ‘Woman with incomplete mole. Genotype?: tiploid 69 OY eee Patient insensitive to burning of hands. Upper extremity muscle weakness: syringomyelia Fuzzy balls in apical lung cavities in a patient with massive hemoptysis: Aspergillus Solitary coin lesion in the lungs of a patient from Mississippi: histoplasmosis Alveolar macrophage with yeasts Patient with prosthetic heart valve has schistocytes im peripheral blood: prosthetic valve is malfunctioning causing hemolytic anemia MCC of bladder diverticula: prostatic hyperplasia Prevent keloid formation: intralesional injection of corticosteroids : ‘Mother and father blind but no family history of blindness, one child already blind, chance other child will be blind: » patients probably have homozygous AR disease and parents were asymptomatic carriers, « all children will have the disease (100%) ‘Vitamin deficiency in pheochromocytoma: vitamin C- catalyzes dopamine to norepinephrine Phases of acute inflammation: begins with transient vasoconstriction before vasodilatation from histamine release Yellow mass extending up inferior vena cava: renal adenocarcinoma invading renal vein with extension up IVC Granaloma in AIDS patient: would have macrophages but they would not be activated to lal phagocytosed organisms since CD, T helper cells are gone —>—~Canse of increased serum cortisol and thyroxine in pregnancy: + increase in synthesis of their respective binding proteins by estrogen stimulation of liver, « TSH and ACTH are both normal, « free hormone levels arc normal. Mother. with polyhydramnios and fetus with anal atresia: VATER syndrome~ vertebral defects, anal atresia, tracheoesophageal fistula, radial (absent) and renal abnormalities Patient with deep venous thrombosis only given warfarin (not ciara) cpa os cine opagation of venous clot: heterozygote protein C deficiency, © short life of protein aes ibe patient hypercoagulabie in 6-8 hrs, since factors V and VIII are increased due to 4 ‘Note: This material is copyrighted. All rights reserved. Edward Gotjan, M.D. 2002 complete absence of protein C in the blood, «patient could also have developed hemorrhagic skin necrosis, 7 Patieat develops oliguria after an intravenous pyelogram: » developed — tubular necrosis, « dangerous to order an IVP in multiple photo Rheumatoid factor: igM antibody against gG h* ¥SlOmm and diabetes melas Child whose father has a history of osteogenic sarcoma dev. . ph sn ee Sen ot ae osteogenic sarcoma and breast cancer are other cancers associated with this suppressor gene, » likely that this was an autosomal dominant inheritance partcrn, so one of the Rb suppressor pees so Teaenaaasiiien birth, * ee a mother's side are all normal, thea the child's eye lem wo. sporadic jon (rrutation uppressor chromosomes have to be inactivated. pHa eee poneeP ae w Weight lifter develops numbness and tingling in the arm while sleeping: thoracic outlet syndrome with compression of subclavian artery and brachial plexus by hypertrophied anterioy scalenus muscles = Male child with chronic infections with a granulomatons reaction: » chronic grenulomatous disease of childhood, « XR inheritance, * absent NADH ‘oxidase in neutrophils and monocytes therefore cannot convert molecular oxygen into superoxide free radicals (absent respiratory burst) and cannot Kill bacteria, especially Staphylococcus aureus, © without superoxide, peroxide cannot be produced by superoxide dismutase, * without peroxidase, myeloperoxidase cammot combine peroxide with chloride to produce bleach. ‘+ Chromosome 21 codes for an Alzheimer precursor protein (APP), part of which is amylokd-B (A-B) protein: + toxic to neurons, association with Down syndrome (3 fimctional chromosome 21s) MCC of Alzheimer's in patient's under 40. 7 — Apolipoprotein gene E, allele e4 located on chromosome 19: « produces a product that increases the neurotoxicity of the Af protein in Alzheimer's disease, + cause of familial late-onset type Abnormality on chromosome 14 in Alzheimer's disease: synthesizes a Tau microubule- associated protein located in neurofibrillary tangles * Hamartoma: + overgrowth of ssue normally present in the organ, * bronchial hamartoma: increase in cartilage, * Peutz Jeghers polyp, * hyperplastic polyp in the sigmoid colon, + angiomyolipoma of kidney: seen in tuberous sclerosis : = Heterotopic rest (choristoma): * normal tissue in a place it is not normally present, « pancreatic ‘tissue in wall of stomach, « gastric mucosa in a Meckel's diverticulum: 2 Patient with restrictive lung disease, Raynaud's phenomenon, renal disease: © progressive systemic sclerosis (scleroderma), + esophageal dysmotility is another major problem leading to replacement of smooth muscle by collagen, « dysphagia for solids and liquids * AAT deficiency: * AR disease, « AAT is an anti-clastase that destroys clastase (protease) released by neutrophils, © children with certain phenotypes develop cirthosis: PAS + AAT in the hepatocytes (defect in secretion of AAT out of hepatocytes), » young adults with certain phenotypes have complete deficiency of AAT and develop panacinar emphysema (entire respiratory unit destroyed) in lower lobes (absent cil globin peak on a serum protein electrophoresis) + First step im management of tension pneumothorax: « positive pressures in pleural cavity push the lungs and mediastinum in opposite direction, hence compromising respiration and venous rerum to the heart, « insert needle into second intercostal space (anterior chest) to release positive pressure buildup in the pleural cavity _ Graves disease: IgG antibody directed against the TSH receptor: type Il hypersensitivity reaction Newborn with tetany and small ears: ¢ DiGeorge syndrome, » 3rd and 4th pharyngeal pouches do not develop (absent thymus and thymic shadow and absent parathyroids), * expect hypocalcemia a4 94 35 Note: This material is copyrighted. All rights reserved. Edward Goljan, M.D, 2002 a4 a4 NSAIDs: « prolongs bleeding time: inhibiting cyclooxygenase : platelets cannot aggregate, * normal platelet count and Tema ca pt Det Nine month old black child has dactylitis: « sickle cell crisis, « sickle cell di a eit hoe mutation) where valine replaces glutamic acid im the 6th position of the B Pernicious anemia: » autoimmune destruction of parietal cells, » ‘ deficiency) and acid production, eachlochydria eases epee ee Sexe Bia femophilia A: « SXR, edeficient VII: coagulant Sian to activate factor X ongulant, VITK complexes with platelet factor 3, DXa, and NSAIDs (e.g., indomethacin): « decrease PGE;, which normally maintains the Tae Sechelt ita dooms prcoonsea igen cones ol pepeicuioos, epee ‘Women with excessive weight loss (e.g. marathon runner, aerobics instructor, anorexia nervosa): decrease secretion of GnRH from hypothalamus leading to decrease in gonadotropins—> secondary amenorrhea» osteoporosis (lack of estrogen) Woman in 40's with malignant ascites and large ovarian mass: probably a mucinous cystadenocarcinoma: very large ovarian tumor, often associated with pscudomyxoma peritonei, if it was bilateral, then a serous cystadcnocarcinoma would be most likely ° Posterior acute myocardial infarction, right heart enlarges in 1 week, pulmonary capillary wedge pressure is decreased: « probable right ventricular infarction with pure right heart failure, « potentially could be a pulmonary embolus producing right heart strain Gross (G) and Microscopic (M) changes in an acute myocardial infarction (AMI): © 0-4hrs— no G/M change «4-12 hrs~e no G change, « M shows coagulation necrosis after 6 hrs . 12-24 brs— « G shows early pallor, » M shows more advanced coagulation necrosis © 1-3 d-« G — definite pallor, » M — neutrophilic infiltrate coming in from the perimeter of the infarct + advanced coagulation necrosis . 347 d- «© period ‘of maximal softness, « time for ruptures, ¢ macrophages move in from the perimeter to remove dead tissue © 7-10 d-« Gis the same as 3-7 d, + M shows collagen deposition yromegaly, normal thyroxine and TSB, presence of antimicrosomal and antithyroglobalin antibodies: early stage of Hashimoto's thyroiditis Patient develops dementia after instrumental involvement in the brain: CJ disease due to rions Baby with syndactyly: defect in apoptosis, which normally would have caused regression of thase tissues between the fingers and/or toes ; Bipolar patient develops polyuria: probably on lithium which produces nephrogenic diabetes insipidus Hematopoiesis in last month of fetus: occurs in bone marrow ; Baby chokes with every feeding and develops pneumonia: + TE fistula, blind proximal aehagus, = distal esophagus arises fom wachea: stomach distended, ¢ mother with polyhydramnios . Diethylstilbestrol: interferes with development of Mullerian structures | ; Patient with a central line develops hemiparesis: + central line is inseried either into the subelavian vein or the internal jugular vein, « clot develops in the central line ‘and embolizes to right syjum and enters systemic circulation through an atrial septal defect - Hyponatremia: could be due to an A. excess of free water Caaporopriate ADH on reabsorption of proportionately more water than salt (edema states like right ae fe or C. loss of proportionately more salt than water a diuretics, 21-hydroxy| 36 tea Note: This material is copyrighted. All rights reserved. Edward Goljan, M.D. 2002 deficiency, Addison's dseate with lots of mineralocorcoids, REA by rescricting water and salt iatake and using diuretics, Re Ge cere % Sex differentiation: « Y chromosome determines genetic sex: deaseeyre nal saline difereatanon of germinal sue into ovaries: wolfian duct suenice dere ean Of chromosome causes germinal tise to differentiate into teste: mailleran hibreny neers synthesized» apoptosis of millerian tissue in the male fetus, « fetal tatoree ge epididymis, seminal vesicles, vas deferens, « fetal il stcrone (DHT) develop prove (underscores why DHT is responsible for prostate hyperplasa/cancerandot estos ers male genitalia: fusion of the labia —+ scrotum, extension of clitons -» penis, © Sorcohecen converts testosterone into DHT ee True hermaphroditism: + patient has both male/female gonads: majority are 45 XX enone © ovary and testis, or votes, « * Psendohermaphrodite: + patient whose phenotype (appearance) is not in agreement wi genotype (tue gonadal sex), * male pseudohermaphrodite: genotypic male (XY a, Phenotype appears female (female genitalia), eg, testicular feminization, * femile Pseusiohermaphrodite: genotypic female (XX with ovaries), phenotypically resembles a male. eg. virilization in adrenogenital syndrome ” ‘7 — S-a-reductase deficiency: + male pseudohermaphrodite; not present in females, » absence of DHT effect on male fetus: testes located in inguinal canals, absence of prostate gland: no DHT effect, absence of ali millerian structures: no tubes/uterusicervis/uppet one-third of vagina since millerian inhibitory factor is present, extemal genitalia female due to absence of DHT effect blind vaginal Pouch, vagina partly milllerian in origin and partly of urogenital sinus origin, * presence of testosterone effect including: epididymis, seminal vesicles, vas deferens * Testicular feminization: + SXR disease: MCC of male pseudohermaphroditism, * deficieacy of androgen receptors: DHT and testosterone are present but cannot function without a receptor, « no milllerian structures; millerian inhibitory factor is present, « no male accessory structures: no testosterone effect, absent epididymis/seminal vesicles/vas deferens/prostate gland, * extemal genitalia remain female: no DHT effect, vagina ends as a blind pouch, « testicles located in inguinal canal or abdominal cavity; surgically removed owing to a risk for seminoma, « estrogen unopposed, since estrogen receptors are normal: secondary female characteristics are well developed, © patient 1s reared as a femaic, « laboratory findings: normal testosterone/DHT, increased LH: LH does not* respond to the negative feedback of testosterone * Smoker with flank pain and mass, hematuria: « renal adenocarcinoma, « derives from proximal renal tubular epithelium, « smoking MCC = Family tree with AD inheritance pattern showing disease occurring at am earlier age in affected members: « trinucleotide repeat disorder: disease gets worse in future generations due to addition of trinucleotides (c.g, CAG), © ¢.g., Huntington's disease an AD movement disorder associated with dementia would occur at an earlier age, + ¢.g., female carriers of fragile X (SXR disease) will develop symptoms of mental retardation (makes it look like an SXD disorder). 2 Stillborn of diabetic mother in poor glycemic control: likely see hyperplasia/hypertrophy of B- islet cells in the pancreas as a fetal response to maternal hyperglycemia Cirrhosis of liver: nodules are regenerative nodules composed of hepatocytes with no cord- sinusoid-cord pattem = Cirrhosis of liver: + antibody test that would be helpful in determining the cause would be ~ antibodies against HCV, antigen that would be useful is HBsAg Newborn has cataracts and vomiting: © probably an inborn error of metabolism (€.g, galactosemia or hereditary fructose intolerance), * order a urine for reducing substances: detects fructose and galactose water, Rx B by 37 Note: This material is copyrighted. All rights reserved. Edward Goljan, M.D. 2002 9 9 @ m TV infusion of thiamine (B1) in an alcoholic: thiamine is a cofactor dehyt a-ketogiutarate dehydrogenase, and a-ketoacid dehydrogenase, « To NADH, which, in tum, generated 6 ATPs * Teactions produce Electrolyte abuormalities in diabetic ketoacidosis: increased ani . increase in AcAc and B-OHB, hyponatremia: dilutional effect of water moving ea ee ee by osmosis, Ippertaenia ansecular kit of Kou of als moves ine te cel eee ae patient is deficient in K+ duc to osmotic diuresis, loss of excessive amount of Na” and Ke in ‘osmotic diuresis of excessive amount of Na" and K" ia ure: Child with petechia, thrombocytopenia, normal PT and PTT, no schistocytes: thrombocytopenic purpura, type Il hypersensitivity Se eee aaa Vegetarian with microcytic anemia: ¢ probable iren deficien i cy, plant irom is fe Sonvered into ferrous tobe reabsorbed, «very small rik for iron deficiency * Fame and must be /oman with facial rash, arthralgia, positive VDRLL with negative FTA-ABS: system erythematosus, note the biologic false positive syphilis serology due to anti-cardiolipin, saad Baisd re lesions i AIDS patent: « mos often duct Kapos’s scam (HSV), «cou also angiomatosis due to Bartonella henselae: silver stains identify organism, also cause cat scratch discase “demon — Animal with AB alleles crossed with animal with AC alleles. What i receive an AA organ? 75% peta Sepia cs aA Cc A AA AC BC Classic ABG patterns: PaCO2 HCO; G3-45 mm Bg) (22-28 mEq/L) A 69 Fs B. 26 u c. 68 34 D. 2 14 E. 48 38 F. 24 2 Acute uncompensated respiratory acidosis: barbiturate overdose, CNS trauma, ARDS, paralyzed diaphragm or other muscles of respiration Metabolic acidosis with partial compensation: any’ cause of increased AG metabolic acidosis (lactate, AcAc, B-OHB, formate [methyl alcohol poisoning], oxalate [ethylene glycol poisoning), renal failure, salicylate), any cause of normal AG metabolic acidosis (diarrhea, renal tubular acidosis) Chronic respiratory acidosis with partial compensation: chronic bronchitis, bronchiectasis, lungs in cystic Aibrosis Mixed primary respiratory alkalosis and primary metabolic acidosis: salicylate intoxication, endotoxic shock Metabolic alkalosis with partial compensation: vomiting, loop/thiazide diuretic, mineralocorticoid excess (primary aldosteronism) Respiratory alkalosis with partial compensation: pulmonary embolus, diseases, anxiety, restrictive tung 38 Note: This material is copyrighted. All rights reserved, Edward Goljan, M.D, 2002 7 Classic electrolyte profiles: Serum Na” Serum K* Seager Serum BCO; (138-147 mEq/L) (@.5-5.0 mEq/L) A. ns 3p) ORNS meV) 2-28 mEWl) B. 130 3.0 30 6 e 130 55 8 c D. 128 58 14 a E. 140 3.0 4 1 Fr 150 2.0 107 a Patient A has inappropriate ADH syndrome~ note the diluti : ta fas inaperop syn note the dilutional effect on all the analytes, especially a could has metabolic alkalosis due to vomiting or diuretics (thiazides or loops) atient C has increased anion gap (G2 mEq/L ie canaeatiiena gap ¢ ) metabolic acidosis. hyperkalemia is due to Patient D bas Addison's disease or a type [V RTA due to mine id deficien these patients have a normal AG metabolic acidosis and hyperialemiy sor note that Patient E has a normal anion gap metabolic acidosis due to diarrhea or type I or type I RTA~ note the hypokalemia : Patient F has mineraiocorticoid excess (eg. primary aldosteronism) = Asian man with nasopharyngeal mass: nasopharyngeal carcinoma, « associated with EBV 7 Myeloperoxidase deficiency: « acquired or genetic, « absent azurophilic granules in neutrophils in peripheral blood, © respiratory burst is normal— can generate superoxide FRs, « cannot kill bacteria~ * microbicidal defect, * no MPO to produce bleach * — Congenital adhesion motecule (B,- integrins) defect: * failure of the umbilical cord to separate in newboras, « no adhesion of neutrophils to the endothelial cells, « no inflammatory cells in the umbilical sturmp MCV Hypersegmented neutrophils ‘Autoantibodies: against IF and arietal cells. ‘Achlorhydria ‘Serum gastrin levels Risk of stomach adenocarcmoma Plasma homocysteine Urine methylmalonic acid Neurological disease ‘Schilling's test = Differential features of the microcytic anemias: bolded areas represent key differential points Fedeficiency ACD a,f-Thal minor — Sideroblastic (Pb poison) Mcv Low Low Low Low Serum iron Low Low Normal High TBC High Low Normal ow % Saturation Low Low Normal High Serum ferritin Low High Normal High RDW High Normal Normal Normal RBC count Low Low High Low 39 Note: This material is copyrighted. All rights reserved. Edward Gollan, M.D, 2002 Fedeficiency ACD o, B-Thal minor Sideroblastic (Pb pot RBCFEP High High Normal a (Pb poison) [ Hb electro, Normal Normal ce-thal: normal Nena es f }-thal: Marrow iron Absent High Rasy THe ArandE " Miscellaneous Ferritin best Hgb clectro. Comme basophil gold standard stippling. oe [ FEP = free erythrocyte protoporphyrin. RDW ~ red blood celll distribution width “ a * _ Summary chart with tab differential for polycythemia: [ Condition RBC mass | Plasma volame | S20, rytbropele Polycytheria rubra vera Tncreased | Increased Normal —| Boe ‘Appropriate polycythemia | Increased | Normal Decreased | Increased COPD. cyanotic CHD Ectopic polycythemia Tnereased | Normal Normal | Tnereased renal disease, HCC Relative polycythemia: Normal Decreased [Normal ‘Normal’ [volume depletion ‘CHD = congenital heart disease, HCC = hepatocellular carcinoma = Oxygen saturation (S20;) findings in congenital heart disease: ‘Normal | Patient | Patient | Patient | Patient | Patient S20, A B Cc D E Right arium 75 75 80 75. 75 zo | Right venmricle | 75 80 80 75 |i 80 Pulmonary art_|~75 80 80 80 75 80 (Pulmonary vein |_95 95 95 95 95 95 [Leftvenrricle {95 95 95 95 20, 20, Aorta 95, 95. 9s | 95 80 80 ‘SaO, = oxygen saturation A= VSD, B = ASD, C= PDA, D = Tetralogy, E = transposition > — Summary of pulmonary function tests in obstructive and restrictive lung disease: Parameter Obstructive Restrictive Total lung capacity(TLC) Increased Decreased Residual volume (RV) Increased Decreased Tidal volume (TV) Decreased Decreased Vital capacity (VC) Decreased Decreased FEV ine Decreased Decreased FVC Decreased Decreased FEV jue/FVC Decreased Normal to increased FEV ye: = forced expiratory volume 1 second, FVC = forced vital capacity = _ Comparison of ulcerative Colitis (UC) and Crohn’s disease (CD): Characteristic | Ulcerative Colitis Crohn's Disease Extent ‘Mucosal and submucosal ‘| Transmaral 4 Location Primarily targets the rectum. May | Terminal ileam involved in 80%.Terminal extend up into left colon in cont- ileum alone (30%), ileum/colon (50%), colon , inuous fashion (no skip lesions) or —_ alone (20%). Anal involvement 75%- ! involve entire colon. Does not involve | fissures, fisnulas, abscesses. Involves other | other areas of GI tract. areas of GI tract (mouth to anus) A are Characteristic Ulcerative Colitis Crohn’s Disease Gross features Inflammatory polyps— pseudopolyps representing areas of inflamed residual mucosa. Friable, red mucosa bleeds easily when touched, Ulceration’ hemorrhage. No skip areas. Fat creeps around the serosa, wall and narrow lumen. Skip os oe Gmportant feature). Strictures, fistulas (very i ic) in areas other than anus. Deep linear ulcers, cobblestone pattem, aphthoid ulcers (smail mucosal ulcers, ‘origin of linear ] ulcers, early sign). , Microscopic | Active disease: mucosal inflamma- | Very di: a 2 an | ' features tion with crypt abscesses (neutro- | granulomas (60%), transemurat | phils). Dyspiasia/cancer may be inflammation with subserosal lymphocytic + present. Chronic discase: neutrophils | infiltration, replaced by lymphocytes! plasma i cells. Dysplasia/ cancer may be | present. Complications | More common than CD: 1. toxic More common than UC: |. fistula ! megacolon— hypotonic/distended formation, 2. obstruction, 3. disease in other; bowel, 2. sclerosing pericholangitis, | areas of Gi tract, 4. calcium oxalate renal 3. HLA B27 + ankylosing spondylitis! | stones uveitis, 4. pyoderma gangrenosum, 5. adenocarcinoma-~10%, greatest risks are pancolitis, early onset of UC, and duration of disease >10 years Clinical Left sided abdominal cramping (not | Right lower quadrant colicky pain obstruction), diarthea with blood/ mucus, rectal bleeding, tenestaus (obstruction in area of terminal ileum) with diarrhea, Bleeding if colon involvement. i : i Anti-HBe-IgM_ Ant-HBs “| Interpretation negative | negative positive negative negative | serologic gap positive | positive positive negative negative | acute infection or chronic if >6 mths negative _| negative negative positive Positive | recovered from HBV | negative _| negative negative negative | _positive | immunized ! positive | negative negative negative negative _| earliest phase of acute HBV | = Summary table for polyuria: CDI = central diabetes insipidus, NDI = nephrogenic diabetes insipidus i POsm post H,O0 UOsmpostH,O UOsm post ADH (vaso deprivation deprivation —_pressin) injection NORMAL 292 750 760 CDL 319 110 550 NDI 312 98 120 PSYCHOGENIC 288 760 780 4l 7 _ Summary table of thyroid disorders: Disorder ‘Serum T, Grave's Increased Factitious thyrotoxicosis Thyroiditis (acute, subacute) Primary hypothyroidism Increasea Vi \ Gashimoto) al Secondary hypothyroidism [Decreased | Decreased | Decreased | Decreased TB _ Chypopituitarismy/ypothal) L ' Increased TBG Increased | Decreased omar (c.g. increased estrogen) Nomat [Normal [Neral Decreased TBG. Decreased Incre: i i cased Nom (increased androgens) Nomnal | Normal a ‘= Summary of laboratory studies in Cushing's Syndrome: - Laboratory Test Pituitary Cushing’ ing” i Seram Cortisol Treesed asad ne® Peon Cashing’ Urine for free cortisol Increased Increased ineressed Low dose dexamethasone Cortisol not Cortisol not Cortisol not suppressed. suppressed suppressed High dose dexamethasome Cortisol suppressed Cortisol not Cortisol not suppressed Suppressed Plasma ACTH. “Normal” to Decreased Markedly Increased Increased @ — Non-caseating granuloma in an African-American with hypertension: sarcoidosis = Soft, tender lump in the inguinal area of a child: cryptorchid testis = Kidney of hypertension: nephrosclerosis— due to hyaline arterioloscterosis Woman with a3 cm mass removed from breast. 1 year later, the lung is involved. Mechanism of spread?: hematogenous 2 Increased intraocniar pressure and swelling of retina: glaucoma Woman with left flank pain, microscopic hematuria. CT sbows enlarged bladder and kidney. Site of obstruction?: bladder-urethra junction * * — Crateriform lesion on inner side of eye. Site of origin?: basal cell carcinoma— derives from basal cell layer ® — Parotid mass with and without atypia: » with atypia is a mucoepidermoid carcinoma, « without atypia is pleomorphic adenoma = Patient with AIDS has a space occupying lesion in the brain: toxoplasmosis HIGH YIELD NOTES ANATOMY ® Arteriogram of upper arm with 2 history of an absent radial pulse: damage to brachial anery- extension of the axillary artery and divides into the radial and ulnar artery Bypoglossal nerve in brain stem: exits in medulla at preolivary sulcus Cerebellum: Purkinje (rabies inclusions), mossy fibers in lower cerebellum Spinal spinothalamic tract and medial lemniscus: both go to thalamus ‘Types of junctions in lateral region of cells: # zona occludens (tight junctions), * zona adherens, « desmosome, « gap junction (nexus, passage of materials, dye passes from one cell to the next) Leaky junctions: present in proximal tubules of the kidneys Actin: present in muscle in iris but are not responsible for motility in sperm a4 a9ee 4 qq 42 ‘Note: This material is copyrighted. All rights reserved. Edward Goljan, M.D. 2002 944 14 4aq 94 944 signs (loss of lateral corticospinat tract), » senteiibratory sensation from a sien lets ee He disciminaton/posiional rain stem for localization of CN VII: patient Seeesin, side and buceinator and facial muscles on ight se ows PAYS Of ofbiculasis oes on right Brain stem for localization of CN VIN: « cerebellopontine angle NE sory ‘neural hearing loss, » located at Brain stem for localization of CN VI: history of problem with lateral gaze- loss of function of Solitary nucleus: receives general visceral afferents afferents fom CN VIL, IX, and X and projects thes te th none no ea Siliiciaiasin Neuroanatomy of brain slices for locations of structures involved in different diseases; « caudate nucleus— absent in Huntington's chorea, « lenticular nucleus~ degenerated in patient with Wilson's disease and is associated, « substantia nigra— see depigmentation in Parkinson's disease Hemiballismus in patient with Wilson's disease: degeneration of the subthalamic neste. Tabes dorsalis: dorsal column syndrome loss of tactile discrimination, loss of joint and vibratory sensation, paresthesias and pain (dorsal root irritation), astercognosis, hypo/areflexia (dorsal root deafferentation), urinary incontinence/eonstipation/impotence (dorsal root deafferentation) Romberg’s sign (standing patient unsteady with eyes closed but okay when open) CT of abdomen: kidney is bchind the pancreas CT of thorax: identify structure at the level of the sternal angle Mascte that attaches at iliac erest: « gluteus medius, which abducts and rotates the thigh « distal attachment is greater trochanter of femur Visual field defect with eraniopharyngioma or acromegaly: bitemporal hemianopsia- impinges on optic chiasm Outpouching of floor of diencephalon: cerebrum ‘Neurotransmitter of raphe ancleus: serotonin Location in a cystourethrogram of the urogenital diaphragm: © muscles are deep transverse perineal and sphincter urethra muscle, « in male, it is located just distal to prostate, « in female, probable at beginning of urethra when it exits the bladder Spinal cord site for loss of two point discrimination in left leg: fasciculus gracilis in dorsal columns covers two point discrimination for the ipsilaterai leg (closest to midline), « fasciculus cuneatus (lateral to cuneatus) covers two point discrimination for the ipsilateral arm Spinal cord site for loss of pain sensation in left leg: lateral spinothalamic tract on right side of cord (remember the cross-fibers) Patient with dyscalculia, finger agnosia, left/right confusion: + lesion in visual association cortex (angular gyrus) in left parietal lobe, + called Gerstmann's syndrome Papilledema: swelling of the optic nerve~ indicates an increase in intracranial pressure Omphalocele: » midgut loop fails to return to abdominal cavity, © shiny sac visible at base of umbilicus X-ray of duplicated ureter: congenital anomaly Horseshoe kidney: lower poles fuse, « kidney trapped behind inferior mesenteric artery Nerve cut producing sensory loss on medial side of leg when taking saphenous vein for CABG procedure: saphenous nerve, which is a branch of femoral nerve Note: This material is copyrighted. All rights reserved. Edward Goljan, M.D. 2002 a4 V99NGdg ag 4 a4 a% 9% 4 a4 CT of abdomen: structure that splenic vein empties into: « empties into portal vein neck of pancreas), « PV is composed of splenic vein and superior Gosterior to aon aes cade by foliage splenic vein ‘Desenteric vein (largest vessel), « ‘Ulnar nerve functions: * medial epicondyle injuries, * normally adc ‘of paimar and dorsal interosseus muscles, « adducts thumb sacra pense: us a Gerad to hemiated disc compressing root of plasma cell: sceretes antibodies, prominent rough endop! ek mithearakeal appearing nuclear chromatin endoplasmic rectum, cecenric muleus Characteristics of postganglionic peripheral nervous system fibers: « . Duodenal ulcer perforation and bleed, petolocend oer ‘bers: shor,» near end organ pa of knee joint in a patient with a positive posterior draw sign: identify posterior cruciate ligament Bicormuate uterus: failure of fusion of paramesonephric duct CN V: goes through superior orbital fissure ‘Melatonin: produced in the pineal gland Abdominal incision to avoid interrupting blood supply: midline Cleft palate: maxillary prominence fails to fuse with medial nasal prominence ‘Hematopoiesis at 28th week: bone marrow main site Schwann cel/oligodendrocyte: » Schwann cell makes myelin for PNS oligodendrocyte for CNS, « ‘Schwann cells develop from neural crest cells Location of neurohypophysis in the pituitary: neurohypophysis is posterior lobe where ADH is stored ‘Loss of pain and temperature on side of face: CN V lesion Derivation of adrenal medulla: « neural crest origin (S100 antigen Positive), © neuroblasts develop into ganglia, * know the layers of the adrenal gland from outside in from the cortex to the medulla— glomerulosa (aldosterone), fasciculata (cortisol), reticularis (sex hormones), medulla (catecholamines) Tibial nerve fauction: + plantar flexion of toes, + injury causes~ + loss of plantar flexion, « foot Gorsiflexed and everted (calcaneovalgus-cavus), * sensory loss on sole of foot Composition of aortic valve (also pulmonic valve): « lined by endothelium and have abundant Abroclastic tissue plus a dense collagenous core, « avascular, * MV and TV have a loose connective ‘issue core composed of dermatan sulfate which is increased in mitral/ricuspid valve prolapse (myxomatous degeneration) Break humerus, wrist drop; radial nerve injury Post-radical mastectomy—winged scapula: indicates injury to the long thoracic nerve; paralysis of the serrams anterior muscle Patient with 2 nosebleed and rhinorrhea: fracture of cribriform plate in ethmoid sinus Medial lougitadinal fasciculus demyelination in maltiple sclerosis: bilaicral internuclear ophthalmoplegia Parathyroid derivation: third and fourth pharyngeal pouches Aorde arch derivatives: « first- part of maxillary artery,» second stapedial and byoid artery, « third— common carotid artery and proximal portion of the internal carotid artery, « fourth aortic arch on the left and proximal portion of the right subclavian artery on the right, « sixth~ proximal portion of the pulmonary artery on the left and the ductus arteriosus Nerve that runs along the radial artery: median nerve Artery affected in femoral neck fracture: medial femoral circumflex artery- damage leads to aseptic necrosis of the femoral head EM of egg: where does sperm penetrate (zona pellucida) Note: This material is copyrighted. All rights reserved. Edward Goljan, M.D. 2002 o aq qq er 94 a9 Types of collagen: « [- * bone, + tendon, * skin, * greatest tensile strength, * Il. « ‘of wound repair, * replaced by type I via collagenase with Zn as Sen . Tet collagen membrane, ¢ X- epiphyseal plate ‘basement Wallerian degeneration: ¢ sprouts are remyelinated and reestablish continuity with the motor end plate of the mune Cirete of Willis: name arteries and mow how to identify them on an angiogram CT liver: hepatic vein drains into the inferior vena cava Eye closed cannot open and eye deviated down and out: oculomotor nerve palsy; eye dowa and inz trochlear nerve palsy Blunt injury to the back of the throat: potential injury to the injury to the cervical sympathetic ganglion ptosis and meiosis of the right eye: Vertical diplopia: cranial nerve IV palsy Patient with headache and physical findings of mydriasis In the right eye in association with mild lid lag, and deviation of the eye down and oat: an aneurysm compressing cranial nerve II. headache is the giveaway for aneurysm Patient with a recent history of bacterial meningitis has horizontal diplopia in the left eye, which is worse on gaze to the left: cranial nerve VI palsy- lateral rectus weakness from VI nerve palsy Patient with bilateral lateral rectus muscle weakness: increase in intracranial pressure~ + classic sign, * papilledema usually present Paralysis of upward gaze in an infant: » hydrocephalus secondary to stenosis of the aqueduct of Sylvius, « this is called Parinaud’s syndrome Multiple ocular motor nerve disorders: diabetes mellitus- common cranial nerve palsies from osmotic damage to nerves Weakness of the quadriceps muscle and an absent knee jerk reflex: hemiated Ly-L. disk Fall on outstretched arm with pain in the middle and lateral portion of clavicle and upper extremity remains in abduction, extension, and internal rotatiou: © nerve injured is C;-Ce- Erb-Duchenne syndrome, or superior brachial plexus injury due to a clavicular fracture (most common fractures in newborns), ¢ Cy-T, are inferior brachial plexus injures or Klumpke’s syndrome Patient has paralysis of the ocalomotor nerve after a bead injury: uncal hemiation with compression of the Ilird nerve— ptosis of eye, mydriasis Numbness of the thenar aspect of the hand: median nerve (carpal nmnel) ‘Wrist bone with greatest incidence of aseptic mecrosis: scaphoid bone Fetal circulation: ductus venosus and umbilical vein have the highest oxygea content EM of alveolus with macrophage, type IT pnenmocytes (lamellar bodies [surfactant]) EM of small bowel: look for microvilli on the surface ; Histologic section of seminiferous tubule: Sertoli cell synthesizes sex hormone binding globulin and also synthesizes inhibin, which has a negative feedback with FSH Bands in skeletal muscle: A band has myosin ATPase: contracts Respiratory bronchiole: « last airway structure with cilia, © respiratory unit where gas exchange occurs is the respiratory bronchiole, alveolar duct, and alveoli J : Terminal bronchioles: ¢ cilia but no goblet cells, « site of obstruction in asthma, cystic fibrosis, and chronic bronchitis, ¢ site where turbulent air flow becomes laminar due to parallel branching of the airways ; 45 ‘Note: This material is copyrighted. All rights reserved. Edward Gollan, MD, 2002 4 94444 q 4q 4 94 gage 99s C444 G Gg aqaaq9 Normal EM of 2 cell: © know all the normal organelles in Gua, Posttranslational modification of various compounds, * rough ead eie roe aPparatus~ symtiesis, + smooth endoplasmic reticulum» ste of eyachrome PASO mee ee Poet steroids, * site of y-giutamyl transferase im * synthesis of Voice hoarseness post thyroid sargery: injury to laryngeal nerve MRI of orbit: location of the superior oblique muscle MRI of abdomen: splenic artery is above the pancreas Chest x-ray: fluid in the cestophrenic sulcus in CHE X-ray with enlargement of the posterior heart: « enlarged left atrium in a stenosis, * most posteriorly located chamber in the heart, » see best with raed il MRI of the carotids with occlusion of the anterior cerebral artery: would effect the contralateral leg Schwannoma in jugular foramen: © weakness of palate/loss gag reflewlaryngeal paralysis (X). tePezius/stemocleidomastoid (XD, «loss taste sensation posterior thd of tongue (CX) ° Weber syndrome: « lesion of oculomotor nerve and UMN signs, © usually aseovisted with « midline, midbrain lesion Horner's syndrome: + diagram of vertebra and sympathetic trunk, « pick superior cervian sympathetic ganglion Papil light reflexes with eye diagrams of pupils: one set is an oculomotor nerve lesion with the light in the pupil in a down and out location Brainstem anterior view: find arca of oculomotor ‘nerve Frontal fobe lesion; affects personality Inferior quadrantanopia: defect in the superior fibers in the parietal lobe [Loss of sensation in the bands, history of burus without knowing it: syringomyelia in cervical cord and involvement of crossed spinothalamics G, transection of fasciculus gracilis: effects vibration and fine touch of lower extremity only CN TIL and UMN signs on opposite side: midline midbrain lesion Loss of pain and temperature and UMN signs on opposite side: mid pons lesion Horner’s syndrome with localization of lesion on a diagram: iateral medullary syndrome with associated cranial nerve palsy in the medulla and hypothalamus with associated temperance regulation problems Parkinsons disease: MPTP drug of abuse association~ derivative of meperidine ‘Cross-section of an embryo: identify neural crest tissue Blood production prior to birth: yolk stc-> liver» bone marrow Bochdalek hernia in posterolateral part of diaphragm on left: © present carly in life, « visceral contents extend into the chest cavity causing severe respiratory distress at birth, « parasternal diaphragmatic hemias extend through the foramen of Morgogni beneath the sternum and do not usually develop syerptoms until later in life Artery associated with foregut, midgut, hindgut: + foregut— mesenteric, » hindgut~ inferior mesenteric Damage to hearing in a rock and roll band player: injury to cochlea Small vs large bowel: plicae semilumaris go around entire circumference of small bowel and are interrupted in large bowel Argyil-Robertson papil: accommodates when patient follows finger moving towards the nose but does not react to direct light; neurosyphilis MRI of liver: hepatic vein- vessel emptying it he inferior er cva Pathogenesis of hypospadias: faulty closure of urethral fol Pathogenesis of eplapadias: defect genial tubercle assisted with exstrophy of bladder Feces draining from umbilicas in 4 day old: persistent umbilical (vitelline) sinus Urine draining from umbilicus in 4 day old: persistent urachal sinus midgut superior 46 a4 a4 eaq4 Femoral neck fracture: * bleeds into the capsule and may compromise medial femoral circumflex artery leading to avascular necrosis Embryology of miillerian duct, wolffian duct, urogenital sinus: « structures derived from the Mullerian duct include- * fallopian tubes, * uterus, * cervix, * upper 1/3rd of vagina, * in male fetuses, milllerian inhibitory factor (MIF) causes regression of the millerian duct by apoptosis, « in male fetuses, the paired wolffian (mesonephric ducts) form the epididymis, seminal vesicles, and Vas deferens * testosterone controls this, * dihydrotestosterone develops the external genitalia of the male fetus and the prostate gland, « in female fetuses, the wolffian duct normally regresses— inclusions may persist to form cysts (e.g., Gartner duct cyst in the vagina/cervix), * structures derived from urogenital sinus include * vestibule, + lower 2/3rds of the vagina ‘Anatomy/histology of ovary: « lined by coelomic epithelium surface derived ovarian tumors derive from this (€.g, serous cystadenoma), * outer cortex contains the follicles , * medulla contains mesenchymal tissue and steroid-producing hilar cells~ stromal thecosis of the hilar cells leads to hirsutism, « ovaries in a postmenopausal women should not be palpable— enlarged ovary is cancer until proven otherwise Oogonia at birth: + arrested in prophase I of meiosis and are called primary oocytes, + arrested cells are diploid (2n) and have 46 chromosomes Primary oocyte: « primary oocyte completes the first meiotic division within the mature follicle shortly before ovulation» secondary oocyte, + secondary oocyte is haploid (In) and contains 23 chromosomes, « it enters the second meiotic division but is arrested in metaphase II at the time of ovulation, * the secondary oocyte completes meiosis II during fertilization “Anatomy/bistotogy of cervix: « cervix = endocervix (mucous secreting columnar cells) + exocervix (squamous cells, begins at cervical os)}- squamocolummar junction (SCJ) is where the two epithelium's meet, « endocervical epithelium normally migrates down into the exocervix and replaces the normally squamous epithelium with mucus secreting colummar epithelium ° columnar cells exposed at the cervical os undergo squamous metaplasia (called the transformation zone), * the TZ is the area where squamous dysplasia end cancer develop Bilateral internuclear ophthalmoplegia: ¢ demyelination of medial longitudinal fasciculus, pathognomonic of multiple sclerosis, « when the patient is asked to look right, the right eye moves and has jerk nystagmus but the left eye is still looking straight abead, * when the patients asked to Took left, the left eye moves and has jerk nystagmus but the nght eye is stil looking straight ahead Rotator cuff tear: + common cause of shoulder pain, « components of the rotator cuff include the tendon insertions of- # supraspinams, * infraspinatus, « teres minor, * subscapularis muscles, « S/S of rotator cuff injuries~ pain/wealmess with active shoulder abduction ‘Shoulder dislocation: « majority are anteriorly dislocated, « often injure the axillary arvery/nerve Teanis elbow: « pain occurs in the area where the extensor muscle tendons insert near the lateral epicondyle, « common in— * raquette sports, * repetitive use of a haramer or screwériver Golfer's elbow: pain is located where the flexor muscle tendons insert near the medial epicondyle Ulnar verve compression: * the ulnar nerve may be entrapped in the following areas * transverse carpal ligament, * elbow (“funay bone area"), © there is pain and numbness of the ulnar aspect of the forearm and ring and litle finger plus weakness ofthe intrinsic muscles of the hand Reflex loss in C.-C; disc: biceps reflex associated with musculocutaneous nerve Cs-Cs Reflex loss in Cs—C, dise: supinator 3 Reflex loss in C.-C; disc: triceps reflex associated with radial nerve Ce-Ce DeQuervain’s disease: « chronic stenosing tenosynovitis of the first dorsal compartment of the wrist, « duc to overuse of the hands and wrist, « the first dorsal compartment contains the abductor pollicis longus (APL) and the extensor pollicis brevis (EPB)- excessive friction causes thickening 47 Note: This material is copyrighted. All rights reserved. Edward Goljan, M.D. 2002 of the tendon sheath and stenosis ofthe osseofibrous tunnel, « pain occurs on the radial aspect of the wrist and 1s aggravated by moving the thumb~ pain inthe region ofthe radial styloid process © — Ganglion cyst: » common cause of a bulge on the dorsum of the wrist when the wrist is flexed, © ganglion” (synovial) cyst is filled with mucinous material, © somet abil en ckenvanlts puichite bye ees Ea he, ihe ra > — Compartment syndromes: + increase of pressure in a confined space pressure reduces perfusion, which may lead to permanent ischemic contractures of the muscle (s) in that commpartment, « S/S of a compartment syndrome~ * pain, * paresthesias, * pallor, + paralysis, * pulsclessness, « causes of compartment syndromes- * fractures, * injuries to arteries/soft tissue, * prolonged limb compression (¢.g.. tight fitting cast), « Volkmann's ischemic contracture~ * it is a complication of a supracondylar fracture of the humerus, + there is injury to the brachial artery and median nerve, © brachial artery ischemia leads to increased pressure in the closed muscle compartments of the forearm with a subsequent decrease in venous and then arterial perfusion: this may lead to permanent ischemic contractures of the muscle % Carpal tunnel syndrome: + entrapment syndrome of the median nerve in the transverse carpal ligament of the wrist, * it may also be entrapped between the bellies of the pronator teres muscle along with the radial artery, * causes~ * RA/pregnancy, * overuse of the hands/wrist, © amyloidosis, * hypothyroidism, ¢ S/S—* pain, mumbness, or paresthesias in the thumb, index finger, second finger, third finger, and the radial side of the fourth finger, * thenar atrophy ("ape” hand appearance), + pain is reproduced by tapping over the median nerve = "Claw hand": « ulnar nerve palsy, « ulnar nerve functions— * adduction of the fingers due to innervation of the palmar and dorsal interosseus muscles, * adduction of the thumb (adductor pollicis muscle) = "Wrist drop”: « radial nerve palsy, « radial nerve fumctions- * extensor muscles of the wrist and digits, + wrist drop refers to a hand that is flexed at the wrist and cannot be extended, injury may bbe due to- * midshaft fractures of the humerus, * draping the arm over a park bench (called “Saturday night palsy") + "Waiter's tip deformity": « brachial plexus lesion involving C; and Cs upper trunk injury, © clinical findings in Erb-Duchenne's palsy~ * birth injury of the brachial plexus with damage to Cs-Ce nerves upper trunk injury, + loss of abduction of the arm from the shoulder, * inability to externally rotate the arm, ¢ inability to supinate the forearm, * absent biceps reflex. * asymmetric Moro reflex: no movement on the affected side & — Klumpke's paralysis: « birth injury of the brachial plexus with injury to the CC, and T,- lower trunk injury, paralysis of the hand, « Homer's syndrome Axillary nerve injary (C.-C): « fracture of the surgical neck of the humerus, * dislocation of the shoulder joint * usually anterior dislocation, * may also injury the axillary artery, » patient cannot abduct the arm to the horizontal position or hold the horizontal position when a downward force is applied to the amm- paralysis of deltoid muscle, * weakening of lateral rotation of the arm~ paralysis of teres minor muscle = Arteries, nerves, tendons cut with a deep laceration of the radial side of the wrist: « srtery— radial artery, « nerve~ median nerve: courses along the radial artery, « tendons * palmaris longus, + flexor carpi radialis 2 Arteries, nerves, tendons cut with a deep laceration of the ulnar side of the wrist: « arery- ulnar artery, * nerve— ulnar nerve, « tendon flexor carpi ulmaris ; & — Cause of “shin splints": repetitive loading of the anterior compartment muscles of the tibia * inflammation occurs at the musculotendinous insertions: these are often called "stress fractures" © — MC site for a compartment syndrome in the leg: anterior compartment of the tibia 48 Note: This material is copyrighted. All rights reserved. Edward Coljan, M.D. 2002 Mechanisms of low back pain: * MCC is spasm of the paraspinal anterior/posterior longitudinal ligaments, © nerve root irritation with nerve root compression, * diseases involving vertebral col myeloma, * osteoporosis with compression fractures = ees eee disc siete degeneration of fibrocartilage/nucleus pulposus + the nip isc material may hemiate posteriorly and compress the nerve root andor Spinal cord. Pain (sciatica) radiates from the low back to the buttocks, down the leg, below the knee, « §/S of ise herniation + leg pain is aggravated by straight leg raising * 2 — Herniation of (Les i somo loss~ » lateral and posterior calf, « plantar aspect of the foot, + reflex loss— Achilles’ reflex (tibial nerve L,-S,), « itm sat ee ed al ne Li-Si), « motor defict~ « loss of plantar flexion, «loss ‘7 Herniation of Lil dise: + sensory loss * dorsum ofthe foot, + webbed space between the great toe, « reflex loss none, » motor deficit loss of dorsiflexion of the big (great) toe due to weakness of the extensor hallueis longus * Herniation of Ly-Li disc: + sensory toss~ medial leg to the malleolus, « reflex loss~ knee jerk (femoral nerve Ly-L,), * motor deficit- * quadriceps weakness due to weakness of knee extension, + loss of dorsiflexion of the foot due to weakness of the tibialis anterior Cauda equina syndrome; » bowel or bladder dysfunction, « saddle area anesthesia Spondylolisthesis: + forward subluxation of one vertebral body on another, © MCC is spondylosis knee is acutely flexed-> foot is grasped and the leg is extemally rotated.» lore ve slowly extended while the other hand feels the posteromedial margin of the knee joint-> a slick along the posteromedial margin indicates a medial meniscus tear: femur passes over the tear = test. for a lateral menisous injury: same procedure as above except the leg is rotated intcmally and extended-+ a click is palpated along the posterolateral margin of the joint Location and function of the anterior cruciate ligament (ACL): ACL anaches the anterior part of the ubis to the fateral condyle of the femur— prevents anterior movement of the tibia in relation vo the femur Location and function of the posterior cruciate Ligament (PCL): PCL extends from the posterior part of the tibia to the medial condyle of the femur prevents posterior movement of the nbia in relation to the feraur Tests used to evaluate the cruciate ligaments in the knee: « anterior draw test to evaluate ACL— patient supine, hip flexed 45°, knee flexed 90°» examiner places hand on the posterior aspect of the Ubia—> anterior force is applied in neutral, external, internal direction» positive anterior draw test is when there is anterior displacement of the tibia, « posterior draw test to evaluate PCL— patient supine, hip flexed 45°, knee flexed 90°» examiner places hand on the anterior aspect of proximal tibia-> posterior force is applied in neutral, external, internal direction positive posterior draw test is when there is posterior displacement of the tibia Functions of the medial collateral ligament: supports the medial side of the knee joint- « attaches the medial epicondyle of the femur with the shaft of the tibia, * resists valgus and extemal rotational forces of the proximal tibia in relation to the distal femur Functions of the lateral collateral ligament: supports the lateral side of the knee joint « attaches the lateral epicondyle of the femur to the head of the fibula, * resists varus forces and rotational forces of the proximal fibula in relation to the distal femur S/S of a meniscus injury: © pain, + knee catches, locks, or gives way when walking, © swelling/popping of the nee Medial meniscus injury: + mechanism of injury~ * MC internal derangement of knee joint, « most _ commonly part of a valgus injury (¢.g., clipping in football), « structures damaged- + medial meniscus, * medial collateral ligament, * ACL, # positive McMurray's test~ click on posteromedial margin with the knee flexed, extemaily rotated, and slowly extended Lateral meniscus injury: * mechanism of injury— * varus injury, + injury to lateral collateral ligament, * positive McMurray’s test- click on posterolateral margin with the knee flexed, extemally rotated, and slowly extended 50 Note: This material is copyrighted. All rights reserved. Edward Gatjan, M.D. 2002 & ACL injury: + mechanism of injury + MC ligament injury, + to injury secondary to clipping or skiing, © positive anterior dra sign mm most commonly ina valgus 7 PCL injury: « mechanism of injury- + hyperextension of the lnce secondary pushing the tibia in a posterior direction, « positive posterior draw eat” ‘oancnanetie eres = Medial collateral ligament injury: tom with valgus injuries % Lateral collateral ligament injury: tom with varus injuries @ — MCC of am ankle sprain: sprain of lateral ankle ligamen sakeabs in Dashaeee, players, volleyball, ane foottall from inversion of plantar Axed fot > MC lateral ligament that is sprained: anterior talofibular ligament very stabilization of planar flexion in the foot Har Hgersat- very important ligament in Positions the knee joint i forced into that results in injury: « valgus positon~ angulation avay from the midline g, clipping injury in football, « varus position—angulation towards the midlng, medially originating force is applied to the knee MC nerve injured with clavicular fractares: ulnar nerve MC nerve injured in proximal humerus fractures: axillary nerve MC nerve injured in mid-shaft/distal third of humerus: radial nerve~ * nerve travels in the spiral groove, * wrist drop = MCC of pain in the elbow and inability to supinate the forearm in 1-4 yr old: subluxation of the head of the radius~ usually due to jerking 8fUie hand By an imipabent Sr abusing parent — 7 MC fracture associated with falling on the outstretched hand: * Colles fracture of the distal radius * MC fractare of the wrist, + radiologically, it produces a “dinner fork” deformity of the proximal radial fragment (displaced upward and backward), * second most common Face un osteoporosis in women * MC carpal bone fracture: scaphoid + pain in the anatomical snuff box located below the radial styloid process, * high incidence of aseptic necrosis 7 MC hip dislocations: = posterior usually due to a car accident with the flexed knee in an abducted position is forced into the dashboard, * limb is shortened, flexed, adducted, internally rotated at the hip, * danger of damage to sciatic nerve, » anterior « limb is shortened, » abducted, + extemally rotated at the hip, * neurovascular compromise of the femoral artery, vein, nerve, « aseptic necrosis of femoral head MC femoral fracture: + feroral neck fracture~ * it most commonly occurs ia the elderly male patient with osteoporosis, * groin or knee pain is present, + complications aseptic necrosis of femoral head due to damage to the medial femoral circumflex artery 7 — MC foot bone fractured after a fall from a height: calcaneus + MC fracture associated with ecchymoses of the mastoid, basilar skull fracture: + petrous portion of the temporal bone, « otorrhea (CSF fluid leaking out of the ear) may also occur). 7 MC fracture associated with rhinorrhea: » orbital fractures, © orbital fractures also produce raccoon eyes (periorbital hemorrhage and ophthalmoplegia (eye muscle entrapment). 7 Cross-section of spinal cord: + location of pain in right hip, © location for sympathetic preganglionic fibers & Shining tight in left eye causes pupil to constrict in left but not the right eye. When shining light in right eye, pupil constricts in the teft but not the right eye: right CN Ili dysfunction o-~-Pattene wittr spasticity-aud loss'of tactile dlsctitithatoW and vibratory sensation: subscilté combined degeneration in By, deficiency = Foreign body in alveolus: phagocytosed by alveolar macrophage 2 Site for hemineglect: right parietal | = Patient smells rubber. Mass located in left temporal lobe. Visual field defect: right upper quadrantanopia © Pineal gland: site for melatonin formation a44 SI Note: This material is copyrighted. All rights reserved. Edward Goljan, M.D. 2002 v4 94 944 Superficial cerebral veins: drain into superior sagital sinus Site odontoblasts develop from: * odontoblasts develop into dentin, « odontobl aspect of the developing tooth, * outer portion develops enamel from ameloblasts lasts are on inner HIGH YIELD NOTES BIOCHEMISTRY © Hep finsesavaea « type Land V have chylomicrons associated with them, © type Tis increase in LDL, # is remnant disease (dysbetal . Vand ope V ape (dysberalipoproteinemia), « increase in VLDL in type PCR mechanism: uses DNA polymerase to break down DNA into fragments Locations of biochemical processes in cell: « cytosol * glycolysis, * pentose phosphate shunt, « FA synthesis, * glycogen synthesis, * mitochondrial matrix * B-oxidation of FAs, « TCA cycle, inner mitochondrial membrane- oxidative phosphorylation, © both cytosol and mitochondria « gluconeogenesis, * urea cycle, * heme synthesis ‘Neurotransmitter from an essential amino acid: serotonin coming from tryptophan Brain energy during starvation: * ketone bodies, » uses glucose during fed and fasting state Gluconeogewic enrymes: * pyruvate carboxylase, * phosphoenolpyruvate carboxykinase, « fructose 1,6-bisphosphatase (rate limiting), * glucose 6-phosphatase (deficient in von Gierke's glycogenosis) RBC ribosomes: + lost after RBC leaves bone marrow, * persistence in peripheral blood produces basophilic stippling, * coarse basophilic suppling sign of Pb poisoning (ribonuclease denatured by Pb) B Thalassemia mechanisms: « most often a splicing defect for mild forms, « severe B-thal is due toa stop codon preventing 8-chain transcription Man with 2900 calorie diet with 30% of it representing fat, how many grams is fat: © fat has 9 cal/g, + 2900 x 0.30 = 870 calories is fat, « 870 + 9 =~97 grams Vas and Ke: © glucokinase high Km (low affinity for glucose) and high Vm (only reacts with glucose), « hexokinase low Km (high affinity for glucose, good for fasting’state) and low Vin (reacts with all hexose sugars) Essential fatty acids (FAs): * linoleic acid- + C18:206, * produces arachidonic acid, * not cardioprotective, + com oil/safflower oil, ¢ c-linolenic— = C18:303, * cardioprotective (lower triglycerides, inhibit platelet aggregation, produce anti-inflammatory prostaglandins, less damage to myocardial tissue in infarctions, * found in fish oils, canola oil (best oil) I cell disease: inability 10 phosphorylate the mannose residues of potential lysosomal enzymes located in Golgi apparatus, hence they cannot be taken up by the lysosomes to degrade complex substrates ber of glucoses necessary to build palmitic acid a 16 carbon compound: 4 glucoses, cach glucose rum produces 2 acetyl CoA, the latter containing 2 carbons cach - Insalin lack in DKA: decreased glycolysis, glycogenesis, farry acid synthesis, storage of fat in adi Compara Chart of the Well Fed State, Fasting State, and Starved State: Well fed state ~ Fasting state [Starved state Glycogenesis Increased fone ‘None Glycogenolysis Decreased: none in| Increased: early supply | None: glycogen used up the liver, some in _| of glucose derived from muscic liver not muscle $2 Note: This material is copyrighted. All rights reserved. Edward Goljan, M.D, 2002 ‘Wall fed state Fasting state Sin Gluconeogenesis None Increased: primary] Beteased jan cnan = ] source of glucose after | supply RBCs lysis Triacylglycerol synthesis | increased None Tee : in liver/adipose Lipolysis None Tnereased [increased Fate of glycerol ‘Synthesize more ‘Substrate for Substrate for riacylglycero! in liver | gluconeogenesis [eee | Brosidation of arty acids | None Increased Markedly increased: primary fuel for muscle Muscle catabolism None: inereased Increased: supply amino | Decreased: conserve | protein synthesis and | acids for ‘muscle for important body | t uptake of aming acids | gluconeogenesis functions i ‘Urea synthesivexcretion [Remains constant: | Increased: deamination | Decreased: less muscle | handles NH." load | of amino acids used for | breakdown of protein with | from protein gluconeogenesis less amino acids to degradation in gut by | increases urea synthesis | degrade 2 bacteria i ‘Ketone body syathesis | None Tncreased Markedly increased: by- | product ofacetyiCoA from increased B- | : oxidation of fatty acids | [Muscle use of glucose for | Primary Fuel Decreased ‘None: mainly uses fatty fuel . acids )Musele use of fatty acids | None Increased: primaty fuel {Markedly increased, for fuel primary fuel ! jMasele use of ketones | None ‘Some: alternative fuel | None: allows the brain to for fuel use ketones for fuel Brain use of glucose for [Remains constant | Remains constant Decreased: allows RBCs | | fuel to primarily use glucose for fuel [Brain use of Ketones for, [None None Increased: primary fust fuel i TRBC use of glucose for [Remains constant | Remains constant Remains constant 1 fuel ; = Acquired and Genetic Hyperlipoproteinemias: Lipid Disorders ‘Clinical Comments [Laboratory Findings —____ ‘Type Rare childhood disease. Tnoreased chylomicrons and TG with + Familia! lipoprotein ‘anormal cholesterol and LDL, «stan | | lipase deficiency ing chylomicron test: supranate but no | Apo ci pyaar Pathogenesis: cannot hydrolyze chylomicrons 53 Note: This material is copyrighted. All rights reserved. Edward Goljan, MLD. 2007 Lipid Disorders ‘Glinical Comments Laboratory Findings Type tl + AD disorder with premature | » Type Ila: increased ‘+ Familial hypercholester- | CAD. « Achilles tendon xanth- melt) and Eanes To. ‘olerma ‘omas pathognomonic, * Rx: type Ib: increased LDL, cholesterol, Pathogenesis: absent or | “statin” drugs and TG. " defective LDL receptors | « Acquired causes: diabetes, 2 hypothyroidism, obstructive i jaundice, progesterone in birth | ‘control pills. i Type * Increased CAD risk, » hyperun- |» Choleserol and TG equally levarsa. = Famitial cemia, + obesity, « diabetes + crease in chylorucron and IDL dteclipoprotenemia remnants, ¢ ultracentrifugation followed | remnant disease” lectrophoresis eran Pathogenesis: deficiency we “donates ™ ‘of apo E. Chylomicron and DL remmants are not metabolized in the liver. Type IV ‘+ AD disorder, « most common | « Marked increase in TG and slight + Familial byperlipoproteinemia, « TG increase in cholesterol, « standing hypertnglyceridemia begins increasing at puberty, | chylomicron test: turbid infranate, « Pathogenesis: decreased | increased incidence of CAD and | HDL decreased (inverse relanoaship caabolism or increased | perspheral vascular disease, « Rx: | with VLDL). | synthesis of VLDL. fibric acid derivatives, reduce carbohydrate and alcohol intake. + Acquired causes: alcobolism, diuretics, B-blockers, renal failure. Type ¥ ‘+ Particularly common in ‘+ Markedly increased TG with normal Most commonly a alcoholics and DKA,* hyper | LDL, « standing chylomicron test. familial chylomicronemia syndrome: supranste and infranate hypertriglyceridemia with | abdominal pain. pancreatitis, exacerbanng factors dyspnea (impaired oxygen Pathogenesis: exchange), hepatospleaomegaly combination of type Iand | (fatty change), papules on skin. type IV mechanisms. CAD = coronary artery disease, DKA = diabetic ketoacidosis, IDL = imcrmediare density ipopromm, LDL = low density lipoproteins, Rx = teament, TG - triacylglycerol, VLDL * very low density lipoproteins ‘Rate limiting reaction in cholesterol synthesis: « HMG Co reductase: inhibited by cholesterol and statin drags (competitive inhibition with mevalonic acid), = enzyme converts HMG CoA into mevalonic acid Rate limiting reaction in fatty acid syathesis: » acetyl CoA carboxylase, * enzyme converts acetyl CoA into malonyl Com ~ Rate limiting reaction in Boxidation of farry acids: « carnitine acyltransferase I, ¢ enzyme in ‘outer membrane of mitochonéria removes acy! group from farty acyl CoA and wansfers it 10 cammitine (acyleamitine), © mner membrane enzyme removes acyl group from acylcamitine and transfers it back to CoA to produce fatty acyl CoA Rate limiting reaction in glycolysis: « phosphofructokinase [, + enzyme converts fructose 6 phosphate into fructose 1,6-bisphosphate q q Rate limiting reaction in gluconeogenesis: © fructose 1,6-bisp! fructose 1,6-bisphosphate to fructose 6-phosphate tose |.6-bisphosphatase, * enzyme convens Rate limiting reaction in glycogen synthesis: « . glycosidic linkages between a glucose unit from ior eae nonredci or oad existing glycogen chain icing end of an Rate Umiting reaction in glycogenolysis: « liver eer (releases glucose I-phosphate) but stops working Apietiecia from seen aiaess Rate limiting reaction in pentose phosphate pathway: « ae Peay tece te wiasece B phenrtat Wed phngieglengets Bait Sehesclieelapiosgrume Rate limiting reaction in lipolysis: * hormone sensiti 3 ane miacylglycerol into farty acids and glycerol ves geal, Rate limiting reaction in urea cycle: carbamyl phosphate F enzymeeon NEL + ATP ino carbamyl phosphate mi phosphate syrihase I converts COs + Rate limiting reaction in pyrimidine metabolism: « carbamy! ph acai see Teens . Rate limiting resction in purine metabolism: + glutamine PRPP aminotransferase, + enzyme converts PRPP + glutamine into 5‘-phosphoriboysylamine Rate limiting reaction in heme synthesis: « ALA synthase, * enzyme converss succinyl CoA + glycine into 5-aminolevulinic acid Rate limiting reaction in ketone body synthesis: © HMG CoA syathase, * converts acetoacetyl CoA into HMG CoA Tncoupling agents: « render the inner mitochondrial membrane permeable and carry protons with then into’ the mitochondrial matrix (destroys the proton gradient): eg, dinitrophenol, pentachlorophenol (used 10 teat wood to prevent insect invasion), thermogeain (natal incoupling agent in brown fat of newborns that helps keep newborns intemal temperatures highet), « rate of chemical reactions increases to produce more NADH and NADPH since its being siphoned off into the mitochondrial matrix without the synthesis of ATP—> poteatial for typerthermia, «mitochondrial poisons (aleobol, slicytates) also reader the inner mitochondrial wot iorane permeable to protons, but they do not directly take protons with them into the matrix (not true uncoupling agents): | ATP synthesis ejent with pheochromocytoma: « decrease tyrosine (not an essential AA) in the diet, since itis the precursor for the catecholamines, « also decreased ascorbic acid which converts dopamine into norepinephrine Tesch Nyban: « SXR with absent HGPRT, + self-mutilation, » hyperuricemia, ¢ mental retardation Glucokinase and hexokinase: « glucokinase: only in liv, high Vm and high Km, nat inhibited by Slucose é-phospbate, « hexolanase: in all tissues, inhibited by glucose G-phosphate, low Vm and Tow Kim Branched chain amino acids and maple syrup urine disease: ¢ only muscle can metabolize teanched chain amino acids, © missing dehydrogenase enzyme Locations of glucose 6-phosphatase (glacomeogenic hormone): + liver, * kidney, + itesunal epithelium (lesser extent than others), + absent in voa Gierke’s disease qeaities and what they carry: + camitine- even chained fay acids, malate and glyoerel 5 ~ pkosphaté—NADE™ venedons of cholesterol: « vitamiri D synthesis in the skin (7-dchydrocholesterol), © steroid synthesis, © cell membranes, + synthesis of bile saltsacids ‘Acetyl CoA uses: # FA synthesis, » CH synthesis, « ketone body synthesis. ¢ synthesis of citrate along with oxaloacetic acid, + not a sut 35 re ) Ketone body synthesis: + FIMG CoA synthase is the ratelimiti converts HMG CoA into acetoacetic acid, while in cbileirsl atte ieee lyase | converts HMG CoA into mevalonic acid, « 23 ATP produced for fuel a Enzyme kinetics: * Vu. ropresents the maximum velocity of an sites are fully saturated, » K., (Michaelis-Menton constant) of an enzyme concentration at which the reaction velocity is one-half of Vmax (Vina) ie dae enzymes affinity for substrate, + increased Ka indicates decreased affinity of the ee | substrate, + decreased Ky indicates increased affinity of the enzyme for subsets Competitive inhibitors: * Vies is not changed since the reaction rate is unchanged whether the competitive inhibitor (e.g. ethylene glycol, methyl alcohol) is binding to the active site of the enzyme (e.g, alcohol dehydrogenase) or alcohol, + Ky increases (decrease in the affinity of the enzyme for alcohol), since the enzyme is also actively binding with ethylene glycalimethyl alcobel + infusing alcohol decreases the metabolism of ethylene glycol/methyl alcohol, hence the effect of a competitive inhibitor is reversed by increasing substrate, + methotrexate is also 2 compentive itor Reversible noncompetitive inhibitors: * Vea is decreased, however the Ke remains the same, « reversible noncompetitive inhibitors bind reversibly (non-covalent bonds) away from the active binding site of the enzyme and form either unreactive enzyme-inhibitor complexes or enzyme. substrate-inhibitor complexes, + affinity of the enzyme for substrate is unchanged (K,.), since the active binding site is unaltered and normally binds with substrate, # Vy is decreased since the inhibitor inactivates the enzyme, which automatically decreases the eflective covcentration of active enzyme, * increasing substrate concentration does not reverse the effect of the inhibitor, since the inhibitor is blocking enzyme activity away from the active binding site, » examples of reversible noncompetitive inhibitors include angiotensin converting enzyme inhibitors (substrate is angiotensin I); physostigmine, which is a cholinesterase inhibitor (substrate acetylcholine); and, allopurinol, an xanthine oxidase inhibitor (substrate xanthine), * irreversible inhibitors permanently inactivate enzymes by forming strong covalent bonds: examples include lead (irreversibly inhibits ferrochelatase and S-aminolevulinic acid dehydrase), aspirin (irreversibly — inhibits platelet cyclooxygenase), and organophosphates (irreversibly inhibit cholinesterase), enzyme Kinetics are the same as those for reversible noncompetitive inhibitors ATP count using palmitic acid as an example: (1) _ divide the number of carbons in the FA by 2 to arrive at the number of acetyl CoAs produced: | palmitic acid = 16 carbons 8 acetyl CoA. (2) cach acetyl CoA produces 12 ATP when converted to CO; and H,0 in the TCA cycle: 8 x 12 | =96 ATP (3) subtract 1 from the number of acetyl CoA produced to arrive at the number of NADH: and FADE) produced: 7 NADH +7 FADE; (4) each NADH produces 3 ATP: 7x3 =21 | (5) each FADH; produces 2 ATP: 7x2= 14 (©) tol ATP= 131 ; ' Urea cycle: « method of eliminating ammonia, « located in the hepatocyte, » chronic liver disease: low BUN, elevated ammonia Epinephrine given and only small branched chaios of glycogen found: debrancher enzyme | deficiency 4 Cholesterol synthesis: first few steps are similar to ketone body synthesis except HMG CoA Tyase is used instead of HMG CoA reductase Apolipoprotein 100 (liver) and 48 (intestine) | 56 944 a4 Tasulin: * key hormone of the fed state,» activates phosphatase i" Glucagon: » key hormone of the fasting statc, « eae pel ae ‘ohotpy Mannose 6 phosphate: « involved in transfer of dolichol (lipid) in the =e linked glycosides, involved in transfer of lysosomes from Golgi a acid—arginine Mechanism of ketoacidosis in DKA: increased ti i CoA, which is used by the liver to synthesize Sater Of any acid and prodveton of seen Promoter location: upstream location Energy source for protein synthesis: GIP Isvenzyme with 2 genes, 4 subunits: LDH isoenzymes; 5 isotypes: LLLL, LLLH, LLHH, LHEDH, Second messengers: atrial natriuretic peptide: eGMP, insulin: tyrosine kinase, nicotinic: ion channels Best method of detecting relatedness of a new bacteria: restriction fragment Iength polymorphism Biochemistry of hepatic encephalopathy: * increase in aromatic amino acids phenylalanine, tyrosine, tryptophan (mnemonic— PTT) Icads to increased synthesis of false neurotransmutters (GABA, octopamine) in hepatic encephalopathy, « branched chain amino acids inhibit synthesis of false neurotransmitters- reason why they are given for Rx Energy in cardiac muscle: B-oxidation of fatty acids Lipid facts: * fimctions of HDL- = reservoir for apolipoproteins in the blood, * transport of esterified cholesterol to the liver, * transfers cholesterol esters to VEDL in exchange for triglyceride using cholesterol ester transport protein, « taken up by scavenger receptors in the liver, * in patients with diabetic ketoacidasis you would expect... activation of hormone sensitive lipase in the adipose, conversion of glycerol 3-phosphate into dihydroxyacetone phosphate, «in the fasting state, you would expect~ loss of inhibition of camitine acyltransferase by malonyl-CoA, + in the fed state, you would expect- * activation of citrate lyase in the cytosol, * inactivation of hormone sensitive lipase in adipose, + increased production of palmitic acid in the cytosol Findings in PKU: « AR discase with deficiency of phenvislanine hydroxylase, * newborn must be exposed to phenylalanine (PHY) in the diet before it is increased, © mousy odor, « projectile vomiting simulating congenital pyloric stenosis, « tyrosine missing, hence it must be supplied in the diet, + can diagnose by amniocentesis and finding the abnormal gene, « eliminate phenylalanine from dict Nuzasweet is aspartate and phenylalanine, s0 cannot use it, « woman with PKU who is pregnant must be on a PHY free diet— affected fetus will develop permanent CNS damage inf utero owing to exposure of PHY . Disorders of galactose metabolism: * galactose derives from lactose metabolism lactose + lactase glucose + galactose, » galactose metabolism in sequence is as follows- * galactose + galactokinase > galactose 1-PO,, + galactose 1-PO, + GALT (galactose-I-phosphate uridyl wansferase) + UDP-glucose glucose 1-PO, + UDP-galactose, + glucose 1-PO. + ST Note: This material is copyrighted. All rights reserved. Edward Goljan, M.D, 2002 phosphoglucomutase —> glucose 6-PO, (6 carbon intermediate) + phosphatase (gluconeogenic enzyme)—+ glucose, « cibeiings Vanden be 2S {positive urine Clinitest: Clinitest detects all reducing sugars except sucrose, hich i cant reducing sugar, ~ galactosemia~ + AR disease with deficiency of GALT. « excew galaciows Converted into the galactitol (poiyol or alcohol sugar), which, like sorbitol, is osmotically seauee damages lens, nerve tissuc, CNS, liver, » excess galactose 1-PO, is toxie: eithonis, mental reardation, renal damage, neonatal hypoglycemia (lack of glucose 6:PO,, « subseate for gluconeogenesis), * Rx is a lactose fice dict for the first two yrs, + 1 galactosemis can synthesize lactose in their breast milk via the following reactions: UDP-glucose + UDP-hexose epimerase > UDP-galactose, UDP-galactose + lactose synthetase-+ lactose + UDP 7 Disorders in fructose metabolism: + fructose metabolism in sequence- * sucrose + sucrase + glucose + fructose, * fructose + fructokinase —» fructose 1-PO,, + fructose 1-PO. + aldolase B + glyceraldehyde 3. phosphate + dihydroxyscetone phosphate (DHAP, both are 3 carbon intermediates that are gluconeogenic tc9), + fructose can be synthesized from mannose (and vice versa) and sorbitol, + fructose is an essential nutrient for sperm stored in the seminal vesicles, « essential fructosuria~ * AR disease with missing fructokinase, * positive urine Clinitest due to fructose, * heredi clase intolerance~ * AR disease with a deficiency of aldolase B. accumulation of fructose 1-PO,, which is toxic to the liver (cirrhosis), * fasting hypoglycemia due to a decrease in 3 carbon intermediates for gluconeogenesis, = severe hypophosphatemia: excess fructose traps phosphate in cells, depletion of ATP leads to RBC hemolysis and rhabdomyolysis, increased adenosine monophosphate (AMP), which is a purine, is converted into uric acid leading to gout, * must elit table su 1m the dict 2 Sorbitol: « osinoically active solute that is synthesized in those tissue containing aldose reductase, which include~ « fens, * ova, * seminal vesicles (note in the biochemical reaction listed below how glucose is converted into fructose in the seminal vesicles), » Schwann cells, * reuna, * kidneys, © aldose reductase converts glucose into sorbitol and sorbitol dehydrogenase converts sorbitol into * fructose- NADPH _,NADP'NAQ’ NADH: Glucose Soe Fructose aldose reductase —_sorbitol dehydrogenase «in hyperglycemic states, like diabetes mellitus, there is an excess of sorbitol produced in the above tissues leading to osmotic damage * cataracts, * peripheral neuropathy duc to destruction of Schwann cells, « microaneurysms in retinal vessels due to destruction of pericytes = Homocystinuria: * AR disease with a deficiency of evstathionine sunthnse, + metabolism of homocystine in sequence is as follows- + methlonine + ATP —> S-adenosylmethionine (SAM) + methyl acceptors + methyleransferase —> S-adenosylhomocysteine + methylated products (donates tmeihyl groups for | carbon transfers), © S-adenosyihomocysteine + HO -+ homocysteine + idenosine, * homocysteine + serine + cystathionine synthase ~> cystathionine, + i= homocystinuria, both homocysteine and methionine are increased in serum, © SS of homocystinuria that resemble Marfan syndrome (example of genetic heterogeneity) * dislocated lens, « aracinodactyly, + eunuchoid, » distinctive features of homocystinuria~ * increase in plasma homocysteine levels leads to vessel damage/thrombosis (strokes, AMI, * mental retardanon, « increased urine homocysteine and increased serum/urine methionine . + Aleaptonuria and hereditary tyrosinosis: + metabolism of phenylalanine is as follows” phenylalanine + phenylalanine hydroxylase (deficient in PKU)-+ tyrosine, * tyrosine > & hydroxypheny! pyruvate +> homogentisate, * homogentisate ~ homogentisate oxidase (deficient in allaptonuna) > maleylacetoacciatc, + maleylacetoacetate + fumarylaceioaceate hydrate {(Gcticient in hereditary tyrosinemia) ~> fumarylacetoacetate, * fumarylacctencetate —> ‘acetoacetate 58 Note: This material is copyrighted. AUl rights reserved. Edward Goljan, M.D. 2002 + furnarate (present in TCA cycle), « alkaptonuria~ « a : as oxidase, + accumulation of black, homogentisate pigment in joints/cartilage joint disease, + urine tums black when oxidized upon exposure to light, ecicea = AR disease with a deficiency of fumarylacetoacetate hydrolase, * increase in cabbage-like odor, + citthosis with an extremely high incidence of hepatocellular carcennan renal disease (aminoaciduria), * death in the first yr of life 2 Lysosomal storage diseases: + definition- + absence of degrading enzymes in lysosomes: contain hydrolytic enzymes, + accumulation of complex substrates in lysosome: sphingolipids, slycosaminoglycans, glycogen (Pompe's disease), « most are AR discases with the exception of two diseases, which are SXR- + Fabry's disease, + Humer's disease, + biochemistry of lysosomes * lysosomal enzymes are synthesized in the rough endoplasmic reticulum— enzymes are transported to the Golgi apparamus-+ enzymes undergo post-translational modification enzymes are phosphorylated at one or more mannosyl residues to form mannose 6-phosphate, ‘which is attached to the side chains-> mannose 6-phosphate receptors on the inner surface of the Golgi apparatus membranes bind to the mannose 6-phosphate residues on the targeted lysosomal enzymes-> small transport vesicles are pinched off the Golgi membrane that contain the receptor- bound enzymes—+ the vesicles fuse and release enzymes into lysosomes located in the cytosol—> receptors rerum to the Golgi apparatus to repeat the process over again = Glycosaminoglycans (GAGs) and their disorders: + GAGs— complexes of predominantly branched, strongly negatively charged polysaccharide chains with repeating units of amino sugars (D-glucosamine or D-galactosamine) and acid sugars (L-iduronic acid or D-glucuronic acid), « chondrostin sulfate~ + most abundant GAG, * important component in cartilage, » heparan sulfste— mainly responsible for the negative charge of the glomerular basement membrane, + heparin anticoagulant, + keratan sulfate, + hyaluronic acid~ major component of synovial fluid (joint lubricant), « dermatan sulfate * ground substance in heart valves that is increased in mitral valve prolapse, * increased in pretibial myxedema, + Hurler’s disease~ AR disease with a deficiency of a-l-iduronidase, + lysosomal accumulation of dermatar/heparan sulfate, * severe mental retardation, « coarse facial features, + comeal clouding, * coronary artery disease: lipid accumulates in coronary vessels, *vacuoles in peripheral blood leukocytes, + Hunter's disease SKR disease with a deficiency of L-iduronosulfate sulfarase, + lysosomal accumulation of dermatan/ heparan sulfate, + milder disease than Hurler's © — Sphingolipids and their disorders: « sphingolipids include~ * sphingomyelin, + cerebrosides, * gangliosides, * sphingomyelin + involved in the synthesis of cell membranes in nerve tissue, * sphingosine is the backbone of sphingomyelin, * sphingosine is used to produce ceramides: sphingosme + fatty acids > ceramide , « ceramide + phosphorylcholine+ sphingomyclin, © ceramide + glucose or galactose gluco- or galactocerebrosides, « ceramide + oligosaccharides —> gangliosides, + Tay-Sachs disease~ + AR with a deficiency of hexosaminidase (ct-subunit): a 4 nucleotide insertion leads to a frameshift mutation and an abnormal hexosaminidase, * lysosomal accumulation of GM; ganglioside, * common in Ashkenazi Jews, + normal at birth—+ severe mental retardation by 6 months, + blindness with a cherry red spot in the macula: common picture on USMLE, * no h Jenomegaly, * electron microscopy exhibits whorled configurations in lysosomes that look exactly the same as lamellar bodies with surfactant in type I pmeumocytes, © Niemann-Pick- AR disease with a deficiency of sphingomyelinase, * lysosomal accumulation of sphingomyelin: bubbly appearance in macrophages/ neurons, * mental retardation, * hepatosplenomegaly, + EM exhibits zebra bodies in lysesomes: look like zebra stripes, « Gaucher lisease~ + AR disease with deficiency of glucgcercbrosidase. * lysosomal accummlation of glucocerebroside: fibrillary appearance (crumbled up newspaper) in macrophages, + adult type associated with massive hepatosplenomegaly and an increase in serum total acid phosphatase 59 ee ee eee See ee derived from macrophages, + metachromatic leakodystrophy- i aryisulfatase A and a lysosomal accumulation of sulfatide: sense Ae] ryelin, * sulfatides stain positive with metachromatic stains, + peripheral neuropathy, © ne arylsulfatase activity decreased/absent, + Krabbe diseases AR disease with a defiean ne galactosylceramidase and a lysosomal accumulation of galactocerebroside: results in the atl = of an abnormal myelin, + progressive psychomotor retardation, + multinucleated globoid cells (histiocytes) in CNS, » Fabry disease~ + SXR disease with a deficiency of a. ‘A and an lysosomal accumulation of ceramide tihexoside, * angiokeratomas on skin, = hypertension, * renal failure 7 Glycogen synthesis (glycogenesis): + glycogen synthesis (glycogenesis, occurs in the fed state) in sequence- * glucose + giucohinase > G6-PO,, + G6-PO, + phosphoglucomutase ~+ G1-PO,, © G1-PO, + UDP. glucose pyrophosphorylase-> UDP-glucose + UTP + PPi, + UDP-glucose + glycogen synthetase (insulin enhanced, yate limiting _enzyme)-> glycogen: branched chain polysaccharide of D-glucose residues with a-1=4 linkages, © glycogen synthetase produces a.-1,4 linkages between the glucose residues by adding linkages to an already existing glycogen primer, « glucosyl 4:6 transferase makes branches by transferring 5-8 glucosyl residues from the non- reducing end of the linear glycogen chain to another residue on the chain and attaching it to the chain by a @-1,6 Tinkage~ glycogen synthetase then adds glucose residues fo the new non-reducing ‘ends on the branches and to the old non-reducing ends, « liver glycogen maintains blood glucose during the fasting state until its stores are depleted— gluconeogenesis is the most imporant factor maintaining glucose in the fasting state, « muscle glycogen is used only by muscle Glycogenolysis: © glycogenolysis occurs in the fasting state- + glucagon/epinephrine activate adenylate cyclase —> increases cyclic AMP (cAMP), * cAMP activates protein kinase A: phosphorylation inactivates glycogen synthase, + activated protein kinase A activates phosphorylase kinase, » activated phosphorylase kinase activates glycogen phosphorylase A, » activated glycogen phosphorylase A (rate limiting enzyme, muscle and liver phosphorylases) cleaves 1-4 bonds up to 4 glucose residues of a branch point, * glucosyl (4:4) transferase (debrancher enzyme) removes 3 of the outer glucose residues that are left on the branch and transfers them to the non-reducing end of another chain where glycogen phosphorylase A cleaves off more glucose I-phosphates, + amylo- a-1,6 glucosidase (debrancher enzyme) cleaves off the remaining | glucose on the chain leaving behind a free glucose: ratio of glucose I-PO, to free glucose is ~10/1, * glucose 1-phosphate + phosphoglucomutase—+ glucose 6-phosphate, * glucose 6-phosphate + glucose 6 phosphatase (gluconeogenic enzyme deficient in von Gierke's}—> glucose, + small amounts of glycogen are degraded in lysosomes by a-l,4 glucosidase (acid maltase, which is deficient in Pomme's disease) Von Gierke's glycogenosis: ¢ AR disease with a deficiency of glucose 6 phosphatase, @ gluconeogenic enzyme that is primarily located in the liver and kidneys, « glucose is decreased in the fasting state (fasting hypoglycemia) and glucose 6-phosphate accumulates leading to an increased synthesis of normal glycogen primarily in the liver and kidneys (hepaterenomeraly), « glycogen excess in renal tubules interferes with lactic acid and uric acid excretion increased anion gap metabolic acidosis and increased incidence of gout, + stimulation _tests_for juconeo s¢ cannot increas ‘Pompe's glycogenosis: « AR disease with a deficiency of the lysosomal enzyme 2-14 glucosidase {acid maitase}- only glycogenosis that is a lysosomal storage disease, + accumulation of normal glycogen in lysosomes in multiple organs, * restrictive cardiomyopathy from glycogen deposition im the heart is the MC COD 1 EDS EIRETASS CORYERBOBERD BE TESS VEEN. Edward Goljan, M.D. 2002 McArdles disease (glycogenosis): * AR disease with a deficiency of mu ‘muscle glycogen cannot be degraded leading to reduced amounts of glucose for mec : early fatigue with exercise (no ATP) leading to muscle ee eee myoglobinuria, * absence of lactic acid in ice, Fesinii ees z does not contribute to blood glucose, * enzyme assay of muscle confirms hae mse wath life CUG (lysine) B. missense mutation: altered codon specifies a different amino acid leading to variable phenotypic effects e.g., UUG (lysine) + UCG (serine) © sickle cell disease/trait: point mutation occurs where adenine replaces thymidine causing valine to replace glatamic acid in the 6th position of the Belobin chain normal B chain DNA crc sickle DNA B-chain CAC AH GAG GUG Gh Val nonsense mutation; altered codon is a stop codon (UAA, UAG, UGA) causing premature termination during protein syathesis— ¢.g., UUG (lysine) + UAG (stop. codon) © P+thalassemia major: a point mutation produces a stop codon leading to termination of DNA transcription of B-globin chain 2. frameshift mutation— ‘A, caused by insertion or deletion of any number of nucleotides not divisible by 3 61 Note: This material is copyrighted. All rights reserved. Edward Goljan, M.D. 2002 B, shifts the reading frame during translation of mRNA Ceeal anil Aud seqikiee tid, REM of a eect introduction of a premature stop codon ee ee mRNA: 5'GCC AAA AGU UAU UUG Gcc3" Ala Lys Sa Tyr Leu Ala delete A 5' GCC AAA GUU AUU UGG 3’ frameshift mutation Val De Trp C, Tay Sachs: (1) 4 base insertion produces a frameshift mutation (2) _ codes for defective hexosaminidase D. cystic fibrosis: (1) 3 base deletion with loss of phenylalanine on chromosome 7: this is not 4 frameshift mutation since it is a multiple of 3 Tle fe Phe Gly Val normal DNA ATC ATC TIT GGT GIT CFDNA ATC AT- —T GGT GIT ATT Me Ie Gly Val (2) tanscribes a defective CF transmembrane regulator that disintegrates before it goes to the cell membrane = — Glycine: « smallest amino acid, inhibitory neurotransmitter in spinal cord (blocked by toxin in tetanus), synthesis of heme and collagen and bile salts and © Alanine: « alanine cycle during fasting, major substrate for gluconeogenesis (transaminated into pyruvate) & Valine: « essential AA, « branched-chain amino acid, « not degraded in liver, » utilized by muscle, ‘« increased in maple syrup urine disease @ — Leucine: + essential AA, * branched-chain amino acid, * not degraded in liver, © ketogenic, © utilized by muscle, « increased in maple syrup urine disease & — Isolencine: « essential AA, ® branched-chain amino acid, * not degraded in liver, * utilized by ‘muscle, « increased in maple syrup urine disease = Methionine: » essential AA, © polypeptide chain initiation, © methyl donor (as S.adenosyl methionine) 2 Proline: © helix breaker, + only amino acid with side chain attached to o-amino group, © hydroxylation in collagen aided by ascorbic acid, « binding site for eross-bridges in collagen = Phenylalanine: = essential AA, + increased in phenylketonuria (PKU), * aromatic side chains: increased in hepatic coma = Tryptopham: « essential AA, © serotonin, niacin, and melatonin precursor, + aromatic side chains: increased in hepatic coma 3 Cysteine: « forms disulfide bonds, « component of glutathione, an important antioxidant in RBCs (deficient in G6PD deficiency) Jingle-carbon donor: converted into glycine when carbon removed, « phosphorylated by = Threonin @ Tyrosine: « precursor of catecholamines, melanin, thyroid hormones, + phosphorylated by kinases: iemportant in second messengers, * aromatic side chains: increase in hepatic coma, « must 'be supplied in diet in phenylketonuria (PKU) essential AA, » phosphorylated by Kinases 62 Note: This material is copyrighted. All rights reserved. Edward Golfan, M.D, 2002 insufficiently synthesized by neoplastic cells,» asparaginase used for treatment of ‘7 Glutamine: + most abundant amino acid, « major carrier of ammonia, « nitrogen synthesis of purines and pyrimidines, + NH, detoxification in brain and liver, oe from skeletal muscle to other tissues in fasting state, « fuel for Kidney, gut, and cells in immune ‘% Histidine: « essential AA, « basic AA, positive charge at pH 7, « effective Physiologic buffer, « residue in hemoglobin coordinated to heme Fe™, « essential for growth in childven sven, charge at pHL7.40 7 Aspartate: ¢ acidic AA, » strong negative charge at pH 7: important for bis of albumin, + forms oxaloacetate (substrate for gluconeogenesis) ers Senin Proper ‘7 Glutamate: © acidic AA, « strong negative charge at pH 7: important for binding properties of albumin, « forms a-ketoglutarate by transaminstion ‘7 Know how to calculate number of amino acids in a gene given the nuraber of nucleotides in exons, introns, and 5' UTR 7 Genetic basis of mild B-thalassemia: Primarily a splicing defect, severe B-thalassemia is a ‘Stop codon (nonsense mutation) 3 — Effect of decreased LDL receptors on HMG CoA reductase; © normally, increased uptake of cholesterol in a cel] decreases synthesis of LDL Teveptors and reduces gene ‘transcription of HMG CoA reductase resulting in less de novo cholesterol synthesis in the cell, * decrease in LDL receptors decreases the uptake of cholesterol inthe cell, therefore, less cholesterol causes ivreaoed Tanscription of HMG CoA reductase, hence an increase in cholesterol synthesis in the ell HIGH YIELD NOTES MICROBIOLOGY © % Diphtheria toxin: « diphtheria toxin inhibits protein synthesis by ADP ribosylation of elongation factor 2, » antitoxin and erythromycin are the Rx of choice, * prevent with diphthena toxoid immunization & Pseudomonas aeruginosa toxin: exotoxin A inhibits protein synthesis using the same mechanism as diphtheria toxin (see above) & Neisseria gonorrhoeae: * gram negative diplococcus, * endotoxin is lipooligosaccharide (not lipopolysaccharide like N. meningitidis), ¢ oxidase positive contain cytochrome ¢, + chocolate agar— modified Thayer-Marten , « pili~ + attach to mucosal surfaces, * resists phagocytosis by neutrophils, + antigenic change responsible for repeated infections, « IgA protease— hydrolyzes secretory IgA to make it easier to stick to vagina and urethra, « capsule— protects against phagocytosis, + sugar fermentation~ glucose (N. meningitidis is maltose and glucose), « plasmid mediated penicillinase producing strains, » discharge in first week after sexual contact, « Rx- ceftriaxone or spectinomycin if allergic to penicillin 7 Francisella tularensis: « gram negative rod, « can survive in macrophages for prolonged periods, « antiphagocytic capsule, + vector— ticks using a wild rabbit reservoir, * MC wansmission— cleaning anvmal hides (e.g, rabbits), « requires cysteine for growth, « Rx- srepiomyein ‘© Campylobacter jejuni: « comma/S-shaped gram negative rods, « ingestion of contaminated poultry (fowl are reservorr), milk, or water, « decreased gastric acidity increases chance of infection, grows at 42°C, « MCC bacterial gastroenteritis in United States, + some strains have enterotoxin 63 ‘Note: This material is copyrighted. All rights reserved. Edward Goljan, M.D. 2002 | 4% aa 4 q4 4 q aagee similar to cholera, + bloody stools resembling ulcerative colitis, « ass. ‘syndrome antibody against organisms cross-react, Amn ae yey Calan Ba | MOC of otitis medis/sinusitis in children: Streptococcus pneumoniae MCC of otitis externa: » Pseudomonas aeruginasa~ Rx with t i hydrocortisone, also cause of malignant otitis external in i Pe eee + | Hemophilus influenzae: » gram negative coccobacillus, « (NAD) for growth ‘requires factor X (hemin, blood) and V_ “Macrophages: + multilobulated nucleus and have granules, * previous . ; macrophage of CNS and reservoir of HIV in CNS aan ioe Haier salseN NK cells: « type II hypersensitivity reactions, « graft vs host a Meremcle Heptocdes vitramieneany a st reaction, © kill tumor cells = called (Clue cell: © Gardnereila vaginalis adherent to cells, © . Cine et: «¢ cs squamous cells, « bacterial vaginosis, © Rx with ‘Sporotrichosis: « prick of thom from rose, « prick from lobster oes prick spine— packed in sphagnum moss, Patient sticks himself with a knife and develops trismas: tetanus Rx of Pneumocystis carinii pneumonia in HIV: timethoprim-suifamethoxazole Picture of budding yeast with narrow based bad: * Cryptococcus neoformans, » Rx with amphotencin Rx of choice for Chlamydia trachomatis: doxycycline: competitively block binding of RNA to the 30S ribosomal subunit involved in protein synthesis Chocolate agar: contains blood which supports growth of bacteria Child with meningitis and picture of gram negative diplococcus: + Neisseria meningitidis, « capsule prevents phagocytosis, « endotoxin (lipopolysaccharide) produces shock, » IgA protease degrades secretory IgA to allow adherence of organism to mucosa in posterior nasopharynx, * prevention— use vaccine, give rifampin to close contacts, « Rx with penicillin G Normal toicroflora of nasopharynx: ¢ anacrobic bacicria~ * Peptostreptococcus, * Fusobacterium, * Bacteroides, « viridans group streptococci, « coaguiase negative saphylococcus, « avirulent Hemophilus and Neisseria meningitidis species Normal microflora of skin: » MC is coagulase negative staphylococcus, * less commonly— * Staphylococcus aureus, * Corynehacieria, » Propionibacter (import in acne), * Clostridium perfringens (20% of healthy people), * Candida, + Malassezia ° Picture of Aspergillus: look for narrow angled septate hyphae and fruiting body Coceidloidomycosis: « think Southwest, » arthrospores in dust, « spherules with endospores in ussue Recurrent vesicles around mouth and vermilion border of lips: + Herpes simplex I, « latent in cranial sensory ganglia Exerly man with painfal vesicles in any dermatome: » Herpes zoster (shingles), « latent in sensory ganglia Review concept of complementation in defective (mutant) virus: replication of another virus provides missing fimetion required by the mutant virus Initial defense against Salmonella in blood: endotoxins released by bacteria activate the alternative complement system Self induced abortion: cervical swab and put into anacrobie medium for Clostridium perringens ‘Know bacterial reproduction types Spirocketes in synovial Nuid: Lyme's disease due to Borrelia burgdorfert Infertile woman with scarred fallopian tabes: Chlamydia rrachomaris MCC JK disease from brain instrument treated with formaldehyde: prions is transmissible agent gagguad 9 999 29 a aggre 4 TT HEE q ee ee ee ee nS Se en eS ee: Coxsackie: MCC of myocarditis (dilated cardi 2G Scitiee 1 « fomyopathy with lymphocytes in biopsy) and ‘Wound infection: gram positive cocci = S. aureus Middle aged woman with meningitis: Sireprococcus pn it Patient with pueamania bas elevated ASO Ger; uA teepene Pasiiseeiplocaea Gram positive rod resistant to heat: Bacillus anthrax MOA of Streptococcus agalactiae (group B streptococcus): inhibi Cause of an immunodeficiency with a igh IgM: defect a fooeeea ess Cyclosporine necessary in identical twin transplant—7why: MAC sites from crossover of chromosomes chivg Melons ¥ Ghee is sal some diference in Major drift with influenza: major changes in the reassortment of genome pieces indicating a need for a new vaccine; only protects against influenza A; egg based vaccine; killed virus vaccine Receptor for HIV: CD, molecule on helper T ceils (also macrophages, dendritic cells) CD common to both B and T celts: CDasis present in all leukocytes ‘CDyy: common acute lymphoblastic leukemia antigen (CALLA) Complement fixation reactions: hemolysis of test system RBCs is a negative test, while lack of hemolysis is a positive test Hants virns: « carried in rodents (deer mice), * ARDS, « hemorrhage, « renal failure, « viral RNA in lung tissue~ PCR test is best overall test Staphylococcus aureus: protein A attaches to Fe receptor of macrophages, hence blocking opsonization of bacteria Proteus mirabilis: moves with flagella; urease producer Mycoplasma pneumoniae: requires sterols Latex agglutination reactions: enibody to capsular antigens is attached to the beads Locations of cells in lymph node: B (follicles), T (paracortex), histiocytes (sinuses) Celis that attack protozoans: CD, T ceils ‘What gives bacteria their shape: peptidoglycan layer in the cell wall Tumbling motility, gram positive rods: ¢ Listeria monocytogenes, « invedes mononuclear cells, © betachemolysis in blood agar, * transplacental infection in fetus or occurs in renal transplant patients, * mainly contracted from eating unpasteurized cheese goat's milk cheese Gray membrane that bleeds when removed: diphtheria Dengue: * transmitted by mosquito- + Aedes aegypti, + same mosquito as in yellow fever, *- ~breakbone fever”, « hemorrhagic fever Infection associated with premature ruptare of the membrane: « group B streptococcus (5. agalactiae), « CAMP test UL-1 function on B lymphocytes: activates B ceils Location for Staphylococeas aureus carriers: anterior nares Rhinoviras: + commen cold occurs more often in fall and winter, * too many types to produce an effective vaccine, » person to person droplet infection and contamination of hands, « acid-labile- does not cause gastroenteritis because of this Virus responsible for a cold in spring and summer: adenovirus, Lactobacillus in vagina: responsible for the acid pH Influenza vaccine: « killed, + egg-based Bib vaccine: antibody against capsular polysaccharide In addition to the normal childhood immanizations, what addidonal immunizations are recommended in sickle cell disease and cystic fibrosis: Pneumococcus and influenzs— Pneumovax is given after 2 years of age ‘Which live vaccine can be given to a patient with AIDS: MMR- MMR is given only because the nansral infection for measles is worse than the one that potentially could happen with the attenuated virus clef at? 94 EES EOE, WAID, 2002, Live vaccines: * cannot give to immunocompromised patients: ¢, 8. organ transplant patient MMR: only one that you can give to AIDS patient, « varicella, “OPYee we MMR: only Ia, © OPV, © BCG, » smallpox. « Polysaccharide vaccines: + Pneumococcus, » Hib- meningococcal vaccine i Killed virus vaccines: » influenza, « rabies, « SALK vaccine * F Immunizations that are contraindicated in patients with anaphylactic reactions agalast egy 3s: * MMR, « influenza, « yellow fever Immunizations that are contraindicated in patients with anap neomycin: « MMR, + varicella~ neomycin is used as a preservative Verracoid lesion in lower extremity In a patient retarning from South America: * South ‘inerican Dlastomny posts yeast with a ships wheel appearance, » North American blastomycosis Newborn baby in HIV positive mother: « new 7 prevent HIV in newborn by giving mother AE ns aD 120 inthe serum (IgG antbody),» Most common cause of diarrhea in children: rotavirus E. coli: attaches to the urogenital epithelium, hence its #1 status for urinary tract infections Bruton’s agammaglobulinemia: + SXR, + defect in pre-B to B cells~ no germinal follicles in nodes or plasma cells, » prone to respiratory infections, « Rx with IV gamma globulin SCID: © combined B and T ceil deficiency, « first immunodeficiency treated with gene therapy- replacement of adenosine deaminase, ¢ no germinal follicles or plasma cells, © no T cells in parafollicular area, ‘accumulanon of dATP, which inhibits ribonucleotide reductase with subsequent decrease in deoxynucleoside triphosphate precursors for DNA which reduces the formation of B and T cell precursors, * BM transplant helpful Wiskott Aldrich: SXR, « triad sinopulmonary infections, eczema, thrombocytopenia, » B/T cell deficiency- + | IgM: poor antigenic response to bacterial polysaccharide, « normal IgG, + T Iga and IgE, + defects in CMI develop late, « increased incidence of leukemia! lymphoma, « Rx- BM mansplant Common variable immune deficiency (CVED): * no inheritance patie, » intrinsic defect in B cell maturation into antibody-producing plasma cells, « presents between 15-35 yrs of age, © recurrent sinopulmonary infections decreased Ig production, * giardiasis, malabsorption due to celiac sprue, « all Igs decreased, + Rx- IV y-globulin 2 Selective IgA deficiency: « MC hereditary immunodeficiency, * intrinsic defect in B cell differentiation into committed B cells synthesizing IgA and/or possible T cell defect that prevents B cells from synthesizing IgA, ¢ clinical * recurrent sinopulmonary infections~ lack of secretory IgA, * giardiasis, * autoimmune disease, ¢ allergies, develop anti-IgA antibodies with exposure to blood products~ danger of anaphylactic reaction when exposed to blood products with IgA, * serum and secretory IgA levels decreased Sex-linked lyruphoproliferative syadrome: * SXR, « B cell deficiency— EBV-related disease, * hypogammaglobulinemia, + malignant lymphoprotiferative disorders DiGeorge syndrome (thymic hypoplasia): * pure T cell deficiency- no inheritance pattern, * failure of the 3rd (inferior parathyroidsAthymus)/4th (superior parathyroids) pharyngeal pouches to develop, * clinical * abnormal facies, * hypoparathyroidism with hypocalcemia and tetany, * absent thymic shadow, truncus arteriosus (cyanotic congenital heart disease), + chronic candidiasis, * Pneumocystis carinii pneumonia (PCP), * graft vs host reaction (must irradiate blood to destroy donor immunocompetent lymphocytes), * Rx- + thymic grafts, * bone marrow transplants ‘Ataxia telangiectasia: « AR disease, « B/T immunodeficiency develops in 2-5 year olds, * clinical * cerebellar ataxia, * telangicctasias in cyes/slin, sinopulmonary infections * chromosome instability syndrome~ ¢ increased susceptibility for chromosomal mutations, * DNA yhylactic reactions against 66 ‘Note: This material is copyrighted. All rights reserved. Edward Goljan, M.D. 2002 29 24 4 g4aa49 q 444 a9 gare a4 aq enzyme repair defects (increased risk for lymphomasleukemias), © TgA/IgE, « increased serum a-fetoprotein + thymic hypoplasia, + low Part of a vaccine that is antigenic: polysaccharide capsule Person working with animal hides develops lung disease: Bacill e ‘ (S ypectone tie apis epeton, ta oclpane ads =e ‘woolsorter's disease Child with anemia and diarrhea: * hookworm (Necator) produces — albendazole ? ston deficiency, © Rx- Lymphocatancous nodales in a rose gardener: esporotrichosis, © Rx— + 7 was treat with potassium iodide spor ee itraconazole, * old Rx Elderly male smoker with non-prodactive congh, bacteria fails to grow on o (must be supplemented with iron and cysteine), need Dieterle silver stain to identify: « Legionelia, « fluoroquinolones or erythromycin + rifampin Macrophage activation: y-interferon secreted from helper T cell AIDS: most common acquired immmumodeficiency ILA system coded on chromosome 6 Byperacute rejection of a transplant: * ABO incompatibility or patient had anti-HLA antibodies against an HILA antigen in the graft, « type II hypersensitivity reaction HLA-A, B, C code for class I antigens: CD, cytotoxic T cells recognize these antigens ILA-D loci code for class II antigens: » CD, helper T cells, « macrophages recognize Graft vs host reaction: + NK cell mediated, + common in bone marrow and liver transplants, + clinical- « rash, «jaundice (necrosis of bile ducts) + diarthea, * danger in T cell deficient patients Receptor for EBV; * CD2; on B cells, « polyclonal stimulator causing increased syathesis of immunoglobulins— reason for hypergammaglobulinemia in AIDS, © increased cell divisions increases risk for 8:14 and development of Burkitt's lymphoma CD type for histiocytes: = CD,, © positive in patients with histiocytosis X- * Hand-Schilller Christian, + Lewerer-Siwe, * eosinophilic granuloma Destroy C. difficile in bedpan: autoclave M. tuberculosis: mycolic acid in cell wall is responsible for acid-fastnecs Blastomycosis: spores associated with beaver dams and inland water ways, broad-based-bud, « verrucoid skin lesion resembles squamous cancer Impetigo: most commonly caused by group A streptococcus Dy: antigen recognition site for T cells ‘Antigen that binds to CD, ow T cells in HIV: gp120 Risk for HIV if accidental needle stick from an BIV positive patient: + 1/330, Rx with triple therapy for 6 months and get ELISA test at repeated intervals Fever in malaria: coincides with rupture of RBCs Hematologic abnormality associated with Rx of malaria: reat ‘with primaquine and develop acute intravascular hemolysis in G6PD deficiency Pathogen community acquired infection rather than a nosocomial infection: Mycoplasma pneumoniae ‘Single most important infections cause of death in the world: tuberculosis ‘Asthma, massive hemoptysis and extermal otitis: Aspergillus fumigarus: narrow angle, septate, corona Cavitary lesion on a chest x-ray: + primary squamous carcinoma of the lung, + Histoplasmosis, « Jung abscess, * reactivation TB is cavitary MMycobocteriom tuberculosis and sarcoidosis are et a its, the former the MC infectious cause and the later the MC non-infectious ioe discoloration of the sputum in a febrile 4 year: ‘old child with cystic fibrosis is MOST LIKELY due to: Pseudomonas aeruginasa- pyocyania oT Note: This material is copyrighted. All rights reserved. Edward Goljan, M.D. 2002 aad 44 Pathogens that are branched with a standard Gram stain: « i é coud fan, | siieiacTobey Actinomyee a sirie anserobe, © ritepedde eee ioe) in jaw, thorax, or abdomen ems 65 year old cave explorer for Indian artifacts in the Sonoran de or ja Bir yuna eioon-predactieenegh aed lala oa amas ee nee i. aspect of his lower left leg (erythema nodosum). A solitary coin lesion with an egg shell ike cavity is noted in the upper portion of his left lower lobe on a chest x-ray: coccidioidomycos ‘An asymptomatic, afebrile 48 year old Black man, who has lived all of his life in Ohio, ts noted to have multiple calcifications throughout both lung fields and in the spleen: histoplasmosis 7 Scotochromogen associated with painless cervical adenopathy in children: Mycobacterium serojulaceum ‘Most common cause of disseminated TB in AIDS: Mycobacterium avium-intracellulare Most common cause of 2 swimming pool granuloma: Mycobacterium marinum ‘Most common cause of intestinal tuberculosis in the United States: Mycobacterium tuberculosis swallow infected sputum and organisms taken up by macrophages in Peyer's patches Photochromogen that produces pulmonary tubercalasis: Mycobactertum kansasii Rapidly growing TB associated with infection in immunocompromised hosts and in prosthetic devises: Mycobacterium fornuitum Pathogen that commonly produces lung abscesses, common secondary invader in the lung in patients with rubeola or influenza, produces tension pneumatocysts leading to tension pneemothorax in cystic fibrosis patients: Staphylococcus aureus Pathogen that is comtracted when the newborn passes through the birth canal. It produces 2 pneumonia characterized an abrupt onset of tachypnes, wheezing, hyperacration, eosinophilia and a conspicuous lack of fever. It is often associated with a conjunctivitis: Chlamydia rackomartis Systemic pathogen that is often associated with the presence of indwelling venous/arterial catheters and immunodeficiency states. It produces a pneumonia characterized by diffase nodular infiltrates and evidence of vessel invasion: Candida albicans Childhood pathogen that may produce a pnenmonia associated with Warthin-Finkeldey multinucleated giant cells: rubeola Water loving pathogen that is most commonly seen in men over 40 years old who.are smokers. It produces a confluent bronchopneamonia, with fever, non-productive cough, hemoptysis and other systemic signs and symptoms. It commonly produces hyponatremia related to development of interstitial nephritis leading to byporeninemic hypoaldosterouist. ‘The pathogen is best visualized with direct immunoflaorescence or a silver stain: Legionella pneumophila Respiratory patbogen with a significant mortality in those over SS years of age and who have underlying renal, cardiac or lang problems. It produces a severe, exudative pneumonia with = propensity for secondary bacterial invasion. There is an association with Reye's syndrome in ‘children that take aspirin: influenza Respiratory pathogen that is transmitted withoat a vector, unlike other pathogens in its family group. Its primarily transmitted by inhalation by individuals who have am association wrth the birthing process in sheep, cows and goats or iu those who work in the milk industry. Itis particalarly common in workers who shovel feces in sheep: Coxiella burnetii Respiratory pathogen that is transmitted by direct hand to hand transfer of infected materia and by respiratory droplet infection. Its main reservoir is school children. Development of x vaccine is highly unlikely: rhinovirus Respiratory pathogen transmitted by droplet infection. It accounts for approximately 10% of community zequired atypical pueumonias and a smaller percentage of cases ‘of bronchitis. Cold agglutinins are not associated with this pathogen, There may be 20 association with 6 Note: This materia is copyrighted, All rights reserved. Edward Galjen, M.D, 2002 coronary artery disease. It responds well to doxycyel Chlamydia pneumoniae (TWAR) ee Strict anacrobe that can produce an empyema that drains through a sinus t skin surface. Vellow flecks of material ta the drainage fund reed et tthe demonstrate its characteristic morphology: Actinomyces arauin rained fa: 7 Pathogen that is commonly contracted in military stations and in erow produces an interstitial pneumonia and is often associated with ceythons ayes bullous myringitis. responds well (o erythromycia: Mycoplasma puccremne me ad 7 Respiratory pathogen that is an example of a zoonosis. It ls associated with an interstitial Pueumonia. Its incidence has declined by putting tetracycline ia savant toon : psiteact, ‘rare Predominantly » respiratory pathogen, this strict aerobe is most commonly seen i ‘ taki y seen in patients with defects in cellar immunity, partcalarly in the setting of heart traseplaceaee Brdaces suicrotbscesses in the lungs, often with granuloma formation. A characters feature. aside from its umusual Gram stain morphology, is that itis partalis Nocardia asteroides aa = Most common cause of death in patients with cystic fibrass: Pseudomonas aeruginasa other members of its family: J Most common cause of pneumonia and bronchiolitis in infants: respiratory syncetal sirus ‘7 Most common cause of croup with inspiratory stridor. A lateral neck x-ray reveals « “steeple sign”: © parainfluenza, « tracheal obstruction ‘* Most common cause of inspiratory strider in a child whose lateral neck x-ray reveals the “ehumbprint” sign. Its incidence has decreased owing to the use of a vaccine: Hemophilus influenzae * Most common cause of bronchopeumonia and lobar pueumonia in the general populato, Streptococcus pneumoniae > An executive in an office in New York City has a window air conditioner that is a favorite roost for pigeons. She and her secretary both develop lung disease: Cryptococcus neoformans Narrow based bud Am BIV positive patient with fever, night sweats, cough, dyspnea with exertion, a CD, T helper count of 350 cells WL, and cavitary apical lung disease: » Mycobacterium tuberculosis, « note that MAL only comes when the helper T cell count is < 100 cells/iL 5 & — Most common fungal infection in an HIV positive patient: candidiasis + Most common cause of recurrent puenmonia in a patieat with AIDS: Streptococcus prewmoniae Inspiratory stridor is commonly associated with: « parainfluenza virus infection (croup), © acute epighomitis 7 Chlamydia trachomatis and the respiratory syncytial virus are BOTH commonly associated with: interstitial type of pneumonia © — Klebsiella pneumoniae: * upper lobe cavitation, + thick mucous and fat gram negative rods, « association with an alcoholi Ima. 30 year old man who lives in Tennessee, you would expect a calcified solitary coin lesion in the lung to represent: an old granuloma (histoplasmosis) 7 A bridge paiater in Brooklyn, New York develops palmonary infiltrate, Which of the following pathogens are on your differential ist: + Histoplasma capsulata (starlings). © Cryptococcus neoformans (pigeons) = 26 year old male, who presented with a sudden onset of 103° F temperature, dyspnea, cough productive of rusty-colored sputum, and pleuritic chest pain in the right upper lung. A chest x-ray reveals a right apper lobe consolidation. A gram stain of sputum is pending: + Streptococcus pneumoniae pneumonia, Rx with Penicillin G co Note: This material is copyrighted. All rights reserved, Edward Goljan, M.D, 200 © systemic fungus with broad-based buds presents with a skin disorder carcinoma as well as lung disease: Blasiomyces dermatitidis © — Systemic fungus that most simulates primary and reactivation TB phagocytosed by alveolar macrophages: Histoplasma capsulatum = Pulmonary pathogens that are vessel invaders and bave the capacity to produce palm raed * Candida albicans, » Pseudomonas aeruginosa, « Aspergillus fumigates « Mucee = Patients who are being treated for tubercutosis may develop si : cn canosral anata padoeempaalon crete, By me Read sat the mitochondria of RBCs, « leads toa sideroblastic anemia asses & Streptococcus pneumoniae: MCC of- © meningitis in patients > 18 ia peritonitis in children with ascites, otitis media, eat 8s = © — 28 year old man with ALDS who presents with chronic, recurrent, prof diarrhea, Au Eotive-Test (string tert) veveals ourjets that ure ee cnr Cryptosporidium parvum © — Afebrile 22 year old mam and several other memabers of his family developed severe vomiting without diarrhea ~1-6 hours after esting potato salad at a picnic. They all recovered uneventfully 12-24 hours later: S. aureus with preformed toxin © — 23 year old man developed explosive, watery diarrhea with blood, leukocytes, and mucus ~3 days after eating chicken that was improperly cooked. Comma-shaped organisms are noted in the feeal smear of stool along with RCs and leukocytes: Campylobacter pylori & Febrile 10 year old child presents with severe right lower quadrant pain that is interpreted by the attending physician as acute appendicitis. At laparotomy, the surgeon motes that the appendix is normal. Mesenteric lymph nodes are markedly enlarged and have focal areas of microabscess formation on cat section: Yersinia enterocolitica © Afebrile 28 year old medical stadent develops vomiting and diarrhea ~4 hours after eating rewarmed fried rice from a Mexican restaurant. Gram positive rods are present in his stool. He recovers uneventfully in 12 hours: Bacillus cereus~ preformed toxin ‘3 25 year old medical student during Spring break in Tijuana, Mexico develops fever, vomiting, abdominal cramps, and watery diarrhea ~14 hours after eating a few tacos purchased from a street vendor. He recovers aneventfully in 48 bours: enterotoxigenic £. coli— secretory diarthea. 2 Afebrile 30 year old man develops profase watery diarrhea, with abdominal cramps ~18 hours after eating 2 dozen raw oysters at a cafe along the Louisiana coast. He is hypotensive, volume depleted, and bas 2 bypokalemic normal anion gap metabolic acidosis: Vibrio cholerae + Ona trip to the Far East, a man develops a high fever associated with bradycardia, absolute neutropenia, and splenomegaly. A blood culture i positive for a gram negative organism: Salmonella typhi > 28 yr old man presents with diarrhea with mucus and blood. A stool for fecal lenkocytes reveals blood and ncutrophils. Pseadomembranes are noted in 2 colonoscopy: could be~ # Shigella sonnei, « Campylobacter jejuni x Patient with AIDS develops diarrhea with steatorrbes. A biopsy reveals macrophages with foamy cytoplasin.: Mycobacterium avium-intracellulare with Whipples-like syndrome 2 — Odynopbagia in a HIV positive 28 year old man with white plaque-like material on his tongue and buccal mucosa that scrapes off and leaves a bloody base: Candida albicans 3 Elevated serum amylase associated with meaingoencephalitis: paramyxovirus infection (mumps) 2 Gastrointestinal lymphomas arising from mucosa associated lymphoid tissue (MALT) in the stomach: Helicobacter pylori me & Most common invasive enterocolitis that commonly simulates ulcerative colitis and pseadomembranons colitis: Compylobacter jejuni simalating squamous and bas yeast forms 70 Note: This material is copyrighted. All rights reserved. Edward Goljan, M.D. 2002 TUTTSATIee F 4449 249 aa Most common cause of diarrhea in children during the wi for aid in its diagnosis: rotavirus Ee iter mae thats am BUSA tase Coramon cause of diarrhea in AIDS and most common i pancreatitis in ADDS: cytomegalovirus mise Eibili trac Bearers ‘Most common cause of gastroenteritis in adults: Norwalk virus Most common cause of a secretory diarrhea when outside the United States: Enterotoxigenic E. coli Most common cause of a microangiopathic anemia, thrombocytopeni: , children: Enterohemorshagic E. coli- serotype O1S7:87 eosr{spenn- Sadun uae Elderly patient develops diarrhea a week after being treated for pneamonia while in the hospital. A flexible sigmoidoscopy reveals a gray-yellow exudate in the rectosigmoid: Clostridium difficile Rx. with metronidazole ‘Common causes of dysentery and hemolytic uremic syndrome in children: Shigella sonnei and Enterohemorrhagic coli Most common cause of noa-typhoid enteric fever and cause of osteomyelitis in patients with sickle cell disease: Salmonella paratyphi ‘Transmitted by a human carrier and produces a disease associated with fever, sepsis, vasculitis, diarrhea, hepatosplenomegaly, and gallbladder disease. It is not the most common cause of osteomyelitis in children with sickle cell disease: Salmonella yphi MC organism contaminating blood transfusions and may be associated with triggering ankylosing spondylitis: Yersinia enterocolitica Produces obstruction of the terminal Hleam: © Mycobacterium tuberculosis, + swallowed organisms from a primary in the lungs Invasive helminth that is often disseminated in AIDS patients and a common cause of antoinfection and superinfection: Strongyloides stercoralis ‘Type (s) of hepatitis with no protective antibodies: Hepatitis C and D ‘Type (s) of hepatitis with no chronic state: Hepatitis A and E Most common type of hepatitis found in day care centers: Hepatitis A ‘Most common type (s) of hepatitis leading to hepatacellalar carcinoma: Hepatitis B and C ‘Most common hepatitis associated with polyarteritis nodosa: HBV Most common hepatitis in traveler's to places outside the United States: Hepatitis A Most common hepatitis in jails and corrective institutions: Hepatitis A Most common sexually transmitted types of hepatitis: Hepatitis A, B,C Most common cause of posttransfasion hepatitis and chronic hepatitis: Hepatitis C Most common type (5) of hepatitis prevented by. immmnization with hepatitis B vaccine: © Hepatitis B, D, « also prevents hepatocellular carcinoma from HBV cirthosis Most common type (s) of hepatitis transmitted parenterally: Hepatitis B, C,D Most common type (s) of hepatitis with protective antibodies: Hepatitis A, B, and E Most common hepatitis producing fulminant hepatitis in a patient with 2 pre-existing hepatitis: Hepatitis D: requires HBsAg to infect hepatocytes Most common type (s) of hepatitis transmitted by the fecal-oral route: Hepatitis A and E Most common type of hepatitis in homosexuals: Hepatitis A: unprotected anal intercourse Most common type (s) of hepatitis that may lead to chronic hepatitis: Hepantis B,C, D Most common type of hepatitis associated with cryoglobalins and membranoproliferative glomerulonephritis: Hepatitis C Most common type of hepatitis transmitted by accidental needle stick: Hepantis B ‘Most common cause of spontaneous bacterial peritonitis in adults with cirrhosis and ascites: « E. coli, + Streptococcus pneumoniae is MCC in ascites associated with nephrotic syndrome in children, n Note: This material is copyrighted. All rights reserved. Edward Goljan, M.D, 2002 Pathogen associated with inguinal lymph nodes containin; vali and localized Iymphedema and rectal strictures as a auae pate venereum- only STD without ulcers peg ana Si year old woman complains of a vagioal discharge that bas a ‘fishy odor’. Iti parti mlipeatie cneng atten, Tieuisokarge ava gH ofS. Kaxmdeacioa ote i iad bacteria adhering to epithelial cells: * Gardnereila vaginalis~ clue cells, « i the patient with, nee ie sells, # not an STD, + oniy Rx = 3S year old diabetic presents with a vaginal discharge that is thick and has cortage cheese: Candida albicans Rx with fluconazole pkenpgenrsiicnt © 32 year old female presents with intense vulvar pruritus. She has a purulent malodorous frothy green vaginal discharge. The discharge can be wiped from the wall of the vagina and leaves patchy vaginal erythema (strawberry vagina), The pH of the discharge is 5.6: Trichomonas vaginalis~ Rx both partners with metronidazole = Most common cause of neonatal meningitis. Causes chorioamnionitis due to an ascending infection from the vagina and cervix. Risk of infection of neonate is greater when there is premature rupture of the membranes prior to delivery: group B Streptococci (Streptococcus agalactiae) 4 23 year old woman on the third day of menses presents with fever, severe lower abdominal pain and adnexal tenderness with movement of the cervix. History reveals sexual intercourse 344 days prior to menses: + Neisseria gonorrhea- the short incubation period is most consistent with GC rather than Chlamydia, © she has PID and should be treated with ceftriaxone and doxyeyeline, the latter to cover possible Chlamydia trachomatis as coinfection 2 34 year old patient bas had an [UD (intrauterine device) in place for 9 mouths as a method of birth control and now complains of vaginal discharge. At the request of the patient, the IUD is removed. Yellow flecks of material are attached to the IUD which ou Gram stain reveal a filamentous gram positive bacteria: Actinomyces & 46 year old woman preseuts with dysuria, increased frequency, and a soucopurulent vaginal discharge. Vaginal smears of the exudate and examination of the urinary sediment are negative for organisms but show many leukocytes. Her last sexual exposure was 10 days ago: Chlamvalie trachomatis ‘@ 33 year old female from Equador bas recently been diagnosed with chronic endowetritis. A jcroscopic examination of the endometrial tissue reveals poorly formed granulomas and ax infiltrate of plasma cells: Mycobacterium tuberculosis = 32 yrold woman has a painfal alcer on the left labia taajora and Ipsilateral painful inguinal iymph nodes. A gram stain from the base of the ulcer reveals gram negative rods in a “school of fish” orientation: Hemophilus ducreyi @ 25 yr old woman has a painless ulcer on the left labia majors and ipsilateral painful ingnioal lymphadenopathy: Treponema pallidum primary syphilis. = 23 yr old woman develops fever, palnfal lymphadenopathy, and painful vesicles on ber ‘external genitalia that later ulcerate. A ‘Tzanck prep from the base of ome of the ulcers is positive: Herpesvirus type 2— note how primary infection is systemic Patient from New Guinea presents with a painfal, serpiginons type of raised sore ow the labia majora. There is not tymphadenopathy. A biopsy is caken aad reveals coacrophages wich intracellular organises: Calymmatobacterium granulomatis: the patient has granuloma inguinale and the intracellular bodies are called Donovan bodies perianal area. Her RPR is negative: human papilloma virus- the patient has condyloms acuminata. or venereal warts a 30-year-old ornithology graduate student presents with altered mental stamas, stupor, snd headache. There is ao nuchal rigidity. He has been studying marsh birds over the last month n ‘Note: This material is copyrighted. All rights reserved. Edward Goljan, M.D. 2002 4 q S889 4 in a mosquito-infested swamp area in the Midwest. A gram stain negative: « patient has an arborvirus type of encephalitis OE SE ETD 18-year-old Navy recruit presents with fever, headache, and a posit ig’ js Neisseria meningitidis based on the crowded conditions " 32-year-old man renai transplant patient who is immunosuppr itive Indi preparation: cryptococcal meningitis sic Bilateral ophthalmia neonatoram during the first week: « N. gonorrhoeae, « transmitted on the ‘way through the cervix Corneal transplant, brain instramentation: can transmit the prions of Creutzfeldt-Jakob disease Diabetic ketoacidosis: « mucormycosis in frontal lobes, * ketoacidosis accelerates the growth of the fungus MCC of meningitis after 18 yrs old: Streptococcus pneumoniae Rubeola: cause of subacute sclerosing panencephalitis— slow virus disease Papovavirus infection: viral agent of progressive multifocal leukodystrophy, a slow virus disease “Bubbles and holes" spongiform change in a brain: « describes C-I disease, » prions yInterferon: » produced by CH, T helper cells and NK cells, © functions~ + activates macrophages to Kill microbial pathogens, * antiviral activity, * induces class I and II antigens, « increases production of IL-2 and IL-12 by CD,-T helper cells Interleukius: « IL-I- + produced by macrophages, * fever of inflammation, + acute phase reactant synthesis of proteins in liver, + osteoclast activator, * stimulates B cell production and antibody production, © [L-2~ + produced by CD, T helper cells, * * primarily a T cell growth factor, * promotes B celi/NK cell proliferation, « IL-3~ * produced by T cells and thymic epithelial cells, « stimulates pluripotential stem cell marrow, * increases hematopoiesis, * IL-4 * produced by activated T cells, * mainly promotes growth of B cells, + switch of IgM synthesis in B cells 10 [gE synthesis in type 1 hypersensitivity reactions, « IL-5— + produced by T cells and mast cells, promotes growth of cosinophils, * promotes IgE synthesis, + IL-6- + produced by T cells, macrophages, endothelial cells, fibroblasts, epithelial cells, » primarily stimulates synthesis of acute phase reactants in the liver in acute inflammation, « [L-12— = produced by macrophages, * promotes growth of CDs T cells, * promotes differentiation of CD. T helper cells into Til and Ty2 lasses, + promotes production of y-interferon, * enhances NK activity “ Granulocyte colony stimalating factor (G-CSF): + produced by fibroblasts, + stimulates ‘neutrophil development in the bone marrow Granulocyte/macrophage colony stimulating factor (GM-CSF): * produced by macrophages and T cells, « stimulates neutrophil and monocyte development in the bone marrow B cells: « 10-20% of total lymphocyte count, ¢ markers- * intracytoplasmic 1 heavy chains: pre-B cell, « surface ys and 8 heavy chains: manire B cell and antigen recognition site, » fimction~ antibody synthesis, « surface receptors- * IgG Fe receptor, * CDzi for EBV, testing- * B cell count: flow cytomeny, * immunoglobulin concentration: order of decreasing concentration IgG, 1gA. [eM, [gD, and IgE, « detect isohemagelutinins, + mitogen stimulation: pokeweed T cells: « 60-70% of total lymphocyte count, « markers * monoclonal antibody marker studies for cluster designation (CD) types, * immature T cells have nuclear enzyme terminal deoxynucleotidy! transferase (141) on their surface, « functions + type IV hypersensitivity, * cytokines regulate B cells, * defense against intracellular pathogens (¢.g., TB, protozoa), « testing * mitogen assays: functioning T cells are specifically activated by phytchemagglutinin and concanavalin A, * skin tests to evaluate cellular immunity: ¢ Candida is the main antigen used, ¢ absence of an immune indicates anergy or a lack of cellular immunity (e.g, AIDS) B Note: This material is copyrighted. All rights reserved. Edward Goljan, M.D. 2002 = AIDS epidemiology: + gp 120 viral envelope protein of virus attac! cells and other cells» monocytes/macrophages/dendritic caldera cae st Tiger + astrocytes, « p24 core protein surounds viral genomic RNA~ = only increased dane nony infection and when the patient develops AIDS, * 2 separate peaks, © e B ae the virus- usually direct HIV cytotoxicity, * reverse as oneal proviral double stranded DNA~ integrated into host cell's DNA with virally encoded inte enzyme, ¢ after transcription, HIV messenger RNA is translated into various proteins— Sem encodes gp120/gp4l, * pol encodes reverse transcriptase/integrase, + gag encodes p24 core ant * vial core consists of genomic RNA surrounded by an inner membrane composed of p24 antigens assembled near the host cell's plasma membrane, * budding of the progeny virion ie eat cell membrane is where the viral core acquires the extemal envelope to become a mature HIV © Mode of transmission in the United States in descending order: « receptive anal intercourse between men, * vaginal intercourse male to female: infected semen has more surface area to infect, ‘* female to male: less surface area in male urethra to infect = Positive enzyme immanoabsorbent assay (EIA) test for HIV in a newborn: © due to transplacental transmission of the IgG antibody from the infected mother, » document HIV infection in newborn by detection of HIV RNA by PCR (best test) and p24 antigen capture assay = AIDS testing with enzyme immunoabsorbent assay (EIA): « initial screening test, « detects anti- gp120 antibody- * sensitivity 99.5-99.8%, * poor specificity due to low prevalence of HIV positivity in the general population 7 AIDS testing with western blot: « confirmatory test for indeterminate or positive ELA, » positive western blot- * presence of p24 and gp4l antibodies and either gp120 or gp160 antibodies, * combined positive predictive value of a positive ElA/westem blot is 99.5% & AIDS tests for movitoring immune status: ¢ CD, T helper cell count, * HIV RNA by PCR- best ov to viral burden & — Non-AIDS defining infections: « oral thrush, * oral hairy leukoplakia (EBV glossitis), © shingles (E. zoster), « molluscum contagiosum (poxvinus) © Diagnosis of AIDS: HIV positive plus- * CD, T helper cell count: <200 celis/pL, * specific malignancies: e.g., Kaposi's sarcoma, + specific infections: e.g., P. carinii pneumonia (MC AIDS- defining disease) . @ AIDS miscellaneous infections: + bacillary angiomatosis- * due to Bartonella henselae: identify with silver stains, » simulates Kaposi's sarcoma, e recurrent bacterial pneumonia— * Streptococcus pneumoniae, * infections encountered with CD, T hélper count 100-200 cells/iL, * MC COD in ‘AIDS, « infections encountered with CD, T helper count <100 cells/pL~ disseminated MAI: usually <75 cells/uL, * Candida esophagitis, * CMV retinitis/esophagitis, * Toxoplasma encephalitis, * Cryptosporidiosis: diarthea, cryptococcal meningitis © Testing of complement system: « classical pathway- low C4 or C2 if activated, « alternative pathway- low factor B if activated, ¢ integrity of both pathways~ low C3, if either system is activated, © activation increases the concentration of split fragments e.g., C3a, C5a, C3b, fictional assessment of the complement system~ total hemolytic complement assay (CHa) & Major Histocompatibility Complex (MHC): « mature RBCs lack class I antigens, » individuals inherit ! FILA haplotype from each parent in codominant fashion: both haplotypes are expressed = Transplantation success requirements: « ABO blood group compatibility most important test, * absence of preformed anti-HLA cytotoxic antibodies in the recipient’s serum, « close matches for HLA-A, B and D loci between recipient and donor = — Lymphocyte crossmateh: screens for recipient anti-HLA antibodies against donor lymphocytes 4 Note: This material is copyrighted. All rights reserved. Edward Golan, M.D. 2002 94 Lymphocyte microcytotoxicity test: identifies HLA-A ji recipient Shae Hpac cg RG edge eee antigen preg Mixed lymphocyte reaction: * used for class I anti , Iymphocytes from the recipient and previously ime Gane) pe Moo : eet together with tritiated thymidine to detect the degree of compatibility between their D i aemeed radon ines incompatibility, « recipient's lymphocytes are irradi les) and functional donor lymphocytes are reacted against , Pee radiated a Get) against the host's HLA-D loci to check for a graft ‘Transplant donors: « siblings are best source— chance of a sibling having another sibli 1. a2 haplotype match is 25%, 50%, and 25%, eonstiecly canes pe ees aplotype match — Graft types: « autograft- * transfer of tissue from self to self, + best survival, » syngeneic graft (isograft)- graft between identical twins, + allograft- graft between unrelated individuals, + xenograft» transplant of tissue from one species to another, + e.g., pig beart transplant ‘Corneal transplants: best overall allograft survival rate ILA haplotypes and disease: + familial predisposition to disease— + weak penetrance, + disease is not invariable, * usually requires exposure to an environmental factor (¢.g., virus), © HLA-A3* hemochromatosis, « HLA-B8/DR3~ celiac disease, « HLA-B27~ ankylosing spondylitis, » HLA DR2- multiple sclerosis, » HLA-DR3/DR4- type I insulin-dependent diabetes mellitus, © HLA- DR4— rheumatoid arthritis Examples of type I hypersensitivity reactions: » atopy- familial predisposition (oultifactorial inheritance) to develop an allergic reaction, + examples» eczema (face, flexor/extensor surfaces), = hives, * seasonal conjunctivitis, * seasonal rhinitis, * asthma, * hypersensitivity to beeiwasp/homet stings: Rx of anaphylactic reactions~ sc. administration of aqueous epinephrine 1:1000 dilution: Examples of type 1 cytotoxic hypersensitivity reactions: involves antibody reactions with or without complement, « examples~ Goodpasture’s syndrome: anti-giomerular/pulmonary capillary basement membrane antibodies, ¢.warm autoimmune hemolytic anemias and cytopenias, RWABO hemolytic disease of newborn, * cells/helminths coated by specific IgG/IgE antibodies, respectively (without complement) are destroyed by cells (e.g., NK cells/eosinophils, respectively) ‘with low affinity IgG/IgE Fe receptors, « myasthenia gravis: anti-acetylcholine receptor antibodies,” « Grave's disease: [gG thyroid-simulating Ig directed against TSH receptor Examples of type If immunocompler (IC) hypersensitivity reactions: « circulating ICs (antigen * IgG/igM) deposit in target tissue (e.g, glomerilus, small vessel)-> activate complement system-> chemotactic agents recruit neutrophils/macrophages that damage the tissue, + pathogenesis of localized IC reactions (Arthus reactions)- first antigen exposure results in antibody production-> second exposure to antigen deposited in tissue leads antigen-antibody ICs-> complement systcm activation—> neutrophil/macrophage damage of tissue, + examples = serum sickness’ RX of rattlesnake envenomations with use of: horse serum antitoxins, + SLE glomerulonephritis (GN): anti-DNA + DNA ICs, * poststreptococcal GN: anti-bacterial antigen entibodies + bacterial antigen ICs, * Henoch-Schénlcin purpura: anti IgA antibodies agains IgA. * polyareeritis nodosa: HBsAg + anti-HBs {Cs, * sheumatoid arthritis: IgM antibodies against IgG (rheumatoid factor), * clinical example of an Arthus reaction— Farmer's lung: antigen is thermophilic actinomycetes Examples of type IV T celkmediated hypersensitivity reactions: + antibody-independent cellular immune reactions involving CD, helper T cells (DRH reactions) and CD, cytotoxic T cells, ° pes OE DRH reactions + allergic contact dermatitis: poison ivy, nickel, + skin tests: tubereulit ‘sensitivity, patch test in contact dermatitis, * granulomas, ° pathogenesis of cytotoxic T cell 15 Note: This material is copyrighted. All rights reserved. Edward Goljan, M.D. 2002 reactions ¢ cytotoxic T cells normally interact with class 1 of class { antigens on target cells activates cytotoxic T cells ieee pehtnturanr eo + examples: neoplastic/virally infected cell, foreign antigens in a transplant cell destroy the cell, Parasitology terms: « definitive host~ host that harbors the adult, or sexual stage of the parasite, « intermediate host- host that harbors the larval, or asexual stage of the parasite © ‘ ee gee ee ee ee cestodes: tapeworms, * trematodes: flukes oraa, Entameba histolytica (amebiasis): « key words * erythrophagocytosis, + liver abscess, # Rx- metronidazole "is, * bloody dysentery, + Naegieria fowleri (primary amebic meningoencephalitis): « ait water lakes, « meningoencephalitis, « Rx— arphaiert a & arpa @ ket ‘Acanthamoeba (keratitis): + key word— soft lens wearers that i clean in proper solutions, » Rx~ propamidine sepia bt Ove e eS aR Cryptosporidium parvum (sporezoan): * key words * acid-fast oocysts, * MC eR ee gaat fea nuks lang mw anisqpeavuiencg rpaiuveEeas ae Rx- paromomycin Toxoplasma gondii (sporozoan): * key words- + MCC of space occupying lesion of brain in AIDS, « pregnant women avoid cats and eating undercooked meat, + Rx- pyrimethamine Babesia species (sporozoan): + key words~ * carried in [odes dammini (same tick as Borrelia burgdorferi), + intraerythrocytic parasite with hemolysic anemia, « Rx clindamycin + quinine Ptasmodia species (sporozoans): © types~ * P. vivar: MC, + P. falciparum: most deadly, + P. malariae, + P. ovale, * female Anopheles mosquito is the vector for malaria~ * sexual cycle (schizogony) develops in the mosquito, * sexual cycle (sporogony) develops in humans, + reinfection of hepatocytes by merozoites as in P, vivax and P. ovale infestations is responsible for relapses, « patients with sickle cell traivdisease, glucose-6-phosphate dehydrogenase deficiency, or thalassemia are resistant to P. falciparum infection: multiple reasons including~ * short life-span of the RBCs, * increased fragility of RBCs, + increased levels of HgF: parasites cannot feed on this type of Heb, © most blacks are Duffy blood group (Fy) antigen negative, which renders them resistant to P. vivax infections the organism requires the antigen as a receptor before it parasitizes the RBC, « pathogenesis of anemia in malaria relates to both intravascular end extravascular hemolysis of RBCs, the latter by removal of infected cells by splenic macrophages, + clinical picture for the "benign forms of malaria includes * periodic paroxysms of shaking chil correlates with intravascular rupture of RBC, * high fever with a specific pattern in all type except. P. falciparum, « quotidian (P. falciparum: daily spikes (paroxysms) of fever that persist for a few days and occasionally (no specific pattem) break every couple of days. tertian (P. vivar and ovale): paroxysms of fever every 48 hrs (alternate days), ¢ quartan (P. malariae): paroxysms of fever every 72 hrs, « splenomegaly is a consistent fearure in all malarias: spleen may’ spontaneously rupmure, « P, malariae infestations may lead to membranous glomerulonephritis, « 2 falciparum mmalaria- * multi-infestation of RBCs by ring forms is commen, * only (ype that only has ring forms and gametocytes in thc PB: common USMLE picture, * massive intravascular hemolysis results in hemoglobinuria: Higb tums 'a black color in the presence of an acid pH, hence the term Ceaser fever, « chloroquine» MC prophylaxis, + safe dunng pregnancy. * Kills Dood schizonts and is gametocidal to all malaria species except falciparum. resistant stains of P.- Jaleiponm, vse qefloguing, + weammeni- * P. viva/ovale: chloroqaine + Pripaahe (echisonueide in liver and gornetocidal to falciparum), * P.faleiparum: IV quinidine giveondt for people who cannot take drugs orally or quinine sulfate + doxycycline if they can take drugs orally Balantidium coll (sporozoan): « key words— * large cysts, ¢ colonic: ulcers with bloody diarrhea, © Rx- tetracycline 16 ote: This material is copyrighted. All rights reserved. Edward Geljan, M.D. 2002 = Giardia lamablia (f_agellate): + key words- + MC protozoal cause of diarthea'malabsorption in United States, * day care centers, * mountain springs, * positive Test, « Rx— metronidazole Eee 3 Trichomonas vaginalis (flagellate): « key words- * STD, * Rx both parmers, * umbling motility, + Rx- metronidazole i = Blemoflagellates: + human infecnons~ * Jeishmanial forms sre within macrophages, + mypanasomes are extracellular and careulate in blood, « leishmanasis- only the intracellular (leishmanial) forms are present in humans, + African typanosomuasis~ only the extracelhular (rypancsomal) forms are present in humans, * American typanosomuasis, or Chagas disease~ boch Jeishmanal (causes tissue destruction) and trypanosomal forms are present in humans = African trypanosomiasis (sleeping sickness): + caused by Trypanosoma brucei gambiense or Trypanasoma brucei rhodesiense, * bite of an infected tsetse fly, + clmeal- + encephalins, * increased IgM, sommolence: release of sleep mediators by the organisms, + wypanosomes are capable of antigen variation, « Rx~ suramin for early infection (non-CNS disease) and melarsopra later stages (CNS infection) = American trypanosomiasis (Chagas disease): + caused by Typancsoma cna, » bite of the reduviid bug (Triatoma, o: kissing bug), + clinical = major cause of progressive heart failure leading to death, = acquired achalasia/ Hirschsprimg's disease, « Rx~ nsfurnmox 3 Leishmaniasis: * visceral leishmaniasis~ Leishmania donovani complex, © cutaneous Ieishmaniasis~ Leishmania tropica complex, « leishmania introduced by the bite of an infected sandily (Phieboromus), © visceral leishmaniasis, or kala-azar~ + massive hepatasplenomegaly and anemia, © Rx: antimony sibogluconate, * cutaneous leishmaniasis + involves skin alone: ulcers (problem in the Gulf war), + Rx: antimony stibogluconate Nematodes: © those with larval phases in the limgs leading to pucumonitishemoprysis— * Ascaris kumbricoides, « Necator americanus, » Strongyloides stercoralis, + Ssrongyloides stercoralis has the only five living larval form that can survive in namre = Enterobius vermicalaris (pinworm, nematode): + key words * anal pruritus, + appendicins, + rethnitis in gurls, « no eosinophilia: not invasive, + cmbryonated egg, + Rx: albendazole & Tricharis trichiura (whipworm, nematode): * key words- * diarrhes, + rectal prolapse, * eosinophilia, « Rx- albendazole Ascaris lambricoides (nematode): * key words * larval phase 1 lungs: cough, pneumonitis. ‘eosinophilia, * adults cause intestinal obstruction: no eosiophilia (not invasive), © Rx— albendazole cor mebendazole > — Ancyclostoma duodenale and Necator americanus (hookworm, pematodes): + key words- * jarval phase in kings, * mouth parts attach to tips of villi, « fron deficiency ancmnia, + larva infect soil and can penetrate fect, * eosinophilia, » Rx- albendazole = Strongyloides stercoralis (nematode): + key words~ + larval phase in lungs, + ehabsitiform larvae (not eggs) pass in stool,» infective Giariform larvae in soil can penewate fect, + autoinfection and ‘superinfection (especially in AIDS), + cosinophilia, + positive Entcro-Test, + Rx~ ivermectin or albendazole © Trichinelia spiralis (crichinosis, nematode): « key words * consacted in man by the eating of raw of poorly cooked pork contamning the encysted larvae of the organism (pig intermediate host) + larvae have a propensity for skeletal muscle: often become calcified, + pronounced eosinophilia,» ‘muscle pain, « orbital edema, « Rx— corticosteroids and albendazole Toxocara canis or eati (visceral larva migrans, nematedes): + key words + Toxooara canis (@ogs) and Toxocara catt (cats) are the etiologic agents in visceral larva migrans, * man (usually children) is an abnormal host and only larva develop: intermediatc host, + visceral larva migrans: hepatosplenomegaly, pronounced cosinophilia, « Rx- diethylearbamazine n Note: This material is copyrighted. All rights reserved. Edward Goljam, M.D. 2002 = Ancyclostoma spp. (cutaneous larva migrans or creeping eruption, aematodes): » key * caused by dog and cat hookworms, * usually contracted by children playing on sandy etek sandy playgrounds where dogs and‘or cats unnate, + larvae penetate the slan amd produce serpigmous tunnels in the sion which causes intense pricitus ae cooraeairiett © Hereeintalbesde * — Anisakis simplex (nematode): * key words- contracted by eating raw fish dishes such as sushi and sashimu, as well as pickled hermg. * larvae penewate gastric and intestnal mucosa and produce cramping abdorunsl pain, epigastnc distress, vormting and diarrhea within a few hours after bemg ingested, « Rx— physical removal of the orgamisms by endoscopic forceps * — Wucheria bancrofti, Brugia malayi (filariasis, nematodes): + key words- + mcroiilana of Fucheria bancrofti or Brugia malay: are the causanve agents, * transrmtted by the bite of infected mosquitos: Anopheles, edes, Cules, * maicrofilana characterisueaily crculate in the bloodsweam at fight and enter into the lymphatics» mature and produce an inflammatory reaction resulting in Imphedema (elephantiasis} of the legs, scrotum, et... * find microfilania in the blood at night: sheathed and lack nuclei in the tail, « Rx~ ivermectin = — Onchocerea volvulus (river blinduess, nematode): © key words- * microfilana of transmitted to man via the bite of an infected blackfly (Simulium), + larvae migrate across the eye and produce Dlimdness. + adulk worms lodge in the Inmphatics: produce subcutaneous nodules, + microfilana 60 not enter the bloodstream: 0 periodicity (night or day time rhythm}, « find microfilaria in a sen biopsy that are unshesthed and de not have nuclei extending to the tail, © Rx- wernmectn and remove the nodules = Dirofilaria immitis (heart worm disease, nematode): « key words~ + dog heart worm that usually lives in the nght ventricle of the dog, + microfilaria transmitted so man through the bite of a mosquito, + larva enier the right heart and embolize to the Iung: small pulmonary unfarct presenting as.acom lesion, # Rx- no effective drugs 2 Hymenolepis nana and diminutas (cestode): © key words- MC tapeworm infection ia Uoited ‘States, # Rx praziquantel + DiphyHobothrium latam (fish tapeworm, cestode): « key words- * larvaean lake trout, * cause of Bz deficiency, » Rx- preiquantel 3 — Taenia saginata (beef tapeworm. cestode): » key worts- * ingest larvac in undercooked beef man is definitive host, « Rx- praziquantel * = Taenia soliam (pork tapeworm, cestode): © key words + can contrast by eating larvae in undercooked pork (intermediate host): man defininve host, * can contract by esting eggs fom an infected human (definitive host) and only larva develops in patent (intermediate host): #called custicereosis, #CNS lesions with seizures « Rx~ praziquantel ® — Echinococeus granalosis or multilocularis (bydatid disease, cestode): * key words- + sheep with larvae (jnitermediate host)—+ eaten by dog (develop adults who lay eggs, definitive hast)}-+ rman ingests the egg (develops larvae which penetrate the liver, intermediate host), anaphylactic reaction if cyst fluid enters peritoneum, * Greek and Basque sheepherders, + Rx- percurineous ‘drainage + albendszole 7 Trematode disorders: + wematodes, or flukes, are flat, unsegmented worms with a complex developmental cycle, * in schistosomal disease, the cycle is as follows egg (human) —> ciliated miracidia larvae —+ infect snails (1 intermediate host) ~+ produce fork tailed cercaria larvae —+ penetrate skin of human —+ produce disease, « in non-schistosomal disease, the cycle 1s as follows: ‘egg (human) —+ ciliated miracidia larvae —> infect snails (1° intermediate host) produce fork tailed cercaria larvae —+ infect a 2" intermediate host: + aquane vegetation (Fasciolopsis buski, Fosciola hepatica), * fish (Clonorchis sinensis), * crabs (Paragonumus, westermani) —+ form 8 ay infective metacercaria > man ingests the 2™ intermediate host -> disease, © note that snails are involved in the life cycle of both schistosomal and non-sel Schistosoma mansoni, japonicum and hematobium (schistosomiasi ilharziasi trematodes): » infection acquired by the penetration of the ned crop aac pene snails into the skin which then enter the lymphatics and distribute into subcutaneous tissue and the mesenteric veins, © S. mansoni favors the intrahepatic portal veins~ + penetration of the skin by the larvae produces pruritus ("swimmers itch"), * larvae develop into adult worms that travel against the circulation and deposit eggs (egg has sharp lateral spine), to which the host develons an inflammatory response marked by concentric fibrosis ("pipe stem cirrhosis") in the vessel wall, $ hematobium favors development in the urinary venous plexus where the eggs (nipple at end of egg) incite an inflammatory reaction (hematuria common) and squamous metaplasia of the bladder epithelium, which predisposes to squamous carcinoma of the bladder, * Rx— praziquantel Clonorchis sinensis (Chinese liver fluke, trematode): * key words— * ingest larvae in fish (usually fish paste), + larvae ascend bile ducts into liver and gallbladder and become adults in the bile ducts, * causes jaundice and cholangiocarcinoma, * Rx— praziquantel Scorpions: + key words~ * poisonous species capable of causing fatality lives in Southwestern deserts (Centruroides gertschi), * neurotoxin, * bite site shows no initial reaction~» increased sensation» no sensation in the area of the bite+ whole extremity becomes numb-» increased blood pressure—> ascending motor paralysis death, « Rx— no specific treatment for the poisonous species Mites: © key words for chiggers— pruritic dermatitis best treated with topical antipruritie agents (crotamiton and calamine lotion), # human itch mite (Sarcoptes scabies}—* tissue injury by adult females boring into the stratum comeum, * burrows are visible as dark lines between the fingers, at the wrists, on the nipples, or on the scrotum, * females lays eggs at the end of the tunnel: responsible for intensely pruritic lesion, * adults: disease is limited to the webs between the fingers, intertriginous areas, and spares the soles, palms, face and head, + infants: no burrows, and the palms, soles, face and head are involved, * Rx: permethrin cream Head, body, and the public lice (“crabs”): * Pediculus humanis capitis is the head louse + lays its “nits,” or eggs, on hair shafts, * Rx: permethrin (kills newly hatched lice) followed by lindane (Kwell), if the initial Rx is unsuccessful, « Pediculus humanis corporis is the body louse + lives on the surface of the skin and breeds in the clothing, * Rx the clothing, not the patient: use malathion powder or DDT powder, * Phthirus pubis is the pubic louse (crabs)- * lives in the pubic hairs: looks like a crab, « Rx: permethrin Fire ant bites: « sharp, painful bite, « wheal/flare reaction: type I hypersensitivity reaction, + reaction followed by vesiculation/skin necrosis, « bites commonly occur in people that craw! under houses Patient with fever, cough, diarrhea/vomiting, organism requires special medium for calture: most likely transmitted by acrosolization from envi sources Virases: all have protein and nucleic acid Patients serum is reacted against antigen, sensitized RBCs, and complement. RBCs do not hemolyze: « complement fixation test, « lack of hemolysis indicates patient has antibodies against the antigen, « presence of hemolysis indicates no antibodies Interpretation of minimal inhibitory concentration: lowest concentration that drug inhibits growth, determine minimal bactericidal concentration: must culture the tubes to see which one has no growth of the organism Bloody diarrhea after returning from Mexico. Neutrophils in stool: amebiasis, « Rx with metronidazole ‘TB reactivates in apex: « highest 0,, ¢ TB is strict aerobe a4 qeaed 4 BAF a HIGH YIELD NOTES PHARMACOLOGY ® ‘AMlopurinol: inhibits xanthine oxidase Isoniazid: * causes pyridoxine deficiency leading to peripheral neur : ‘opathy and siderobl anemia, « MOA~ INH is nicotinic acid derivative that inhibits synthesis of mycolic aaa mycobacteria cell wall Suceinylcholine: © depolarizing agent used as muscle relaxant during surgery, » binds to nicotinic receptors in skeletal muscle causing persistent depolarization at motor end-plate, * hydrolyzes plasma cholinesterase, * initial fasciculations followed by muscle paralysis, + duration of action only S~10 min, « effects not reversed by cholinesterase inhibitors~ no pharmacologic antidote to overdose, # some patients have atypical cholinesterase and cannot metabolize the drug Preganglionic neurotransmitter acetyicholin : ¢ activates muscarinic and nicotinic receptors, © muscarinic effects- * pupillary miosis (contracts iris sphincter, used after cataract surgery), * accommodation of lens for near vision (contracts ciliary muscles), bronchoconstrictor, » slow heart rate (SA node effect), * increase PR interval (AV node effect), + stinmulate GI secretions, increases GI motility (enteric nervous system stimulation), * micturition (stimulate detrusor muscle, which relaxes internal sphincter of bladder) Nitric oxide: + potent vasodilator synthesized in endothelial cells, + activates cyclic GMP. which inactivates myosin light chain kinase in smooth muscle cells leading to vasodilatation, + Ach can lead to its synthesis when injected into vessels Woroan in third trimester has premature contractions: use terbutaline, a selective B2-adrenergic receptor agonist that inhibits uterine contractions Effect of aspirin on kidney: « decrease PGE; synthesis (vasodilator) leading to unopposed AT IT effect, « renal papillary necrosis Methotrexate: © blocks dihydrofolate reductase— macrocytic anemia due to folate deficiency, + S phase inhibitor, « leucovorin rescue, + interstitial fibrosis in lungs Peripheral neuropathy and cancer drug: vincristine Nerve-ending locations affected by botulinam poison, reserpine, guanethidine MOA of sucralfate: « viscous polymer of sucrose octasulfate + aluminum hydroxide that adheres to ulcer crater, « stimulates PGE, in mucous cells— responsible for mucous barrier Effect of mixing statin drugs with niacin: rhabdomyolysis~ each drug by itself can produce rhabdomyolysis MOA of amphotericin B: ¢ binds ergosterol in fungal cell membrane, which increases its | permeability, « nystatin has same mechanism MOA of clotrimazole: « inhibits ergosterol synthesis, » other azole compounds have similar action MOA of flacytosine: inhibits nucleic acid synthesis MOA of griseofulvin: inhibits microtubule function and mitosis Kaow the definitions for potency and efficacy of drag Propylthiouracil: « drug of choice for decreasing synthesis of thyroid bormone in Graves disease, ++ agranulocytosis, « only drug that can be used in pregnancy Fastest way to increase heartbeat in shock: + order of drugs with decreasing effectivencss is isoproterenol, dobutamine, and dopamine, « dopamine in low doses is best for renal vasodilatanon Dapsone: « sulfone that inhibits synthesis of folic acd by M. leprae, « may precipitate herolyic anemia in G6PD deficiency, + may produce methemoglobinemia, * peripheral neuropathy Prolongation of QRS on ECG: quinidine nephrouc syndrome. * | Note: This material is copyrighted. All rights reserved. Edward Goljan, M.D, 2002 944 9a FH HVT 4 4 444 4 949 aad 44 Anticonvulsanv/antiepileptic drug with effect on epiphyseal plate rerarig of the cytochrome system in the liver and causing vitamin D * eee 35 hydroxylated vitamin D 7 deficiency- incteased metabolism of MOA of ondansetron: « selective serotonin 5-HT3 er chemoreceptor trigger zone (CTZ) in the area postrema, eee ea Gating i Tntbloshing Opioid overdose: » respiratory depression~ respiratory acidass, « miotie pupils First Rx of alcoholic in ER: give IV thiamine to prevent acute Wemicke’s encephal Heparin: MCC of thrombocytopenia in hospital oat Acute Rx for coumarin overdose when bleeding is life-threatening: « fresh f choice, « use IM vitamin K for less serious bleeds eee erene Nitrofurantoio: » concentrated in urine, » urinary tract antiseptic Cyclophosphamide: » hemorthagic cystitis, © transitional ccl inoma, © in Cette = zie cys carci scuvated (not Doxorubicin and prednisone: metabolized in liver Gout after cancer therapy: « prevent by giving allopurinol (xanthine oxidase inhibitor, « danger of urate nephropathy Given log dose curve for 2 drugs and must compare efficacy and potency Muscle fasciculations after neuromuscular block: inital effect of succinylcholine MOA of levodopa: blocks penpheral copa decarboxylase Bronchial asthma; + albuterol mediheler most often used for mild to moderate asthma, « cortcosteroid medihaler used for severe asthma Rx of paroxysmal supraventricular tachycardia: adenosine, including its association with WPW syndrome MOA of clonidine: activates o-adcenergic and imidazole receptors in the central nervous system which reduces sympathetic outflow from the vasomotor center in the medulla. Captopril: « bradykinin side-effects include cough and angicedema- ACE normally degrades bradykinin, + cough not seen with losartan ‘Hydralazine: » better tolerated in fast acetylators, « cause of drug-induced SLE, « can be used in hypertension in pregnancy Patient with hypertension, hypernatremia and hypokalemia: « losartan, an ATU reoxpior antagonist, is the best drug, » corrects HIN and electrolyte problem (blocks release of aldosterone, hence sodiurn is lost in the urine and potassium retained) Rx of pulmonary edema: loop diuretic is very useful- decreases preload Re of opioid overdose: naloxone, which is competitive opioid receptor antagonist Urinary retention in prostate hyperplasia: + use selective ay- blockers~ relax smooth muscle m bladder neck and prostate, © examples * prazosin, * terazosin MOA of saw palmetto: + anti-androgen effect blocks androgen uptake and availability without altering serum levels, « causes shrinking of transitional zone of prostate around the urethra ‘Narcolepsy: Rx sith amphetamines Premature labor and must deliver bab: ive glucocorticoids to increase surfactant synthesis Interpret dose/response curves and whether antagonist is competitive and non-competitive Drug used to eliminate calciam in patient with bypercalcemia: loop diuretic ‘Drog used to remove calcium from urine in a calcium stone former: hydrochlorothiazide Patient with respiratory alkalosis and then metabolic \cidosis: salicylate intoxication S-phase drug used in treatment of acute lymphoblastic leakemia: methowexate ‘Analog of hypoxanthine requiring HGPRTase for bioactivation: © 6-mercaptopunne, * allopurinol could cause toxicity, since the drug is apurine Drug that competes with dUMP for thymidylate syathase: 5-fluorocuracil Drag used in Rx of Wilms tumor: actinomycin D (dsctinomycin) Note: This material is copyrighted. All rights reserved. Edward Goljam, M.D. 2002 v4 990499 48 a £* 94944 a4 ‘Antimetabolite that together with an antibiotic is used Cyarabine + daunomyein (anthracycline antibiotic) a Rofideate myetegeeons leks Nephron site responsible for ACE Inhibitors causing increase 1 TI (works as vasoconstrictor of efferent arteriole : lomenlts) by ACE Eile eae a important control for maintaining intrarenal blood flow causing potential for renal fail (parncularly with bilateral renal artery stenosis) nam Nephron site of action of thiazides: Na‘/CT pump in cortical thick ascending limb Vitamin given in Rx of TB: « pyridoxine (B6)- used a ‘aout iend sfderoe inate a ea ea Drags that are generally safe in pregnancy: » safe— * methyldopa, + hydralazine, + low doses of propyithiouracil, * cephalosporins, * penicillin, « not safe— * methotrexate, * ACE inhibitor, retinoic acid, + sulfur containing drugs Why is gentamicin ineffective in Legionella infections: © aminoglycosides affect the 30S ribosomal subunit, « resistance to any drugs that are protein synthesis inhibitors may be due to- « inautivarion of the drug by bacterial enzymes, + decreased uptake of drugs via porins in bacterial ‘membranes, * decreased binding of drug to 305 ribosomal subunit Classic ECG changes for cardiovascular drugs: « prolonged QRS with quinidine, amiodzrone, « prolonged QT interval with sotalol, amiodarone, © prolonged PR interval with digitalis, « B- blockers, « calcium channel blockers, « giant U wave with quinidine, « ST depression with digitalis (hockey stick configuration), quinidine, « short QT interval with digitalis, * fat T wave with quinidine, » bradycardia with quinidine Know mechanisms for resistance to various classes of antibiotics: note the ones above for 30S protein synthesis inhibitors~ * tetracyclines, + aminoglycosides, * spectinomycin Patient with Salmonella and GOPD deficiency: avoid TMP/SMCX (sulfur drug) MOA of disulfiram reaction with metronidazole: inhibits aldchyde dehydrogenase MOA of norfloxacin: inhibits DNA gyrase ‘Avoid tetracycline in pregnancy MOA of erythromycin: ¢ inhibits 50S ribosomal subunit in bacteria! protein synthesis, ¢ also other macrolides, « chloramphenicol, « clindamycin Chloramphenicol: aplastic anemia in adults- idiosyncratic, not dose dependent as it is in newborns (gray baby syndrome) < Know blockers for different acetylcholine receptors Child eats honey and gets weak: © botulism, « intestinal colonization of organism with toxin production- toxin blocks the release of Ach ‘Autibypertensives increasing blood lipids: « B-blockers, + thiazides MOA of trimethoprim: blocks dihydrofolate reductase Mascle relaxant for electroconvulsive Rx: succinylcholine Rx of prostate cancer: * leuprolide— (GnRH agonist blocks FSH and LH and lowers testosterone and DHT, « flutamide— competes with testosterone for androgen receptor, * finasteride * blocks So-reductase, # decreases DHT, « increases testosterone (good for preventing impotency and hair growth), + mainly used in prostate hyperplasia, « ketoconazole + reduce testosterone synthesis, « more often used in hyperplasia Young woman with vaginal cytology containing malignant cells: DES exposure with clear cell adenocarcinoma CMV infection: ganciclovir first, then foscamet if it does not work Most common side-effect of immunosuppressamt drugs: « infection, » squamous cancer of skin ‘MC cancer Rx of absence seizures: » ethosuximide a2 9 99d 4a VIG aG a4 Vad EEG in patient with absence seizures: * shows 3/sec spike and wave . , hyperventilation » discharge, + increased with Drug abuser with flu-like syadrome, fever, mydriasis: methadone + naloxone __Patient with polyuria, polydipsia: taking lithium= i Rx of ADHD: methylphenidate insipidus... Rx of depression: tncyclic antidepressants— clomipramine blocks neur hd xerovonun (#3) ‘onal reuptake of NOR (+2) Phenytoin interaction with theophylline: phenytoin revs up ¢} metabolism of theophylline 2 Selim gelonoed karen Effect of chronic use of a hypnotic drag to induce sleep: psychologic dependence Patient with Wegener's granulomatosis has hemorthagic cystitis: cyclophosphamide effect Kis the active form of vitamin K Patient with hypertension has SLE-like syndrome: hydralazine Vasoconstrictors that increases IP: drugs activating a-adrenergic receptors Know drugs that have their effects on pacemaker action potentials Know mechanisms of action at receptors (adrenergic, cholinergic, hypothalamic releasing factors, histamine, VIP): e.g, cAMP, IP3/diacyclglycerol, etc. Iron poisoning in child: biggest concer is hemorrhagic gastritis and liver necrosis. Receptors, mechanisms of transduction, neronal tracts, functions: * al, ¢ «2, © Bl, « 62, *D types for dopamine, « GABA, « histamine, » 5-HT types for serotonin, ¢ muscarinic, nicotinic, « NO, « opioid receptors 5, x, Ring finger with blue discoloration on undersurface and Hx of drug abuse and frequent sexual activity: IV heroin abuser using finger as site of injection that is selling sex for drugs Gold salts: « used in Rx of rheumatoid arthritis, » oral drug called auranofin, « parenteral form called goid sodium thiomalate, * takes 3-6 mths before effects are noticed, © complications— + potentially fatal dermatitis/stomatitis common, * bone marrow depression, * flushing, * hypotension, « tachycardia, renal disease Hydroxychloroquine: © used in Rx of rheumatoid arthritis, * complications- + retinal degeneration, * dermatitis, + bone marrow depression Penicillamine: « used in Rx of rheumatoid arthritis, ¢ complications * aplastic anemia, + renal damage (membranous glomeruloncphritis) * Penicillin MOA: » B-lactam antibiotic- cell wall synthesis inhibitor, + binds to receptors in bacterial cytoplasmic membrane, * inhibits transpeptidase enzymes that cross-link peptidoglycan chains in bacterial cell wall, « activates autolytic enzymes in bacterial cell wall Chloramphenicol MOA: « inhibits bacterial protein synthesis, « binds to 50S ribosomal subunits, hence indirectly inhibiting transpeptidation by peptidyltransferase Mesna: + mercaptocthanesulfonate, » reduces incidence of hemorchagic cystitis associated with cyclophosphamide by reducing formation of acrolein Ribavirin MOA: « inhibits guanosine triphosphate formation, which prevents capping of viral mRNA, « blocks RNA-dependent RNA polymerases, « used in Rx of RSV infections ‘Ondansetron MOA: 5-HT; receptor blocker- central antemetic Know pharmacokinetics: « halflife, ¢ volume of distribution, « maintenance dose Asninoglycosides: » affect 30S ribosomal subunit in bacterial protein synthesis, » nephrotoxic and ‘ototoxic, « MCC of drug-induced nephrotoxin acute tubular necrosis, * ototoxicity related to accumulation in labyrinth and hair cells of cochlea, « vestibular signs/symptoms— * nystagmus, * dizziness, + vertigo, * N/V, « cochlear signs/symptoms- * tinnitus, * sensorineural hearing loss H. pylori: « Rx amoxicillin + clarithromycin + proton pump blocker, * prevention proton pump blocker + bismuth 94 q4 Antifangal drug destroying fungal cell membranes: « polyenes fi bind to ergosterol in cell membrane and cause formation OF ae een B polyenes toxicity of amphotericin: distal RTA with magnesium and potassium wastin 1, © baci synthesis of ergosterol in cell membrane, » flucytosine blocks nucleic acid synths, ees oe disrupts microtubules ‘Synthesis, ¢ griseofulvin Alkylating agents MOA: « alkylate DNA, RNA, other . strands and cross-linking which inhibits strand replication Proteins, « leads to breakage of DNA HMG-CoA reductase inhibitors: » statin drugs block CH synthesis, « hepatocytes up-regulation of LDL. receptor synthesis leading to increased clearer, sompensitt by ar peiatonsoh ID z clearance of IDL and LDL remnants Sildenafil: + drug for Rx of erectile dysfimction, + inhibits breattown of cGMP by npe $ hosphodicsterasc~ increascs levels of cGMP, which causes vasodilatation in corpus cavereoary and penis, « should not receive nitroglycerin, since it ca eee vasodilator), which elevates levels eee mses release of nitric oxide (potent Nieroglycerin MOA: © release nitric oxide (vasodilator) in endothe-al cells, « primarily a venodilator— causes venous pooling and reduces preload, » has some ar:-vial vasodilatation when administered IV reduces aftcrioad, » uses— + angina, + acute myocardial iafarcuon Nitroprusside MOA; « used in hypertensive emergencies and in dissecting aortic aneurysms, © metabolized to cyanide in RBCs cyanide converted 10 thiocyanate, must measure levels of thiocyanate during Rx to avoid toxicity Phentolamine: « nonsclective a-adrenergic receptor antagonist competitive receptor antagonist, « produces vasodilatation reduces TPR and blood pressure, + reverses effects of epinephrine, « clinical uses~ * hypertensive episodes due to pheochromocytoma, « reversal of effects of accidental injection of epinephrine in subcutaneous tissue Naproxen: + newer, long-acting NSAID that blocks cyclooxygenase, * very significant potential for renal damage in those with preexisting renal disease Doxorubicin: + antibiotic, « MOA~ = interealate base pairs, + interact with topoisomerase H, « generate fice radicals, * block synthesis of DNA/RNA and cause DNA strand scission, « side effects * dose-dependent congestive cardiomyopathy via fice radical damage, * marrow suppression, « dexrazoxane, a FR scavenger, may protect against cardiotoxicity . MAO inhibitor used in treating a patient with resting tremor: « sclegiline, » blocks MOA type B, which normally metabolizes dopamine, so brain levels increase, » hepatic metabolism produces amphetamine Dantrolene MOA: « blocks caleium release from sarcoplasmic reticulum of skeletal muscle, © DOA in Rx of malignant hyperthermia, « used in Rx of muscle spasms in cerebral palsy Digitalis: blocks Na/K” ATPase pump in cardiac muscle- 3 Na into muscle and 2 K* out of muscle Antihypertensive with Coombs positive hemolytic anemia: » mcthyldopa, © alters Rh anges con surface of RBC, £0 IgG autoantibodies react agains it, « type II hypersensitivity Know different insulin preparations and times of onset of activity Fluoroquinolone MOA: « blocks bacterial DNA syathesis by inhibiting topoisomerase I (DNA gyrase) and topoisomerase IV, « DNA gyrase important in relaxation of supercoiled DNA involved im normal wanseription and duplication, © type IV topoisomerase important in separation of replicated chromosomal DNA during cell division Benzodiazepines: retrograde amnesia Propylthiouracil MOA: « reduces iodination of tyrosine, « reduces coupling of MIT and DIT to form T; and T., « inhibits peripheral conversion of Ts to Ts by inbibiting the outer ring 5 deiodinase, « side-effects rash, agranulocytosis qq4a4 944 944 4 99aedd "@ a4 Drug lowering HDL: + probucol, * may cause torsade configuration, » type of polymorphic ventricular tachycardia, « Drugs causing torsade de pointes: drugs that slow ventric interval- * quinidine, + sotalol, * phenothiazines, * cisapride MOA inhibitor associations: « inhibit MAO type A (metabolizes NOR, serotonin, pramin) a B (metabolizes dopamine), » hypertensive reactions if taking foods i tyramine- poke = ‘yrosine metabolism, + bananas, * fermented products like cheese, beer, wine, «symmpathona “a reaction in that it causes the release of stored catecholamines lead: bypertensio administered with fluoxetine (SSRN) produces serotonin syndrome. * np Men, (failure to relax muscle} myoclonus Stage frighttest anxiety: give propranolol 1B, antagonists for Rx of hay-fever B-2 agonists: activate adenylate cyclase to increase cAMP Bronchial asthma: « zileuton~ inhibits S-liporygenase, zafirlukast and montelukast block leukotriene receptors MOA of methadone: Rx of opioid abuse, « saturates CNS opiate receptors Receptor involved in tardive dyskinesia: D, Phencyelidine toxicity: + angel dust, reacts with opioid-like sigma receptors and subtypes of glutamate receptors, * dissociative anesthesia~ e.g, loss of sensation in lower limbs, agitatiowviolent behavior cannot bold patient down Acyclovir MOA: » guanosine analog tbat is activated to form acyclovir triphosphate a competitive substrate for DNA polymerase- subsequent chain termination following incorporation into viral DNA, « clinical uses varicella/zoster virus, mucocutaneous/ genital herpes Girl on rifampia becomes pregnant while on birth control pills: rifampin revs up the liver increases longevity in CHF 4 “Arsenic poisoning: Rx with dimercaprol Chloroquine was used in the treatment of malaria and malaria recurred—why?: © cxoerythrocytichhepatic stage ¢,, P. vivax, P, ovale, © drug Kills active disease but docs not eradicate hepatic stage a Primaquine in treatment of malaria: not good in the active stage but does kill the hepatic stage of P. vivax and ovale = Dantrolene: « reduces the release of calcium from the sarcoplasmic reticulum of skeletal muscle, antispasmodic drug, « also used in treating malignant hyperthermia Methanol: » increased anion gap metabolic acidosis due to conversion of methanol into formic acid, « optic nerve degencration and blindness, « treat with aleabol infusion to block metabolism of methanol by alcobol dehydrogenase a Botulism toxin: « blocks the release of acetyicholine~ diagram may be given of neurotransmitter synthesis and must locate the block, « used to treat LES spasm in achalasia Ribavirin: used in severe RSV infections in children ‘Rx of asthma albuterol: © B,-selective agonist bronchodilator, © albuterol may cause ypokalemia~ drives K" into cells by activating the ATPase pump @ Acetylcholine breakdown: occurs in the synapse into choline and acetate by acetylcholinesterase in the cleft, « products are recycled and not excreted a % aaa qaqa a4 a4 87 a = Ketoconazole: inhibits the metabolism of nonsedsting antihist cardiac arrhyimuas He peldiees Seer Codeine sseuboler ie morphine in small amounts owmg to significant first pass ston Delirium tremens; Rx with benzodiazepines Lovastatin; intubyts HMG CoA reductase Patent ductus arteriosus: keep open with PGE, Cepbalosporins + aminoglycosides: synergisuc effect of enhancing neptrotoucry Erythromycin: © interacts with the SOS subunm of bacterial ribosoraes leading to inhibstion of protein synthesis, © inhibits the formation of the mutanon complex and interferes with tanslocanon reachons, © resistance develops secondary to plasrmd-mediated formanon of enzymes that methylate the receptor that erythromycin binds with, « coliforms produce a tensmissible plasmd that produces an esterase that hydrolyzes the lactone ming of exychromycm Chlorampbenicol: gray baby syndrome- dose related aplastic anemia TMP: « inhibits dihydrofolate reductase, ¢ plasmid-medizied resistance, « TMP ~ SMX has a synergenc effect from the sequential blockade of folate symbens, since SMX blocks dcopterate synthase, which iS an enzyme that converts paraammnoberzpic aci¢ mo Ehydrofolic 4a yey ay 2eid Metronidazole: produces a cisulfirem-like reacton Flnconazole: teatnent of esophageal candidiasis in AIDS Cholinomimetic used in treating open angle glaucoma: « pilocerpine, « physostigmine Dobutamine: + motropic (merease contractility) vasodilator (decreases afterload) thar activates a and 81 >Q2acnvity without much chronotropic effect, © used in the meamnent of shock (cardiogenic) associzted with hypoteasion and hypotension associated with reaal failure or CHF Dopamine: « stimulates cardiac B1 receptors, peripheral a-receptors, md dopaminepe receptors in vessels in the renal and splanchnic bed, « at low doses, it is pramarily 2 vasodilator ther mermzses renal 2nd splanchnic blood flow, « at high doses, 7 increases cardiac contractility (mozopic) znd ceré:ze oumput via its 2ccvation of cerdiac B1 receptors 2 Benzodiazepines: « via its own receptors in the thalamus, limbic stucaures, and cerebral comex hich are part of the GABA ceceptor-chloride ion chamel macromolecular complex benzodiazepines facilitate the inhibitory acnon of GABA via increased conductance in the chionde jon chamels, « flumazeni] blocks this effect by blocking the receptor for beuzodiazepines aad is che treatment of choice for benzodiazcpam overdose ‘An elderly woman on thiazides is wost at risk for developing?: diabezes meffins ‘A child who ingests 30 adult aspirins will mest likely develop?: + ax increased muon gp cReabolic acidosis, « children, unlike adults, do not commonly develop a mixed metabobc acidosis ind respiratory alkalosis, « Rx is t0 perform gastic lavage and add activated charcoal end 10 produce an alkaline urine for iocreased excretion of the acid pen angle glancoma ts best treated with?: » Bi and B2 blocker, » timolol is a favored seb ° plocaipine way also be wsed, + open angle glaucoma is the MC type of plamcoma- prodocss gradual loss of peripheral vision (amnel vision) and opsic azrophy = Framdone: « second generation anti-depressant, + inhibition of serotonin reuptake, © sedanor. © 444 4 q q a4 dean a) algo protects against osloporosis 2nd can be used im women who tere ES positive tumors, * can be used in treating progestin-resistant i ‘flushing (menopausal -puopioms), vaginal Bleeding, potential for endometrial byperplasa/-ano= Note: This material is copyrighted. All rights reserved. Edward Goljan, M.D. 2002 4a Ve ©1994 4 ‘Acetaminophen: * analgesic and antipyretic but not an anti-inflammat : prostaglandin synthesis in the CNS, # very weak cjabueipeie: GINiieas wcities sala rere Fnac Reratiia(s cont iol lite ACG Wialats Coiloer-ce glia iene Micaiccaa FRs,» acetyicysteine treatment replaces GSH——~————--—- poate: paces Digitalis toxicity: reat with digoxin antibodies (FAB fragments) Cephalosporins: + first goneration- drug of choice for surgical prophylaxis in many cases,» Saeed grecrndbi ©7RS of Bauaits (cebartime), * Weal ined asacroiellnfericoas ind generation * Rx of meningitis, « Rx of GC and Lyme's disease: ceftriaxone Clozapine: blocks D, (dopamine) and 5-HT;, receptors moreso than Dy receptors Fever in a patient on a loop diaretic: » do not use any type of NSAID (including aspirin) since it blocks renal synthesis of prostaglandin, which vasodilates the afferent aneriole, + use acetaminophen to lower fever, since it does not interfere with prostaglandin synthesis, » loop diuretics Iead to volume depletion, hence angiotensin II will be elevated (vasoconstrcts efferent ateriole)- loss of prostaglandin effect predisposes the patient to renal failure Red man syndrome: IV vancomycin Drug for bacterial carrier states (S. aureus, N. meningitidis, A. infhuencae) rifampin Single dose drug for GC and Chlamydia: azithromycin Rix of malignant hypertension: nitroprusside Antihypertensive and antiarrhymic drug that lowers blood pressure and increases heart rate: calcium channel blocker~ e.g. nifedipine Estrogen: « lipid effects~ + lowers LDL/VLDL, + prevents osteoporosis inhibits osteoclast activating factor (IL-1) secreted by osteoblasts, * tuombogenic— * natural estrogens are less thrombogenic than synthetic estrogens, * increase is of coagulation factors, * decrease 'ATIIL, « increases liver synthesis of tanscortin and thyroid binding globulin increases total comtisaV/thyroxine levels without an increase in free hormone, + increases liver synthesis of sex (testo: ‘or andros ins lowers iree testosterone, * cancer risk- * endometrial, * breast, + intrahepatic cholestasis, « cholelithiasis ‘Oral contraceptives. «pill effects— + inhibit LH surge, which prevents ovulation, + increase malar ‘eminence pigmentation- "pregnancy mask", » ethinyl estradiol (synthetic estrogen) increases liver Synthesis of many proteins, + 19-nortestosterone (progestatonal agent) effects ° water” nention/weight gain, + reduction in estrogen receptor synthesis (atrophy of endomenrial glands), + noase LDL, * decrease HDL, complications + thrombogenic, * stimulate tryptophan vn tabolisrn lowers serotonin-» depression, * increases liver synthesis of angiotensinogen: MCC of hypertension in young women, + intrahepatic cholestasis, + hepatic adenoma: tendency 49 TuPHre. © inarease gallstone formation, * cancer risks~ + cervical, + breast controversial, « ‘hepatocellular carcinoma, « protectivelpreventive effects of pills * fibrocystc change in the breast» ‘endometrial saever * pvanan cancer: less ovulation reduces risk for cancer, * pelvic inflammatory disease: Neisseria gonorrhoeae not Chlamydia trachomatis, uterine leiomyomas, * endomemosi, © sens, = rheumatoid arthritis, * hirsutism Tous tonieity, «clinical serting- accidental overdose of ferrous sulfate in children, « SS of iron tonicity.» hemorshagie gastritis, hepatie necrosis with liver failure, « shock/metabolig acidosis, * scmays reveal undigested radiopaque pills in GI wact, « Rx ~ + iron-binding agents * coral phosphate 2x bicarbonate salts (precipitate unabsorbed iron), * parenteral deferoxamine Organophosphate poisoning: « MOA- * i (noncompetitive inhibitor), + accumulation of acetylcholine st sypapses/myoneural junctions, * source- pesticides, * initial autonomic system overactivity- * excessive lacrimation/salivation, fecal incontinence, constricted pupils, * nicotinic effects later in toxicity~ ¢ muscle ine Note: This material is copyrighted. All rights reserved. Edward Goljan, M.D. 2007 44 weaknessiparalysis, “+ muscle fasciculaions, © low serum and REC cholinesterase (pecudocholinesterase),« Rx * atropine Rx of choice, + pralidoxime (2-PAM) also may be used er hypertenston (HTN): weight reduction is the most important factor in lowering BP Casbonte anhydrase inbibitors: « MOA~ * blocks the proximal reclamation of bicarbonate, = thearbonate binds with Na" fons ands excreted in the urine (diuretic effect), + produces a nm ee rreuabelie acidosis from bicarbonate lost in the urine, « clinical uses~ * chronic management of fHavcoma reduces the rate of aqueous humor formation and reduces intraocular pressure, * siehimzes the urine: good for Rx of drug toxicities (eg, salicylate intoxication), © Exof gue Mountain sileuess: produces metabolic acidosis, which isthe compensation for respiratory alkalosis ‘Ginteal uses of thiazides: «initial drug used in Rx of HTN in elderly patients also reduces the cievenee of strokes and fatal AMIS inthis age group,» inital drug used in Rx of systolic HTN in the elderly, © one of the initial drugs used in Rx of HTN in African-Americans, » Rx of choice it wre theat ‘of nephrogenic diabetes insipidus: volume depletion from increased proximal reabsorption of Na” and water reduces degree of polyuria, « Rx of hypercalciuria in calcium stone formers, « Rx of proximal RTA increases reclamation of bicarbonate (Mimics uses of loop dinretics: initial drug used in Roc of HTN in chronie renal filme, « Rx of crane for hypercalcemia after @ diuresis is started with isotonic saline, » congestive heart fale creh acute pulmonary edema, * Rx of halide poisonings— * fluoride, » bromide, » acute renal Talure flush out tubular cells obstructing che lumen and can change an oliguric to polyune renal failure POX of spironolactone: * blocks the aldesterone-enhanced Na‘/K” pump in the Tate dist roa ed tubule and collecting duct~ * K” sparing effect, * danger of hyperkalemia, « blocks the protonK” ATPase pam in the collecting tubules danger of normal ‘AG metabolic acidosis, * Plocks androgen receptors~ * Rx of hirsutism, + produces gynecomastia in rea On, of teiamterene/amiloride: * they are not aldosterone inhibitors, « they block Na’ Meabeorption and the secretion of Kin the aldosterone-enhanced NaF parse 1K" sparing effect MOA of nonselective B-blockers: + block Brreceptors jin the heart * reduce sympathetic aramieton, « reduce blood pressure, + decreased contractility ofthe Beart decreas heart rate, * aareaeeoetion of renin in kidneys, + block Br-teceptors in the smooth muscle and liver blocks ‘catecholamine-induced giycogenolysis 5 SOA of selective By blockers: cardioselective @-blockers~ + primarily cnget Brreospior: heart, + less bronchoconstriction than nonselective blockers Side-effects of B-blockers: * conduction disturbances- ¢-g-. AV block, © block adrenergic signlsymptoms of insulin/oral sulfonylures-induced hypoglycemia *n diabetes tmellitus~ does not ase pronchospasm in those with asthma~ less bronchocoostriction with "depression, * lipid effects (mainly in non-selective B-blockers)- * increase TG. * siteace HDL, = rebound angina, * hypertension if sbruptly withdrawn” up-regulation of adrenergic receptors occurs: when they are blocked ‘Clinical uses of Grblockers: * hypertension, * angina, + congestive heart failure and acute tnyocardial infarction increases survival, « cardiac armhythmias, essential tremor— beaign condition with trembling of hands, * prevention ‘of migraine headaches. = Graves’ disease— blocks 100 mm Bg) in pregnancy: ¢ methyldopa, © hydralazine alternative drug choice Drag used ia Rx of HTN in a patient with urinary retention from prostate hyperplasia: c- adrenergic blocker Drags used in Rx of HIN in asthmatics: calcium channel blockers Initial drug used in Rx of HIN in a patient with osteoporosis: thiazides— increased calcium reabsorption in the kidneys ACE inhibitor: « decreases preload- inhibit aldosterone, « decreases afterload— inhibit ATO Hydralazine, minoxidil: decrease afterload alone- vasodilators Nitroprusside: * decreases preload- venodilator, * decreases afterload— vasodilator, * Rx of choice in treatment of malignant hypertension and lowering blood pressure in a dissecting aortic anewysm hiazides: decreases preload alone— reduces volume by losing salt and water Calcium channel blocker: decreases afterload alone— vasodilator Probucol: « an antioxidant that is used in treating recurrent xanthelasmas, * may produce torsade de pointes (polymorphic ventricular tachycardia), * lowers HDI. levels Nicotinic acid and fibric acid derivatives: activate capillary lipoprotein lipase, hence increasing the hydrolysis of VLDL in. the circulation HMG CoA reductase inhibiters and fibric acid derivatives: should not be used together since they both may be associated with rhabdomyolysis. ‘and the potential for ‘polyuric acute ‘renal failure HIMG CoA reductase inhibitors, ‘nicotinic acid, and fibric acid derivatives: associated with drug-induced hepatitis with elevation of the transaminases a ‘Note: This material is copyrighted. Alt rights reserved. Edward Goljan, M.D. 2002 9494 9448 £TE9TR FE 4 4 a4 HMG CoA reductase inhibitors and bile a hepatocytes, hence lowering serum CH and toe Tesins: upregulate LDL. receptor synthesis on HG Co A rednciase Sahbitors and ncsinle ald inhibi exzymes inter mechanism of Flushing associated with nicotinic acid: controlled by pretreatment with aspiri Fibre acid derivatives: greatest overall effect of all liptd eae tceen tty HDL levels BMG CHA reductase inhibitor and fibric acid derivatives: increase the chance for warfarin Bile acid resins: lower warfarin levels, hence causing undercoagulati Bile acid derivatives: cause a malassimilaion of drug that eecomooui used even of heart disease Nicotinic acid: drug of choice for familial combined by . HMG Cod reductase inhibitors: drugs of choice for Tarerng (DL pawl Pidisleneiehe () Nicotinic acid and fibric acid derivatives: significant effect in lowering triglyceride levels Estrogen: « lowers LDL levels, + increases bone density Fibric acid derivatives: associated with the inappropriate ADH syndrome ‘Types of drugs of abuse (DOA): « sedatives— + barbiturates, * alcohol, « stimulants cocaine, « hallucinogens~ lysergic acid diethylamide . MC DOAs in adolescents: ¢ marijuana, ¢ alcohol NS effects of long-term drug abuse: damage to neurotransmitter receptor sites Drug screening: « urine best screening medium for DOA, « blood also used in drug assays Sympathomimetic DOA syndrome: * exarmples- + amphetamines, * cocaine, © S/S- * tachyeardia/sweating, * mydriasis: pupillary dilatation, + hypertension, « hyperthermia Opiate/sedative DOA syndrome: examples— + heroin, + benzodiazepines, * barbiturates, + S/S— + respiratory depression—+ respiratory acidosis, + miotic pupils pinpoint pupils, * hypotension Anticholinergic DOA syndrome: * exarples-* antidepressants, * antihistamines, + antiparkinson-type medications, * atropine, + muscle relaxants, » S/S— + mydriasis, + fever, * dry skin Psychedelic/haliucinogenic DOA syndrome: cxamples- * PCP, * LSD Antidotes used in unconscious patients: « dextrose- R/O possible hypoglycemia from insulin overdose, « naloxone— possible opiate overdose, * intravenous thiamine— glucose may precipitate Wernicke's encephalopathy in alcoholics with thiamine deficiency Intravenous drag abuse (IVDA):*MC localized infection is skin abscesses duc to Staphylococcus aureus, « HBV MC systemic infection, « HIV, * infective endocarditis- * MC tricuspid and aortic valve, « S, aurens MCC, « tetanus- complication of “skin popping” Beroin: + derived from poppy plant, * usually “cut” with some agent (¢.g, quinine, talc) granulomatous reactions occur in skin/ungs from the cutting agents, » ngn-cardiogenic pulmonary edema frothing from the mouth is common, * focal segmental glomerulosclerosis~ hypertension + nephrotic syndrome, # Rx- naloxone, a morphine derivative with a high affinity for opioid binding. sites of the mu receptor type ‘Meperidine: « MC DOA in health professionals, « -methyl-4-pheny!-1, 2, 3, 6etrahydropyridine (MPTP)- + by-product of attempted synthesis of meperidine, * produces is: cytotoxic to neurons in nigrostriatal dopaminergic pathways Methadone: « legal synthetic opioid taken orally~ mainly used to detoxify opiate abusers, « tong acting drug- + sarurates CNS opiate receptors, * prevents sudden cuphoric action associated with heroin, * causes physical dependence/tolerance Benzodiazepine toxicity: * MOA- enhance the frequency of opening up of GABA, receptor chloride ion channels increases chloride ion conductance, * GABA inhibitory neuromansmitter, + dru i alegho!_wit « Rx of 9 aq garg wales fiumaaeni an antagonist of benzodiazepine and does not block barbi ot why Barbiturate toxicity: « MOA- * enhance the duration of openi Deira ot: MOA on te ini f ing of be ABA, cee activating system-+ inhibits the inhibitory effects of GABA and glycine (an amino acid ntubitery neurotransmitter), * bullae over pressure points: erythema . increases its excretion and lowers blood concentrations a, mace Cocaine: « MC COD from DOA in United States, « MOA- blocks upta doparne/NOR by presynaptic axon, « predisposes to—* sudden death, :, oie | ee infarction (AMD), * stroke, * pulmonary edema, * ventricular arthythmias, + myocarditis, © S/S- » hypertension/sinus tachycardia, psychosis/seizure activity, mydriasis 7 Amphetamine MOA- release catecholamines from presynaptic terminals, * examples— of amphetamines: + dextroamphetamine: Rx obesity, * methylphenidate: Rx ADHD and narcolepsy, ¢ methamphetamine; “ice” is street form of the drug, « hallucinations drug that most simulates schizophrenia Marijuana: © MC illegal DOA used in United States, « MOA~ © contains the psychoactive stimulant A’-tewahydrocannabinol (THC), * THC binds to receptors in substantia nigra, globus pallidus, hippocampus, cerebellum, + derives from leaves/flowering tops of hemp plants (Cannabis sativa), * hashish is extracted resin of marijuana that has 510 times the poteney of the parent compound, + high lipid solubility: THC is present in urine for more than a week, « clinical uses * cancer: decrease N/V in cancer patients, * lower intraocular pressure in glaucoma, * analgesia, = SIS of marijuana use- * reddening of conjunctiva, * euphoria, * delayed reaction time: engineer driving a tain involved in a crash with an oncoming train was found to have THC metabolites in tis urine LSD (lysergic acid diethylamide): « MOA sergot alkaloid that binds to D, dopamine receptors in the brain, + also blocks 5-HT; serotonin receptor in peripheral tissue, « predisposes to chromosomal ing to congenital defects, « S/S of LSD toxicity~ * heliuemations, + flashbacks PCP (pbencycfidine): « angel dust, * MOA— reacts with opioid-like sigma receptors and subtypes of glutamate receptors (antagonist), + initially introduced as_a dissociative anesthetic: separates bodily functions from the mind without a loss of consciousness, » S/S of PCP toxicity agitatjon/violent behavior, * coma with the eyes open, * impervious to pain {ise of B-blocker in Rx of Grave's disease: + thyroid hormones normally upregulate synthesis of Brreceptors that interact with catecholamines and produce many of the symptoms of thyrotoxicosis. © giving B-blockers, blocks the P-receptors, hence blocking the adrenergic symptoms of thyrotoxicosis Electrolyte changes when giving insalin: dives glucose into the cell along with potassium and phosphate Rex of carcinoid syndrome: « octreotide (somatostatin analogue), + also used in Rx of acromegaly and VIPomas Patient went into congestive heart failure. What drag could have potentiated this?: ° ‘verapamil, + potent negative inotropic effect Pationt with asthma not responding to albaterol: Rx with corticosteroid medihaler Patient with pnetmenia due to a grarm positive diplococcus (Streprococcus pneumoniae): Rx with penicillin G, which blocks cell wall synthesis Yobimbe: vasodilator used to Rx impotence ‘Ginkgo biloba: improves memory Ginseng: stress-protective Saw palmetto: Rx prostate hyperplasia Echinacea: © wound healing, « immunostimulant 3 Note: This material is copyrighted. All rights reserved. Edward Goljan, M.D, 2002 Kava: sedative Melatonin: « slecp, * Rx of jet-lag ‘St. John Wort: Rx of depression Jojoba: cosmetic, hair growth 7 oe Soy: phytoestrogen Green tea: antioxidant, decrease cancer Valerian: slecp HIGH YIELD NOTES PHYSIOLOGY® © Proximal renal tubule functions (See nephron dlagram): « primary site for Na" reabsorption, © primary site for reclamation of HCO; loss of reclamation leads to type Ul proximal renal tubular acidosis due to loss of Sicarbonate, » primary nephron site for synthesis of ammonia via enzymatic conversion of glutamu: : (non-toxic vehicle for carry NHL’ in blood) into NH." + a-ketoglutarate, « ‘primary site for reabsorption of glucose (cotransport with Na’), urea, amino acids, phosphate " = — Functions of thin descending limb: only permeable to water, hence the urine becomes extremely Iypertonic by the time it reaches the loop of Henle = Functions of thin ascending limb: impermeable to water but permeabl ° UOsm begins to decrease eS Seed Siete > Functions of thick ascending limb (TAL medullary segment): © gencration of free water via the active Na’-K7-2 Cl’ co-transport pump, this pump is the most sensitive part of the nephron for damage due to tissue bypoxia, « blocked by loop diuretics, © pump also reabsorbs calcium (not PTH-enhanced) @ Functions of the cortical TAL segment: « Na'/CT’ pump in carly distal mbule~ Na” and Ca ateel reabsorption) cations share the same channel for reabsorption, * blocked by = Functions of the macula densa: « interacts with the juxtaglomerular (JG) apparatus on the afferent ameriole, « increased Na” in the urine inhibits renin release and vice versa © Fanctions of aldosterone-enhanced ATPase Na’-K" exchange pump in distal collecting tubule and collecting ducts: « Na” is reabsorbed in exchange for K’, « effect of increased distal delivery of Na’ from more proximal acting diuretics (¢.g., loop diuretic or thiazide diuretic) * there is an augmented Na°/K" exchange, which may lead to hypokalemia and increased reabsorpaon of bicarbonate (metabolic alkalosis) & — Functions of the aldosterone enhanced HT'/i” ATPase pump and H” ATPase pamps in the a intercalated cells in the collecting duets (USMLE): primary sites for the excretion of excess HT jons~ dysfunction of the H'AK" ATPase purmp is the primary cause for type I distal RTA. Norma) ltution of urine: « UOsm in the late distal collecting tubule/collecting duet is normally 150 mOsm/kg primarily contains free water and a smaller amount of obligated water thar must aecompany salute,» when POsm is low, ADH is inhibited absence of ADH causes the loss of f= sealer im the urine, « positive free water clearance ~ * CH,O~ V - COsea, where CHO = free water vearance, V = volume of urine in mL/min, COsm ™ obligated water, « to calculate Ost: COs = UOsm x V/POsm, + «positive CH.O indicates dilution (free water is lot in the urine), + example: cam blume 10 ral, POsm 250 mOsm, UOsm 150 mOsm: COsm = 150 x 107250 = 6 mL, CHO * 10-6= 44 mL sr Nommal concentration of urine: « increase in POsm is stimulus for ADH release,« ADE! render aor erry collecting ducts permeable to fee water (not Na", cannot reabsarb obligated water) urine is concentrated, « negative CHO (free water is reabsorbed back into the blood) ¢€ ayaa Note: This material is copyrighted. All rights reserved. Edward Goljan, M.D, 2002 a4 CRO 0 ip = 0m. slay lecaeecuorr cs ee Some 0 WTO =m, : 30 = -20 mL. « ability to concentrate urine is the first abmormality in renal failure Central and nephrogenic DI: ¢ both have low UOsm and increased POsm, * ceatral DI shows $0% increase in UOsm with administration of . a in UOsmn wath vasopressin rasopressin, * nephrogenic DI shows <50% mecrease Respiration changes with increasing altitade: * respiratory alkalosis. hypoxemia due 10 decreued ates PESIE Ota decease m % oxygen na, + nereae 23 BPG ight sits Effect of VIP (vasointestinal peptide) and eakephalins on GI tract: » VIP- relaxes smooth muscle, increases intestinal secretion, increases panereaic secrenon, © contract smooth muscle, decrease intestinal secretion Seapets eo Know PCO; dissociation curve Ean | ste see Sertoli celis in seminiferous tubules, + negative feecback with FSH, « increased if semuniferous tubules are destroyed, * normal if Leydi tex: sterone bas a negative feedback with LH See Oe See ees Calculations: © alveolar ventilation- PAO: = PiO; - PACOVR, where PiO, equals % oxygen x 713) and R is the resprratory quotteat that aoraally equals 0.8, subwact PaO; com PAQ; and you have the A-a gradient, « Fick's equation for cardiac oumpur~ cardiac ourput miL'mn = oxygen consumption + oxygen in pulmonary vein - oxygen in pulmonary artery Hormone increasing/decreasing gastric secretion: © gasmm. hormones inhibiting and secrenon- secretin end gasine inhibitory peptide Placental anatomy’physiology: ¢ caternal surface bas slightly bulging areas called cotyledons, which are covered bya layer of decidua basalis, « feral surface ts enurely covered by the chorionic plate— chorionic vesseis converge with the umbilical cord, whieh is composed of 2 umbilical aneries (venous blood renzaing fom the fetal heart) and | umbilical vem (carries oxygenated blood trom the pizcenta), © chorionic villusumbilical coré~ + choronic vii project im the imervillous space, which contains maternal blood from which oxygen is extracted: spual arenes fom the uterus empty ito the space, * chorionic villi are lined by topboblasne tissue: ouside. layer is composed of syncynotrophoblat: symthesizes hCG and human placencil lactogea (growth hormone of pregnancy) and inside layer is composed of cytotrophoblast: clear ceils, the mumior of the chorionic villus fas fetal blood vessels, which coalesce 10 form the chorionic vessels that couverge wich the umbilical cord, + the umbilical cord contains 2 umbilical arteries (coumins deoxygenated Slood exiting the fetal heart and retuming to the placenta) and 1 umbibeal vein {contams oxygenated blood) Effect of increased total peripheral resistance on vascular fuaction carve: « same 3s the venous ream curve, which depicts the relationship between venous return 2nd right amnal pressure, * inereasing TPR (vasoconstiction) decreases venous retum to heart, decrease right atrial pressure, and decrease cardiac output this causes 2 counterclockwise rotation of the vascular functiog carve, « decreasmg TPR (vasodilation) causes a clockwise rotation of the curve— venous reper mereases, right atral pressure imereases, and cardiac oumput mcreases ACG: = has Itemizmg hormove acuvity, © keeps corpus hmeum of preguancy symthesing progesterone enti] 810 wks and then placenta takes over that fumction Srimalation of histamine Hy receptors: increases secretion of acid by parietal cells GE peptide injected into cerebrospinal ffaid that increases appetite for carbohydrate: somatostaun~ this normally ts made in hypothalamus where it fimerons to inhibit growth hormone. hence anhibiang GH causes hypoglycemia and the body would crave more carbohydrate 1 marease glucose level Ktrial natriuretic peptide: mediated by guanylate cyclase, mereased if left or right amen ss foaded volume ove 95 ‘Note: This material is copyrighted. All rights reserved. Edward Geljan, M.D. 2002 ued | v4 ‘Central diabetes insipidus: myccting vasopressin causes an increase and decrease in unne volume ress teacemtratiea (POS) Location of angiotensin converting enzyme: © pulmonary - Jonmeamsie wah by ACE inhibitors Sara smnioess ‘Vitamin D type in Kidneys: ¢ 1.25 (OH):Ds, Kidney has lochydroxylase & (PTH simulates synthesis), « renal disease MCC of bypovitarsinosis D 7 = Effect of thyroid hormone excess om bone: osttoparosis~ increases bone turnover resulting in loss of bone mass Primary site for temperature regulation: * anterior hypothalamus, « fever is due 10 release of pyrogens simulating IL-i release from macrophages *IL-] mereases synthesis of PGE: in the antenor hypothalamus, *PGE; raises the hypothalame set-point (normal core temperanure viewed as 190 low), hence antenor hypothalamic reactions of heat generation prevail, « hear generating mechanisms if core temperanire 15 below set-pomt include— *mereasing the release of thyroid hormones (imereases metabolic rate), *vasoconstriction of skin vessels (sympatbeac stimulation of ce-receptors in smooth muscle), *sympatbetc stimulation of B-receptors in brown fat (increases metabolic rate and heat production), *shivering (most effective system, center located in posterior hypothalamus, leads to activabon of c abd y motoneurons sioervating skeletal muscle), + best Essipanng mechanisms (coordinated in posterior hypothalamus) if core temperature is above set- point mnclude- “reducing sympathetic tone (vasodilatation) of skin vessels leading to shining of Slood Srough venous plexus m the shin, sincreased sympathetic activity of cholinergic fibers mmervanng sweat glands leading to sweanng. Cause of decrease in jugular Venous pulse on inspiration: + decrease im limg complimce (€.g, sarcoidosis), « decreased volume of air im the alveoli leads to less compression of vessels im the -y Bill up better and drain blood off easier has heme mon, which is ferrous and easy to reabsorb in the small mtestine (primarily the duodent once absorbed into enterocytes, heme is enzymencally degrades to release iron, * most of the iron is diveried to storage as ferritin in the enterocyte (called spofermtn), whiie a smal! amount is delivered to plasma tansfernn, the circulating binding protem ef iroc, plants have non-heme iron. which is tm the fernc state: * non-heme sron first binds to musi in the stornsch (renders 1 soluble}, * the macin-non-heme complex then binds tw proteins on the mucosal surface of the Guodenum for transport into the cytosol where it 1s eazymancally degraded to release ron, * ‘a cytosolic protein called mobilferrin transfers the son to the mucosa) femntin stores or to transferrin in the plasma, ¢ when body stores of iron are replete, mucosal ceils gre shed anth thei stored mucosal ferritin in order t prevent iron overioad and transferrin svathesis in the liver is decreased, « whea body stores are depleted. mos: of the on is directed towards wransierin in the plasma and transferrm synthesis in the liver 1s increased. Know ventricalar volume curve and where different beart sounds are located: c.g.. S; (closure of AV and PV) Renal reabsorption curve: glucose 1s most often asked about Destroy hypothalarons, what hormone increases: prolactin ne dopamine to inhibit Effect of carotid massage: slows beart rate Major site of water reabsorption in the GI tract: in descending order: jejunum. fleum, colon ‘Most effective nephrox site for acid excretion: proximal tubule of the kidney Nephron site for ADH effect: collecting tubule Nephron site for greatest generation of free water/most susceptible to ischemix yok ascending limb mn the medulla Constriction of the efferent arteriole: increases Ure GFR and decreases renal plasma flow Filtration fraction in the glomerulus: FF = GFR/RPF, constrict afferent artenole, no change in FF (¢ GER‘ RPF), constrict efferent arteriole increases FF (TT GER/+ RPE), increase plasma prover 6 Note: This material is copyrighted. All rights reserved. Edward Goljan, MD. 2002 a4 concentration, hence increasing Rec, decreases the FF (1 GFR¢no change RPF); decreasing plasma protein concentration, hence decreasing tcc, mneeases the FF (T GFR/oo change RPF);comsricang the ureter, decreases PBS, hence de Gl the ues lecreasing GFR and the FF without affecting RPE (1 GFR/no ‘Starling equation for GFR ml/min: GFR = Kf [(Poc - Pas) - Roc], where Kf'= wambinin naa Ha, Foc hytroniatic prea tn the gleoeruas copy motes fe Gok rae cout of the capillary: e-g., +45 mm Hog: itis increased if the efferent arteriole is conerictod decreased if the afferent arteriole is constricted), Pas = hydrostatic pressure in Bowman's are tran Hg (pushes fluid into the capillary; ¢.g, -10 men Hg), and gc = the oncotic pressure m the glomerular capillary in sm Hg (brings flud back into the capillary; eg. -19 um Hg); the net filtration is the algebraic sum of the above 3 parameters (in the above example, the net pressure would be -10 + 45 -19 = + 16 mm Hg}fluid should move out of the glomerular capillaries: question: what Starling force changes to produce a net ultrafiltration of zero (answer: noc, which ‘becomes increased [pulls fluid back into the glomerular capillary; using the above example— 10 + 45-35 <0 net pressure itis the only factor that does not rematn constant along the lengih of the capillary owing to differences in Protein concentration; the high ttc at the end of the glomerular capillary extends into the peritubular capillaries that surrounds the nephroas and allows for the reabsorption of solutes). 2, bicarbonate: PCO, 40 mm He is normal, bicarbonate 24 mEq/L is normal 70 7.40 18 Arterial pH «= patent A~ acute respiratory acidosis with very little bicarbonate compensation, * paticat B~ chronic respiratory acidosis where there is more compensation by increasing bicarbouate (metabolic alkalosis) and pH comes closer to normal range, * patient C- metabolic alkalosis where there is compensation by increasing PCO; (respiratory acidosis) » patient D- metabolic acidosis where there is compensation by decreasing PCO, (respiratory allalosis), * patient E~ acute respirmory alkalosis where there is very litle drop in bicarbonate for compensation (metabolic acidosis), patient F~ chronic respiratory alkalosis where there is good compensation by dropping bicarbonate (metabolic acidesis) and pH is closer to normal ‘Negative charge of GBM: heparan sulfate S, heart sound correlates with C wave in the jugular venous palses (C wave is closare of the tricuspid valve); know the jugalar venous pulses: see box in cardiovascular chapter on physical diagnosis ‘Carotid massage: + decreases heart rate and increases vasodilatation (carotid sinus barorecepiot tnnetvated by the IX and Xth nerve, + impulses generated in this receptor imhibit toni discharge of 7 : This material is copyrighted. All rights reserved. Edward Goljan, M.D. 2002 the vasoconstrictor nerves and excite vagal innervation of the heart ' venodilation, drop in blood pressure and heart rate, and a decrease in catenden lation, 3% Bypertonic saline and effect on POsm and ADH levels: + 3% hypertonic saline increases ‘POsm, © increased POsm stimulates the release.of ADH (increases)_».atrial natriuretic peptide is also stimulated and normally does inhibit ADH release, however, bypertonicity overrides ANP Patient walking briskly on a hot day: no increase in rectal temperature, vasodilatation of vessels in skin Marathon runner om 2 hot day: * increase in rectal temperature, © vasodilatation of vessels in skin Vagus nerve functions in the stomach: * increases gastric acid secretion and gastric motility— vagotomy decreases acid secretion and motility, « the vagus nerve represents the parasympathetic innervation of the stomach and duodenum Sympathetic nervous system in the stomach/duodenum: » inhibits secretion and motility, + sympathetic fibers are located in the ccliac plexus from T5-T9 Motilin: hormone modulating GI muscle activity 0.9% normal saline with KCI is the crystalloid solution that is most often used to replace gastric loses Creatinine clearance (CCr): « creatinine clearance formula~ * CCr= UOsm x V / POsm, where V = volume of a 24 hr urine collection in mL/min, + creatinine is not a perfect clearance substance, © causes of a decreased CCr- * increasing age, * inadequate 24 hr urine collection : decreases urine volume in the numerator, * renal failure, + causes of an increased CCr- * normal pregnancy: increase in plasma volume increases GFR and Cr, + carly diabetic nephropathy: FENa’ (fractional exeretion of sodium): * useful in the workup of oliguria~ FENa® = (UNa” x PCr) / (PNa” x UCr) x 100, « values <1 indicate intact tubular function, « values > I(usually > 2) indicate tubular dysfunction Myocardial physiology: « cardiac hypertrophy increases wall stress, sinus tachycardia reduces filling of the coronary arteries, * increasing heart rate raises myocardial oxygen consumption- cardiac muscte uses B-oxidation of fatty acids for energy, + venoconstriction— increases preload ‘alone, « restrict salt and water intake decreases preload alone, * increase cardiac contractility~ no change in preload/afterload Normal gas physiology at the tissue level: « CO; derived from tissue enters the RBC and combines with H,O via carbonic anhydrase to form H,COy, # HCOs dissociates into H* and HCOy the latter leaving the RBC in exchange for Cl" anions, « H+ combines with oxygenated Hgb (Hgb- ;), which releases-O; (Bohr effect), © Op leaves the RBC, dissolves in plasma and increases capillary PO Normal gas physiology at the pulmonary level: + Alveolar O; diffuses into the plasma of the pulmonary capillary owing to a greater partial pressure of alveolar O2, © O enters the RBC and combines with ferrous ions on deoxyhemoglabin to form oxyHgb and H*, « HCO,’ enters the RBC from the plasma and combines with H+ to form H;COs, ¢ Cl° anions leave the RBC to counterbalance the entry of HCO,", © HCO, dissociates into CO; and H,O; CO; leaves the RBC and dissolves in the plasma to increase the PCO;, which enters the alveoli for excretion Important anatomical considerations for urine control: ¢ urogenital diaphragm- + muscles of the urogenital diaphragm are the deep transverse perineal and sphincter urethra muscle, + location of the urogenital diaphragm on a cystourethrogram is just distal to the prostate in a male, * locaton of the urogenital diaphragm on a cystourethrogram in a female is the beginning of urethra where it ‘exits the bladder, « functions of the detrusor muscle~ * relaxed: storage of urine in the bladder, * contracted: emptying of bladder, « functions of the sympathetic in bladder conol~ + relaxes the detrusor muscle: aids in urine storage in the bladder, * contracts internal sphincter: increases urine 98 Note: This material is copyrignted. All rights reserved. Edward Goljam, M.D. 2002 qa 44 a4 storage, + functions of the parasympathetic system—* contracts the detrusor muscle: empti bladder, * relaxes the intemal sphincter muscle by blocki c fae) complies te erage of be bladder by blocking sympathetic inhibition: alliws Thyroid hormone: © diffuses into cytosol, * binds to receptors in the : domain has zinc fingers, other hormones with Himilar binding: wevide,reinahs cake ‘Angiotensinogen: + synthesized in liver, + increased synthesis from estrogen Action potential curve of cardiac muscle: know what part corresponds to the T wave (right atthe end) Ventricular and zortic pressure curve: where is S; (beginning of diastole) Substance requiring ATP for reabsorption in the small bowel: « glucose, » ATP required for cotransport with Na” Major vehicle for carrying CO; in blood: bicarbonate (~70%) Volumes/capacities not directly measured by spirometry: « residual volume (RV): volume of air left over after maximal expiration, © total lung capacity (TLC): amount of air in a fully expanded lung, measured with a nitrogen or belium dilution method, + fanctional residual capacity (FRC): total amount of air in the lungs at the end of a normal expiration (end of the tidal volume [TV]), obtained by a helium dilution technique or body plethysmography Forced vital capacity (FVC), forced expiratory volume in 1 second (FEV...) and FEV,,./FVC: + FVC is the total amount of air expelled after a maximal inspiration: the RV is what is left over at the end of maximal expiration: normal FVC is 5 liters, « forced expiratory volume in 1 second (FEV t1ec): FEV ise is how much air a person can expel from the lungs in 1 second after a maximal inspiration, normal FEV nc is 4 liters, « ratio of FEVi.q/FVC is normally 0.80: 4 liters/S liters = 0.80, » peak expiratory flow meter: outpatient method of evaluating forced vital capacity (FVC), commonly used by asthmatics to evaluate their airways ‘Method of measuring RV: « subtract the expiratory reserve volume (ERV) from the finctioual residual capacity (FRC), * ERV is the amount of air forcibly expelled at the end of a normal expiration (end of the TV), « FRC = ERV + RV, therefore, RV = FRC - ERV Diffusion capacity (Dlco): * Dco is the method of measuring the ability of a gas to diffuse rough the alveolar/capillary interface: utilizes carbon monoxide (CO), * Deo is primarily dependent on the following parameters: CO reaching the alveoli (decreased due wo V/Q tmusmatches: ¢.g., atelectasis, COPD), CO crossing the alveolaricapillary interface (decreased die to pulmonary fibrosis or fluids in the interface) 99 FIGURE 11-1. Schematic of the normal lung volumes and ca pacities and the forced expiratory volame 1 second (FEV._) and forced vital capacity (FV) in a normal person (A), a person with restrictive lung disease (B), and a person with obstructive Tung disease (C). FRC (hinctional residual capacity) represents the volume of gas that remains in the lung at rest at the end of a normal respiration and is the most reproducible part of a pulmo- nary function test, since it does not require patient effort. Person A (normal individual) has an FEVis, of 4 L and an FVC of 5 L, with FEV./FVC ratio = 0.80. Person B with restrictive lung disease has a “miniaturized” curve of person A. Note that the FEViyee and FVC are the same (3 L) owing to the increase in elasticity in the lungs, hence FEV,./FVC ratio = 1.0. Person C with the obstructive pattern is having difficulty in expelling air from the lungs owing to decreased lung elasticity. FEVine = 11, FVC = 3 L, and FEV,,,/FVC ratio = 0.33. Both people B and C have reduced values for FEV. and FVC when compared with tidal volume: VC. vital capacity. A= Aldosterone “1,0 = free water: mostly generated in Na‘/K"/2CT cotransport pamp ADH = antidiuretic hormone : TAL = thick ascending limb (diluting segment) DCT = distal convoluted tubule 1 ~ hydrogen ion that can be secreted with HPO, (HjPO,), which is titratable acidity or NHo (NEAT) Proximal tubule: + carbonic anhydrase inhibitor- blocks reclamation of bicarbonate, which is excreted as ‘NaHCO, or KHCOs: ¢ proximal renal tubular acidasis, ¢1oss of K° and Na Ascending tubule: loop dinretic- blocks Na'/K 72CT cowansport pump: # impairs generation of Bes We ‘ calefum salvo lost in the urine (Rx of hypercalcemia), +]hyponatremia, hypokalemia, metabolic alkalosis « aan juretie- blocks Na-/Cl pump: allows calcium reabsorption with the help of paathormone (useful in calcium stone formers), # hyponatremia, hypokalemia, metabolic alkalosis Distal/ollecting tubule: « spironolactone~ blocks aldosterone (A) pumps: ¢ blocks Na"/K” exchange PAP. ‘blocks EL/K" ATPase exchange pump, #" sparer, ¢normal anion gap metabolic acidosis Functional Aspects of the Nephron

Anda mungkin juga menyukai